Anda di halaman 1dari 136

Apresentao

Caros professores orientadores e alunos do Estgio 2a edio da OBMEP Este um nmero especial da Revista do Professor de Matemtica RPM, elaborado para utilizao no Estgio 2a edio da OBMEP, a ser realizado a partir do primeiro semestre de 2007. A RPM, como seu nome diz, uma revista dedicada aos professores de Matemtica da educao bsica, a alunos e professores de cursos de licenciatura em Matemtica e a todos aqueles que se interessam pela Matemtica do nvel mdio. O tratamento dado aos temas abordados procura ser acessvel e agradvel, sem sacrificar o rigor. A revista uma publicao da Sociedade Brasileira de Matemtica SBM e tem sido editada e distribuda sem interrupes desde 1982. A revista publica crnicas, artigos e sees, como Problemas, O leitor pergunta, Livros, Olhando mais de cima, etc. Nos artigos, temas interessantes de nvel elementar ou avanado so apresentados de modo acessvel ao professor e ao aluno do ensino mdio ou de cursos de Licenciatura em Matemtica. Uma experincia interessante em sala de aula, um problema que suscita uma questo pouco conhecida, uma histria que merea ser contada ou at uma nova abordagem de um assunto conhecido podem compor um artigo da revista. Nas sees, a revista conversa com os leitores, publicando problemas e/ou solues propostas por eles, cartas, resenhas de livros, erros encontrados em textos didticos, etc., sempre visando ao aperfeioamento do trabalho do professor na sua sala de aula.
i

Para este exemplar especial, o Comit Editorial da RPM escolheu artigos que pretendem ampliar o conhecimento dos alunos em diferentes tpicos, bem como temas que motivem discusses ou satisfaam a curiosidade terica e histrica de alunos interessados em Matemtica. Por exemplo, as cnicas so tratadas de modo prtico no texto Sorrisos, sussurros, antenas e telescpios; a intuio desafiada em diferentes situaes no texto Quando a intuio falha; a anlise combinatria utilizada para discutir a funcionalidade da brincadeira Amigo oculto (ou secreto), etc. Apresentamos tambm uma seleo de 30 problemas, cuidadosamente escolhidos entre os publicados na seo Problemas, que abrangem a maioria dos tpicos do ensino mdio. As solues dos problemas propostos esto no fim da revista. Para o ensino fundamental, e tambm para o ensino mdio, selecionamos 30 ...probleminhas, parte integrante da seo Problemas dos nmeros usuais da revista. Os probleminhas so caracterizados por exigir muito pouco conhecimento de contedo especfico, apenas raciocnio lgico-dedutivo e domnio de operaes elementares. a parte ldica, permitindo que professores e alunos se divirtam, resolvendo problemas desafiadores, e se sintam realizados ao obter as solues. As respostas dos probleminhas tambm esto no final da revista. Os artigos aqui apresentados no esto com as referncias bibliogrficas, que podem ser encontradas nos exemplares originais da RPM. Comit Editorial da RPM

ii

Contedo
Como escolher namorada pelo horrio dos trens ................ 1 3

Quando a intuio falha . . . . . . . . . . . . . . . . . . . . . . . . . . . . . . . . . . .

Eleies preferncia transitiva? . . . . . . . . . . . . . . . . . . . . . . . . . 9 A divisibilidade e o dgito verificador . . . . . . . . . . . . . . . . . . . . . . . . 11 O tamanho da Terra . . . . . . . . . . . . . . . . . . . . . . . . . . . . . . . . . . . Problema das idades . . . . . . . . . . . . . . . . . . . . . . . . . . . . . . . . . . . . Fraes egpcias . . . . . . . . . . . . . . . . . . . . . . . . . . . . . . . . . . . . . . As dzimas peridicas e a calculadora Mania de Pitgoras ....................... ................................... ............ 16 21 27 30 34 40 45 54 65

A ilha dos sapatos gratuitos . . . . . . . . . . . . . . . . . . . . . . . . . . . . . . . . . 23

Usando reas . . . . . . . . . . . . . . . . . . . . . . . . . . . . . . . . . . . . . . . . . Trigonometria e um antigo problema de otimizao

Vale para 1, para 2, para 3, ... Vale sempre? . . . . . . . . . . . . . . . . . . . . 48 Semelhanas, pizzas e chopes . . . . . . . . . . . . . . . . . . . . . . . . . . . . . A Matemtica do GPS . . . . . . . . . . . . . . . . . . . . . . . . . . . . . . . . . . . Sorrisos, sussurros, antenas e telescpios . . . . . . . . . . . . . . . . . . . . . . . 58 O problema do amigo oculto . . . . . . . . . . . . . . . . . . . . . . . . . . . . . . . . . 72 O princpio da casa dos pombos . . . . . . . . . . . . . . . . . . . . . . . . . . . . . . . 77 Probabilidade geomtrica: os problemas dos ladrilhos, do encontro e do macarro . . . . . . . . Alguns problemas clssicos sobre grafos . . . . . . . . . . . . . . . . . . . . . . Srie harmnica . . . . . . . . . . . . . . . . . . . . . . . . . . . . . . . . . . . . . . . . . . Problemas . . . . . . . . . . . . . . . . . . . . . . . . . . . . . . . . . . . . . . . . . . . . 83 87 95 104

O que tem mais: racionais ou naturais? . . . . . . . . . . . . . . . . . . . . . . . . . 100 ...probleminhas . . . . . . . . . . . . . . . . . . . . . . . . . . . . . . . . . . . . . . . . . . 108 Solues dos problemas . . . . . . . . . . . . . . . . . . . . . . . . . . . . . . . . . . . 113 Respostas dos ...probleminhas . . . . . . . . . . . . . . . . . . . . . . . . . . . . . . . 131
iii

iv

Como escolher namorada pelo horrio dos trens

Joo amava Lcia, que amava Joo. S que Joo, alm de amar Lcia, tambm amava Letcia e tentava namorar as duas ao mesmo tempo. Durante a semana, at que dava, mas quando chegava o sbado noite era terrvel. As duas queriam Joo e este no possua o dom da presena ao mesmo tempo em dois lugares. Assim, alternadamente, ou Lcia ou Letcia ficava sem sair com Joo, nos embalos de sbado noite. Honesto, Joo decidiu informar Lcia sobre a existncia de Letcia e Letcia sobre Lcia. Com choros e lamrias de todos os lados, Joo continuou dividido, sem saber quem escolher. Joo usava como meio de transporte os trens metropolitanos. Para visitar Lcia, Joo pegava trens que iam no sentido da direita e para visitar Letcia pegava trens que iam para a esquerda. Quanto a horrios no havia dvidas: trens para cada lado de meia em meia hora. Mas como escolher entre Lcia e Letcia? Letcia, que era professora de Matemtica, props a Joo um critrio justo, equnime, salomnico para escolher entre as duas namoradas. A proposta foi: Joo iria para a estao de trens sem nenhuma deciso. Ao chegar pegaria o primeiro trem que passasse, fosse para a direita, fosse para a esquerda. Proposta aceita, Joo comeou a usar esse critrio aparentemente justo e aleatrio. Depois de usar o critrio por cerca de trs meses, descobriu que visitara Letcia muito mais que Lcia, e, se a sorte quis
1

assim, ficou com Letcia e com ela se casou sem nunca haver entendido por que a sorte a privilegiara tanto. S nas bodas de prata do seu casamento que Letcia contou a Joo a razo de o trem a ter escolhido muito mais vezes que a concorrente. Letcia estudara os horrios dos trens e verificara que os horrios eram: Trens para a esquerda (Letcia): 8h00; 8h30; 9h00; 9h30; ... Trens para a direita (Lcia): 8h05; 8h35; 9h05; 9h35; ...

Ou seja, considerando, por exemplo, o intervalo de tempo, 8h00 8h30, o horrio H de chegada na estao, que faria Joo tomar o trem para a direita, deveria ser tal que 8h00 < H < 8h05. Se 8h05 < H < 8h30, Joo pegaria o trem para a esquerda. A situao se repete em qualquer outro intervalo de 30 minutos: 25 minutos so favorveis ao trem da esquerda e 5 minutos ao da direita. Na guerra como no amor tudo vale..., at usar Matemtica.

Baseado no artigo Como escolher namorada pelos horrios do trem do subrbio Manoel Henrique Campos Botelho, RPM 14

Quando a intuio falha

Problema 1 Suponhamos que seja possvel colocar uma corda circundando a Terra, ajustando-a ao equador. Em seguida, retiramos essa corda, aumentamos 1 m no seu comprimento e a recolocamos em volta da Terra, formando uma circunferncia concntrica com o equador. Sabendo que o raio da Terra aproximadamente igual a 6 355 000 m, teramos substitudo uma corda de aproximadamente 2 x 3,14 x 6 355 000 m = 39 909 400 m por uma de 39 909 401 m. Assim, teremos um vo entre o equador e a corda, ou melhor, uma diferena d entre os raios das duas circunferncias. Ento, perguntamos: usando-se somente a intuio, qual o valor aproximado de d? Ou seja, qual a largura aproximada desse vo entre o equador e a corda? Cremos que o leitor dir: no existe vo algum... desprezvel essa diferena... Como a Terra to grande e s se aumentou um metro na corda, claro que o vo muito pequeno e, por conseguinte, desprezvel... Ser? Soluo Vamos calcular o valor de d: 2R 2RT = 1 ou d = R RT = 1/2 0,16 m = 16 cm! Notamos que d independente do raio, independente, portanto, do comprimento da circunferncia. Que tal fazer algumas experincias?
3

Problema 2 Passemos, agora, ao segundo exemplo: consideremos um crculo com raio igual ao raio da Terra. Suponhamos ser possvel cobrir toda a superfcie desse crculo por uma outra superfcie, modelvel, ajustada a ele. Retiramos, em seguida, essa segunda superfcie, aumentamos sua rea de 1 m2 e a remodelamos, at se transformar novamente num crculo, com rea 1 m2 maior. Em seguida, justapomos os dois discos de modo a obter dois crculos concntricos. Assim, haver uma diferena D entre os raios dos dois crculos. Perguntamos novamente: usando-se apenas a intuio, qual o valor aproximado de D? Cremos que o leitor, dessa vez, alertado pelo problema anterior, teria maior cautela para emitir um juzo, baseado apenas em sua intuio. Deixamos o clculo de D para o leitor que deve concluir que, agora, D depende do raio e que decresce na medida em que o raio cresce. Problema 3 Tome uma corda esticada, medindo 400 km, unindo dois pontos, A e B, um em SP e outro no RJ. Tome outra corda com 1 m a mais do que a anterior e fixe suas extremidades nos mesmos pontos A e B. Como ela fica bamba, coloque uma estaca de modo a mant-la esticada. Considere a estaca no a) ponto mdio da corda. b) ponto A correspondente a SP. Qual a altura, h, dessa estaca? maior ou menor que 1 m? a)
A SP
200 000,0 5
h

B RJ

200 000

b)
h

400 001 - h
400 000

A SP
4

B RJ

Soluo a) No tringulo retngulo de hipotenusa medindo 400 001/2 m e cateto maior medindo 400 000/2, temos, por Pitgoras: (200 000,05)2 200 0002 = h2, logo, h2 = (200 000,05 200 000)( 200 000,05 + 200 000), ou h 447 m. Ou seja, a estaca da altura de um prdio de aproximadamente 127 andares! b) Neste caso, o tringulo retngulo tem cateto maior medindo 400 000 m, e a soma dos comprimentos da hipotenusa e do cateto menor, h, igual a 400 001 m. Por Pitgoras: 400 0002 + h2 = (400 001 h)2 ou h = 0,999 m 1m! Perplexos com os resultados? Problema 4 Quantos quadrados so necessrios para cobrir o Brasil, supondo o processo indicado na figura em que o quadradinho inicial tem 1 cm de lado e o quadrado externo tem lado igual a 4.500 km? Antes de resolver, faa estimativas do resultado e compare com os palpites de seus colegas. Soluo 1o quadrado: 1 cm de lado 3o quadrado: 2 cm de lado 5o quadrado: 4 cm de lado ... ... (2n + 1)o quadrado: 2n cm de lado. Por tentativas, verifica-se que 229 = 536 870 912 a primeira potncia maior que 450 000 000 (4 500 km = 450 000 000 cm). Portanto, o (2 x 29 +1)o = 59o quadrado j cobre o Brasil. Podemos resolver o problema de modo mais formal, usando que os lados de todos os quadrados:
5

1,

2 , 2, 2 2 , 4, 4

2 , etc.

formam uma progresso geomtrica de razo 2 logo, queremos determinar o menor inteiro n tal que n 1 > x, sendo x tal que

( 2)
ou,
x = log
2

= 450 000 000

450 000 000

2(log 4, 5 + 8) 57, 5 e n = 59. log 2

Portanto, o 58o quadrado no cobre o Brasil, mas o 59o, sim. Este mesmo problema pode ser resolvido com hexgonos e pentgonos. Que tal tentar? Vejamos agora o que diz nossa intuio na lenda: O jogo de xadrez Segundo uma lenda antiga, o jogo de xadrez foi inventado na ndia, para agradar a um soberano, como passatempo que o ajudasse a esquecer os aborrecimentos que tivera com uma desastrada batalha. Encantado com o invento, o soberano, rei Shirham, quis recompensar seu sdito Sissa Ben Dahir, o inventor do xadrez. Shirham disse a Sissa que lhe fizesse um pedido, que ele, rei Shirham, o atenderia prontamente. Sissa disse, simplesmente: Bondoso rei, d-me ento um gro de trigo pela primeira casa do tabuleiro, dois pela segunda casa, quatro (= 22) pela terceira, oito (= 23) pela quarta, e assim por diante, at 263 gros de trigo pela ltima casa do tabuleiro, isto , a 64a casa. O rei achou esse pedido demasiado modesto e, sem dissimular seu desgosto, disse a Sissa: Meu amigo, tu me pedes to pouco, apenas um punhado de gros de trigo. Eu desejava cumular-te de muitas riquezas: palcios, servos e tesouros de ouro e prata. Como Sissa insistisse em seu pedido original, o rei ordenou a seus auxiliares e criados que tratassem de satisfaz-lo. O administrador do palcio real mandou que um dos servos buscasse um balde de trigo e fizesse logo a contagem. Um balde com cerca de 5 kg de trigo contm aproximadamente
6

115 000 gros (como o leitor pode verificar, fazendo, ele mesmo, a contagem...); foi o suficiente para chegar 16a casa do tabuleiro, mas no alm, pois 1 + 2 + 22 + 23 + . . . + 215 = 216 1 = 65 535*, enquanto, para chegar 17a casa, seriam necessrios 1 + 2 + 22 + 23 + . . . + 216 = 217 1 = 131 071 gros de trigo. (Um fato interessante a observar: o nmero de gros de trigo a colocar numa casa igual a todos os gros j colocados nas casas precedentes mais 1. De fato, pelo penltimo clculo v-se que todos os gros colocados at a 16a casa mais 1 216, que o nmero de gros correspondentes 17a casa.) Traga logo um saco inteiro (60 kg, aproximadamente 1 380 000 gros) ordenou o administrador a um dos servos , depois voc leva de volta o que sobrar. Ao mesmo tempo providenciou a vinda de mais uma dezena de contadores de trigo para ajudar na tarefa, que se tornava mais e mais trabalhosa. O administrador, os servos e os contadores j haviam terminado com 10 sacos de trigo (= 10 x 60 x 23 000 = 13 800 000 de gros) e mal haviam passado da 23a casa do tabuleiro, visto que 1 + 2 + 22 + 23 + ... + 222 = 223 1 = 8 388607 1 + 2 + 22 + 23 + ... + 223 = 224 1 = 16 777215. A essa altura o rei foi notificado do que estava acontecendo e alertado de que as reservas do celeiro real estavam sob sria ameaa. Insistindo, porm, em atender ao pedido de seu sdito, ordenou que o trabalho continuasse.

Estamos usando o seguinte resultado: dado um nmero q 1 e n um inteiro positivo arbitrrio, seja S = 1 + q + q2 + q3 + . . . + qn, logo qS = q + q2 + q3 + q4 + . . . + qn+1. Subtraindo a primeira igualdade da segunda, obtemos qS S = q n +1 1, ou S = q n +1 1 , que a frmula da soma usada, neste texto, para q 1

q = 2 (frmula da soma dos termos de uma progresso geomtrica).


7

Mandou convocar mais servos e mais contadores; ao mesmo tempo, mandou chamar os melhores calculistas do reino para uma avaliao do problema. Esses vieram e, cientificados do que se passava, debruaram-se nos clculos. Em menos de uma hora de trabalho, puderam esclarecer o rei de que no havia trigo suficiente em seu reino para atender ao pedido de Sissa. Mais do que isso, em todo o mundo conhecido na poca no havia trigo suficiente para atender quele pedido! No tempo em que isso aconteceu, pensava-se que o mundo fora criado havia menos de 5 000 anos. Assim, os calculistas do rei puderam dizer-lhe que nem mesmo toda a produo mundial de trigo, desde a criao do mundo, seria suficiente para atender ao pedido de Sissa. Vamos ver por qu. O nmero de gros pedidos por Sissa: 1 + 2 + 22 + 23 + ... + 263 = 264 1 = 18 446 744 073 709 551 615, valor obtido usando uma calculadora cientfica. Como verificamos no incio, um balde de 5 kg de trigo contm 115 000 gros, logo 1 tonelada de trigo (200 baldes) contm 23 x 106 gros. A produo mundial de trigo da ordem de 590 milhes de toneladas (Internet), ou seja, 23 x 590 x 1012 gros. Ora, 264 1 dividido por esse nmero de gros resulta aproximadamente 1360, isto , seriam necessrios 1360 anos de produo mundial de trigo no nvel de hoje para atender ao pedido de Sissa. Incrvel, no ?!
Baseado nos artigos Logaritmos um curso alternativo Renato Fraenkel, RPM 4 Quando a intuio falha Joel Faria de Abreu, RPM 8 De So Paulo ao Rio de Janeiro com uma corda ideal Geraldo G. Duarte Jr., RPM 22 Nmeros muito grandes Geraldo vila, RPM 25

Eleies - preferncia transitiva?

Antes de qualquer eleio nacional importante, sempre so feitas pesquisas, que a populao acompanha com interesse, em inmeros setores da sociedade: empresas, clubes, escolas, etc. Vou falar aqui de uma pesquisa feita em uma escola, antes do primeiro turno de uma eleio para presidente da Repblica. A histria comeou quando ouvi um colega, professor de Histria, conversando com os alunos de uma turma da 3a srie do ensino mdio. Todos eleitores, naturalmente. Perguntava esse meu colega em quem eles votariam no segundo turno, considerando as hipteses, que ele iria apresentar, em relao aos trs cadidatos principais, que chamarei aqui de A, B e C. Esse meu colega perguntou ento para a turma em quem eles votariam se A e B fossem para o segundo turno. E a maioria da turma votaria em A. Em seguida ele perguntou em quem votariam se B e C fossem para o segundo turno. E agora a maioria da turma votaria em B. Dando-se por satisfeito, o professor resolveu comear a aula, mas foi interpelado por um aluno, que lhe perguntou se ele no iria propor a hiptese de A e C irem para o segundo turno. Esse colega respondeu que no havia necessidade dessa pergunta porque naturalmente A ganharia de barbada. A aula comeou e eu me retirei para pensar no caso que agora relato. Na realidade, por incrvel que parea, o professor estava errado. Ele no poderia concluir que a maioria da turma preferiria A a C. Para mostrar que esse raciocnio falso, imaginemos que num grupo de pessoas a disputa entre A, B e C seja equilibrada da
9

seguinte forma: 1/3 das pessoas desse grupo tem preferncia por A, B e C nessa ordem; 1/3 das pessoas tem preferncia por B, C e A nessa ordem, e o restante por C, A e B nessa ordem.

1/3 1/3 1/3

1 A B C

2 B C A

3 C A B

Se esse grupo for submetido s perguntas feitas pelo meu caro colega, veremos que, na deciso entre A e B, 2/3 preferiro A; tendo que optar entre B e C, 2/3 preferiro B; mas, surpreendentemente, se a deciso for entre A e C, 2/3 preferiro C! O aluno estava, portanto, certo e a terceira pergunta deveria ter sido feita. Temos aqui um exemplo de uma relao que intuitivamente esperamos ser transitiva, mas que, na realidade, no . Divagando um pouco, essa notransitividade da relao preferir pode ter espantado algum dia um cozinheiro de restaurante que s sabia fazer trs pratos: um peixe, uma galinha e uma carne, mas, como nunca tinha tempo de fazer os trs, sempre oferecia dois deles. perfeitamente possvel que, quando havia peixe e galinha, a maioria dos fregueses preferisse peixe. No dia em que havia galinha e carne, a maioria preferisse galinha e que no dia em que havia peixe e carne a maioria preferisse carne! Isso pode ocorrer mesmo que os fregueses sejam sempre os mesmos. natural. Para dar um outro exemplo (as mulheres agora me perdoem), diria que o espanto do cozinheiro pode ser comparado ao da moa que recebeu pedido de casamento de trs pessoas A, B e C. Essa moa, que desejava fazer o melhor casamento possvel (na opinio dela, naturalmente), dava importncia igualmente a trs coisas que os candidatos deveriam ter: cultura, beleza e situao financeira. Para melhor avaliar os pretendentes, ela resolveu dar notas a esses quesitos para cada um deles. Nota 3 significando bom; nota 2 significando mdio e nota 1 para ruim. Os resultados esto no quadro:
A B C cultura beleza finanas 2 3 1 3 2 1 3 1 2

Veja ento que, apesar de haver um empate tcnico, se os candidatos fossem comparados aos pares, ela iria preferir A a B porque A vence em dois dos trs quesitos; iria preferir B a C pela mesma razo e ainda iria preferir C a A. Incrvel, no?
Baseado no artigo Eleies Eduardo Wagner, RPM 16
10

A divisibilidade e o dgito verificador

Introduo Recentemente fui obrigado a solicitar uma segunda via do meu documento de identidade e, para minha surpresa, acrescentaram um dgito ao final do meu antigo nmero de registro geral (RG). Na ocasio, fiquei curioso: quais as razes desse dgito adicional? Esclarecimento que s recentemente obtive e que compartilho com o leitor neste artigo. Sistemas de informao e a segurana na transmisso de dados Por mais cuidadoso que seja o digitador, erros podem ocorrer e suas conseqncias podem ser muito srias. preciso, ento, criar mecanismos para detectar o maior nmero possvel de tais erros. Pesquisas recentes sobre a natureza dos erros de digitao revelam um fato curioso. Cerca de 79% dos erros ocorrem com a digitao equivocada de um nico dgito (ou algarismo), como, por exemplo, digitar 1 573, quando o correto seria 1 673. Esse tipo de erro recebe o nome de erro singular. Outros 11% dos erros, chamados de erros de transposio, referem-se troca de dois dgitos (ou algarismos), como, por exemplo, escrever MTAEMTICA, quando o correto seria MATEMTICA. Esses so chamados de erros de transposio. Os demais
11

10% dos erros esto distribudos em diversas categorias, nenhuma delas representando mais de 1% do total. bom que fique claro que existem particularidades em cada sistema de cdigos, ou at mesmo em cada idioma, que podem mudar significativamente essa distribuio de probabilidades. Apenas para citar um exemplo, na Sucia os nmeros de identificao de cada cidado so constitudos por 6 algarismos para a data de nascimento (ano/ms/dia), seguidos de 3 algarismos para dar conta de duplicaes de datas coincidentes. Muitas pessoas, no entanto, ao digitar, permutam os algarismos do ano com os do dia, criando um erro muito freqente, que no singular nem de transposio (trata-se aqui de um erro de trocas duplas). Sabendo-se que nos dias de hoje cada vez mais usamos os computadores para armazenar e processar as informaes digitadas, seria possvel criar um sistema que pudesse identificar com 100% de segurana um erro de digitao do tipo singular ou de transposio? Um tal sistema daria conta de evitar cerca de 90% dos erros mais freqentes de digitao. A divisibilidade e uma soluo do problema O sistema ISBN (International Standard Book Number), criado em 1969 para a identificao numrica de livros, CD-Roms e publicaes em braille, talvez seja um dos pioneiros na utilizao de um dgito de verificao ao final de cada cdigo, capaz de resolver o problema dos erros singulares e de transposio. Por exemplo, o cdigo ISBN 97-26-62792-3 refere-se ao livro O mistrio do bilhete de identidade e outras histrias (Editora Gradiva, Lisboa, 2001). Com exceo do ltimo dgito da direita, que o dgito verificador (DV) (ou dgito de controle, como conhecido em Portugal), os demais 9 dgitos so responsveis por identificar o pas de origem da obra, a editora e o livro propriamente dito. Os equipamentos que recebem a digitao de um cdigo ISBN, x1 x2 x3 x4 x5 x6 x7 x8 x9 e seu dgito de verificao x10, esto programados para verificar se o resultado, S, da expresso 10 x1 + 9 x2 + 8 x3 + ... + 2 x9 + 1 x10 divisvel por 11 ou no: o algarismo de verificao x10 escolhido de tal forma que o resultado dessa conta tenha sempre resto zero na diviso por 11. Veja, no exemplo do livro acima, que
12

10 9 + 9 7 + 8 2 + 7 6 + 6 6 + 5 2 + 4 7 + 3 9 + 2 2 + 1 3 igual a 319, que divisvel por 11. Podemos demonstrar um importante resultado com relao a esse sistema: Resultado Se ocorrer na leitura de um cdigo ISBN um, e apenas um, dos dois erros (singular ou de transposio), ento a soma S no ser um mltiplo de 11. Demonstrao Caso 1: Quando ocorre um erro singular. Seja x1 ... xi ... xj ... x10 um cdigo ISBN com dgito de verificao x10
* e x1 K xi K x10 o resultado da ocorrncia de um erro singular na i-sima posio. Chamemos de S e S* as somas correta e errada, respectivamente. Temos, evidentemente, que S divisvel por 11 e

S * S = (11 i )( xi* xi ) 0.

Se admitirmos por hiptese que S* seja mltiplo de 11, ento, como 11 primo, conclumos que 11 divide 11 i ou divide xi* xi , o que um absurdo, pois 11 i e xi* xi so nmeros inteiros no nulos entre 10 e 10. Logo, S* no mltiplo de 11, o que acusa o erro cometido. Caso 2: Quando ocorre um erro de transposio. Seja x1 ... xi ... xj ... x10 um cdigo ISBN, x10 o dgito de verificao e x1 ... xi ... xj ... x10 o resultado da ocorrncia de uma transposio dos algarismos xi e xj nas posies i e j (i j). Nesse caso, a diferena S* S igual a (11 i)xj + (11 j)xi (11 i)xi (11 j)xj = (j i)(xj xi) 0. A hiptese de S* ser mltiplo de 11 mais uma vez absurda porque nos conduziria concluso de que um dos nmeros j i ou xj xi, que so nmeros inteiros no nulos entre 10 e 10, mltiplo de 11. Segue que S* no pode ser mltiplo de 11. Se agora admitirmos que na digitao de um cdigo ISBN s ocorrem erros singulares ou de transposio, no mais do que um erro em cada
13

nmero, ento no ocorrem erros na digitao de um cdigo ISBN se e somente se a soma S for um mltiplo de 11. bom lembrar que, ao digitarmos um cdigo ISBN cometendo um erro singular ou de transposio, o equipamento que recebe os dados ser capaz apenas de acusar a existncia de um erro devido ao fato de S no ser divisvel por 11, mas no ser capaz de encontr-lo; o que implica dizer que o digitador tem ainda como tarefa procurar o erro cometido. O dgito de verificao do RG Para o Estado de So Paulo e muitos outros Estados brasileiros, o dgito de verificao do RG calculado da seguinte maneira: Seja x1x2x3x4x5x6x7x8x9 o RG de um indivduo. O dgito de verificao, x , calculado de modo que a soma
10

100 x10 + 9 x9 + 8 x8 + 7 x7 + ... + 2 x2 + 1 x1 seja divisvel por 11. Como normalmente se reserva apenas um algarismo para o dgito de verificao, que, neste caso, um inteiro entre 0 e 10 (os restos possveis na diviso de um inteiro por 11), normalmente se usa a letra X para representar o dgito de verificao 10. Por exemplo, no RG nmero 25 135 622 X, verifique que 100 10 + 9 2 + 8 2 + 7 6 + 6 5 + 5 3 + 4 1 + 3 5 + 2 2 + 1 0 divisvel por 11. Observa-se que os raciocnios utilizados na demonstrao do Resultado anterior, aplicam-se quase totalmente nova expresso aqui utilizada. Com efeito, na ocorrncia de um erro singular no dgito xi na digitao de um tal RG, tem-se S* S = i (xi* xi) para i = 1, 2, ..., 9 e se i = 10, S* S = 100 (x10* x10), que no podem ser mltiplos de 11 para xi* xi 0, i = 1, 2, ..., 10. Na ocorrncia de um erro de transposio entre xi e xj, com 1 i < j 9, tem-se S* S = (j i)(xi xj), que no divisvel por 11, se xi xj 0. No caso, entretanto, em que a transposio se d entre xi e x10, S* S = (100 i) (xi x10), que um mltiplo de 11 se i = 1, mesmo que x1 x10 no seja nulo. Isso no tem efeito prtico negativo, pois erros de transposio de alta probabilidade so aqueles entre dgitos consecutivos. A troca, portanto, entre o primeiro e ltimo dgitos no nada comum. J em Portugal, onde o algoritmo de verificao dos documentos de identificao igual ao nosso, com a diferena de que l se utiliza peso 10
14

no dgito de verificao em vez de peso 100, esse problema no se d. Os responsveis pela execuo do sistema decidiram, porm, no utilizar a letra X para o dgito de verificao 10, optando pelo uso do zero para represent-lo. curioso notar, no caso portugus, onde um dgito de verificao 0 pode significar o nmero zero ou o nmero dez, que a concepo do sistema de deteco de erros singulares e de transposio est comprometida para os portadores de documentos de identificao com dgito de verificao igual a 0 ou 10. Ficaria a questo: para que o dgito verificador utilize uma s posio, por que no usar a divisibilidade por 10 (cujos restos possveis so s 0, 1, ... , 9), em vez de 11? O argumento na prova da proposio mostra que foi essencial que 11 fosse primo e maior que 10. bom notar ainda que o sistema brasileiro tambm no uniforme. Recentemente descobri que o dgito de verificao do RG, emitido no Rio Grande do Sul, de um amigo gacho, no segue o mesmo algoritmo vlido para So Paulo e muitos outros Estados.
Baseado no artigo Aritmtica modular e sistemas de identificao Jos Luiz Pastore Mello, RPM 48

15

O tamanho da Terra

O raio da Terra aproximadamente 6 400 km..., mas como que se mede o raio da Terra? Um grande sbio da Antiguidade, Eratstenes, calculou o raio da Terra h mais de 2 200 anos! Mais do que isso, os sbios daquela poca calcularam tambm as distncias da Terra Lua e da Terra ao Sol, e os tamanhos desses astros; e para isso utilizaram noes bsicas de semelhana e proporcionalidade. Eratstenes viveu no terceiro sculo a.C., na cidade de Alexandria, que fica no extremo oeste do delta do rio Nilo. Mais ao sul, onde hoje se localiza a grande represa de Assu, ficava a cidade de Siena, como ilustra o mapa. Naquela poca deveria haver um trfego regular de caravanas entre as duas cidades; e, talvez por causa desse trfego, sabiase que a distncia entre Alexandria e Siena era de aproximadamente 5000 estdios, ou seja, 800 km (tomando o estdio como igual a 160 metros). Decerto os viajantes experientes j haviam feito uma boa estimativa dessa distncia. Quem viaja com freqncia por anos a fio sabe calcular as distncias percorridas, muito provavelmente pelo nmero de dias gastos na viagem e
16

pelo que se consegue percorrer numa jornada. E, uma vez conhecida a distncia ao longo das estradas, seria possvel fazer uma estimativa da distncia em linha reta. Outra coisa que se sabia que as duas cidades estavam mais ou menos no mesmo meridiano, ou seja, tinham a mesma longitude. Isso intrigante, pois, enquanto seja relativamente fcil fazer uma estimativa da latitude de um lugar, a comparao das longitudes de dois lugares diferentes um problema bem mais complicado. Decerto eles achavam que as duas cidades estavam no mesmo meridiano porque para ir de Alexandria a Siena viajavase diretamente na direo sul. O que fez Eratstenes Alm desses dois fatos a distncia de 800 km entre as duas cidades e elas estarem no mesmo meridiano1 , dois outros fatos foram cruciais no raciocnio de Eratstenes: devido grande distncia que o Sol se encontra da Terra, os raios solares que chegam ao nosso planeta so praticamente paralelos; e quando os raios solares caam verticalmente ao meio-dia em Siena (o que era comprovado vendo que as cisternas ficavam totalmente iluminadas ao meio-dia e o disco solar podia ser visto refletido no fundo dessas cisternas),2 em Alexandria eles formavam, com a vertical do lugar, um ngulo igual a 1/50 da circunferncia completa. Com a medida em graus, isso equivale a dizer que esse ngulo era de 7,2.
Plo Norte A
7,2

B raios solares

S
7,2

Isso s verdade aproximadamente, tanto no que se refere distncia entre as duas cidades, quanto igualdade das longitudes. Veja Alexandria e Assu num bom mapa do Egito: Assu, a antiga Siena, fica s margens do lago Nasser, pouco mais de 3o a leste de Alexandria.
17

Veja:

7, 2 72 2 1 = = = . 360 36 100 100 50


Nesse ponto entra o raciocnio de Eratstenes: se a 1/50 de ngulo correspondem 800 km de arco, ao ngulo de 360 corresponder 50 x 800 = 40 000 km. Que Matemtica foi usada? Vamos rever o raciocnio de Eratstenes para identificar os fatos matemticos usados. Ele entendeu que o ngulo de 7,2 em Alexandria (A na figura anterior) igual ao ngulo central em O, o que pressupe que os raios solares que chegam Terra so paralelos, devido grande distncia do Sol3. Portanto, a igualdade dos ngulos em O e A devida ao fato de eles serem ngulos correspondentes em duas paralelas (AB e OS) cortadas pela transversal OA. O outro fato matemtico utilizado o da proporcionalidade entre arcos e ngulos: os ngulos centrais so proporcionais aos arcos que subentendem; assim, o ngulo de 7,2 est para o arco AS, assim como 360 est para a circunferncia completa. Ser que foi isso mesmo? Sim, ser que Eratstenes mediu mesmo o ngulo de incidncia dos raios solares? Para isso ele teria de se valer de algum aparelho, e teria de realizar uma operao meio sofisticada, difcil de ser feita com preciso. Parece que ele procedeu de maneira muito mais simples. Em Alexandria certamente havia um relgio solar, com uma coluna construda bem na vertical, cujas sombras projetadas serviam para marcar a hora do dia. Ele decerto esperou o dia do ano em que se sabia que os raios solares incidiam verticalmente em Siena ao meio-dia; e, nesse instante, mediu o comprimento da sombra projetada pela coluna do relgio solar em Alexandria.

Isso tambm s verdade aproximadamente; hoje sabemos que a antiga Siena ficava uns 60 km ao norte do Trpico de Cncer, que o paralelo de maior afastamento norte do Sol em relao ao equador.

No tempo de Eratstenes j era sabido que o Sol se encontrava a uma imensa distncia da Terra.

18

C
7,2

C
7,2

De posse do comprimento dessa sombra (AB na figura) e da altura BC da coluna, ele teria desenhado um tringulo retngulo ABC (numa folha de papiro, com certeza), com lados AB e BC proporcionais aos lados AB e BC, respectivamente, do tringulo ABC, que tambm retngulo em B (veja as figuras). Seria agora relativamente fcil medir o ngulo de incidncia, ou seja, o ngulo ACB do tringulo ABC da figura. Eratstenes teria verificado que esse ngulo era de 1/50 da circunferncia completa, ou seja, 7,2. A igualdade do ngulo de incidncia em A com o ngulo ACB decorre de esses ngulos serem alternos internos; e a igualdade dos ngulos ACB e ACB devida semelhana dos tringulos ACB e ACB. O raio da Terra Da circunferncia terrestre podemos passar ao raio da Terra sem necessidade de novas medies. No caso da Terra, como C = 400 000 km e lembrando que C = 2r, calcula-se r = C 2 6370 km, usando para a aproximao 3,14. Eratstenes, Ptolomeu e Cristvo Colombo J dissemos que Eratstenes viveu no sculo terceiro a.C., provavelmente entre 276 e 196 a.C., dizem os historiadores mais abalizados. Portanto, era pouco mais jovem que Arquimedes (287-212 a.C.). Ele no foi o primeiro a se preocupar com a medida do tamanho da Terra. Aristteles (384-322) e Arquimedes fazem referncias a outras estimativas e citam valores do tamanho da Terra. Mas eles no explicam de onde provm suas informaes, por isso mesmo esses eventuais clculos anteriores a Eratstenes no so levados em conta.
19

O clculo do tamanho da Terra aparece num livro de Ptolomeu sobre Geografia, livro esse que foi muito usado no tempo das grandes navegaes. Por razes no bem esclarecidas at hoje, ou Ptolomeu valeu-se de um clculo do raio terrestre diferente do que fez Eratstenes, ou registrou um estdio de outro comprimento que o do tempo de Eratstenes4. Seja como for, em sua Geografia, Ptolomeu utiliza um valor do raio da Terra que est abaixo do valor fornecido por Eratstenes. E apresenta um mapa do mundo ento conhecido, o qual contm mais dois erros importantes: a largura leste-oeste do mar Mediterrneo est exageradamente alta, bem como a largura leste-oeste da sia. Em conseqncia desses trs erros, a distncia do Oeste europeu (Espanha, Portugal) ao Leste asitico (Japo, Coria), para quem navegasse pelo oceano Atlntico em direo oeste, seria bem mais curta do que realmente . Cristvo Colombo valeu-se disso para convencer os reis de Espanha de que sua viagem s ndias seria vivel5. Sua sorte foi estar errado em pensar que no havia terra em seu caminho, pois, fosse isso verdade, ele teria perecido.
Baseado no artigo Se eu fosse professor de Matemtica Geraldo vila, RPM 54

4 5

Cabe notar tambm que no h acordo sobre o valor exato do estdio em metros. interessante notar que razes de ordem tcnica ao menos em parte levaram Portugal a no aprovar a proposta de Colombo. Com efeito, os especialistas encarregados de julgar essa proposta constataram corretamente que a distncia a ser percorrida na viagem seria muito mais longa do que Colombo previa, sendo impossvel levar vveres e gua em quantidades suficientes para toda a viagem.

20

Problema das idades

Tenho o triplo da idade que tu tinhas quando eu tinha a idade que tu tens. Quando tu tiveres a idade que eu tenho, teremos juntos 56 anos. Qual a minha idade? Esse problema, com enunciado em estilo de uma charada, est hoje meio fora de moda, mas foi clebre numa poca em que havia uma preocupao de resolver esse e outros tipos de problemas por Aritmtica e no por lgebra. Vamos abordar o problema geometricamente. Se representarmos graficamente, num sistema de coordenadas cartesianas, a evoluo da idade de um indivduo atravs do tempo, obteremos sempre uma reta paralela bissetriz do primeiro quadrante. Na realidade, obteremos a prpria bissetriz se tomarmos o ano zero como sendo o ano de seu nascimento, pois no ano 1 ele ter 1 ano, e assim sucessivamente (isso um fato do qual a experincia j mostrou que podemos convencer mesmo uma pessoa que jamais estudou Geometria Analtica). J a idade de uma pessoa d anos mais velha ter como grfico uma reta paralela, j que a diferena entre as idades dos dois permanecer constante e igual a d.
t
21

Voltemos ento ao nosso problema. H dois indivduos em causa, um que fala, chamamo-lo de E, e um que escuta, T. Evidentemente E mais velho que T (... quando eu tinha a idade que tu tens...), digamos, d anos, de modo que seus grficos de idades se assemelham aos da figura da pgina anterior. H trs pocas mencionadas no problema, que chamaremos P (passada), A (atual) e F (futura). A maneira como se relacionam A e P (... quando eu tinha a idade que tu tens...) e a maneira como se relacionam A e F (... quando tu tiveres a idade que eu tenho...) mostram que elas se situam no grfico como nos casos da figura ao lado.
i

E T

X Y Z 0

P A F

A inclinao de 45o das retas desenhadas acarreta que todos os segmentos verticais compreendidos entre elas tm comprimento d. O dado de que a idade que E tem na poca A (isto , OX) o triplo da idade que T tinha na poca P (isto , OZ), juntamente com o fato de XY = YZ = d, obriga a que OZ seja tambm d (estou evitando escrever a equao 2d + OZ = 3OZ, j que isso pode ser visto na figura). Mas, ento, a reta grfica da idade de E tem que passar por Z e a figura correta a que est ao lado.
i

E T

X
d

P A F

Agora ento claro que, na poca F, a idade de T 3d enquanto a de E 3d + d. Logo os dois juntos tm 7d, que deve ser 56. Logo, d tem que ser 8 e a idade que eu tenho 3 x 8 = 24, que a resposta.

Baseado no artigo Uma soluo geomtrica para o problema das idades Jos Paulo Q. Carneiro, RPM 16

22

A ilha dos sapatos gratuitos

Cena no 1 O problema Um dia, estava eu na faculdade tranqilamente pensando na vida quando chegou um colega e me fez uma proposta inusitada: Voc quer comprar de graa (?!) um sapato? claro que eu topei de cara comprar de graa (?!) um sapato, embora desconfiasse que houvesse algum rolo. As condies eram: 1. Comprar um selinho desse meu amigo. Preo R$ 3,00; 2. Juntar mais R$ 27,00 e o selinho e levar a uma determinada loja. Eu receberia um par de sapatos com valor de mercado de R$ 30,00 e mais dez selinhos no valor de R$ 3,00; 3. Vender os dez selinhos que eu seria restitudo dos R$ 3,00 iniciais de compra do selinho do meu amigo e dos R$ 27,00 que anexei para retirar o sapato da loja. Dei R$ 3,00 ao meu colega pelo selo, fui loja, retirei um par de sapatos por R$ 27,00 e ganhei os dez selinhos que me iriam restituir tudo o que investira. Vendi os dez selinhos com alguma facilidade. Fiz ento um balano: eu tinha at ento gasto R$ 30,00, recebido R$ 30,00 e mais um par de sapatos. Um par de sapatos de graa, portanto. Como isso seria possvel? No estaria essa promoo violando a Lei de Lavoisier ou a Segunda Lei da Termodinmica? Fiquei estarrecido com o problema. Como interpret-lo?
23

Cena no 2 As explicaes convencionais Aturdido com o problema que aparentemente violava leis naturais nunca dantes questionadas, sa a conversar com meus colegas de faculdade. O primeiro a tentar responder foi Altarimando. Ele se entusiasmou. No se preocupe se essa promoo fere ou no as leis da natureza. O importante que funciona. Assim como voc conseguiu comprar sapatos de graa, vamos expandir o negcio para comprar arroz de graa, roupa de graa, etc. Talvez esse seja o perdido caminho para a humanidade alcanar o Nirvana, o to desejado Shangril. No se esquea de que as Leis de Mercado so superiores Lei de Lavoisier. Desconfiei que ele estava mais para poeta transcendental que crtico de Matemtica e Fsica e fui procurar o Souzinha, um crtico de tudo. Logo deu seu parecer, claro e taxativo, incisivo e demolidor, caracterstico de todo jovem de menos de quarenta anos: Estamos diante da chamada Bola-de-neve, Conto da venda sucessiva ou ainda da Corrente da felicidade. um estratagema que favorece barbaramente os compradores iniciais e altamente desvantajoso para os finais. O universo possvel de compradores um nmero finito e os compradores dos selinhos so: 1 na primeira etapa, 10 na segunda, 100 na terceira, etc. Ou seja, os envolvidos na corrente so em nmero de 100 + 101 + 102 + 103+ ... Quando o somatrio excede o nmero de possveis compradores, a corrente pra e os ltimos no tero para quem vender os selos, sendo prejudicados. Logo, essa artimanha to simplesmente uma falcia. Continuam vlidos, portanto, a Lei de Lavoisier e o Segundo Princpio da Termodinmica. Fiquei feliz, confesso, por essa explicao do Souzinha. As pessoas como ns, que estudam Matemtica, com a mente criada e disciplinada por critrios lgico-formais cartesianos tm verdadeiro horror a situaes que fujam desse modo e, o que pior, funcionem. Se isso pudesse ocorrer, ficaramos inseguros, e toda uma vida ficaria questionada. Cena no 3 A explicao diferente Quando eu j estava disposto a encerrar o assunto, encontrei um velho amigo, Ado, estudante de Economia na Getlio Vargas.
24

Apesar de jovem, Ado crtico ponderado e profundo em seus conhecimentos. S como curiosidade, expus a ele o problema e as duas respostas que eu tinha ouvido at ento. Ado, filosoficamente, comeou a raciocinar socraticamente. Quanto mesmo que a loja recebe por par de sapatos vendido? Ora, Ado, respondi, o enunciado claro. Ela recebe R$ 30,00 por par de sapatos. Acho que a temos uma pista, no esse o valor, ponderou Ado. E continuou: Admitamos uma ilha com 1 111 pessoas potencialmente clientes dos sapatos e mais uma pessoa, que o dono da loja, totalizando 1 112 pessoas. O dono da loja prope o negcio a um primeiro cliente. Compre um selo por R$ 3,00, adicione R$ 27,00 e deflagre o processo. Esse primeiro cliente vende dez selos. Dez compradores vendem depois para 100 outros compradores. J so 111 compradores. Os cem compradores vendem agora para 1 000 compradores. Esses ltimos 1000 compradores, que j gastaram, cada um, R$ 3,00 pelo selo, no tm mais para quem vender. Uma de suas opes perder esse selo. Outra (mais razovel) acrescentar R$ 27,00 e ir buscar o seu par de sapatos, que, como sabemos, vale no mercado R$ 30,00. Logo, esses ltimos compradores no sero prejudicados financeiramente (s no tero os seus sonhos de sapatos grtis). Agora faamos um raciocnio. Quanto recebeu a loja de sapatos e quantos pares de sapatos foram entregues? Curiosamente voc ver que a loja no recebe R$ 30,00 por par de sapatos vendido. A loja recebeu em dinheiro: do 1o comprador: de 10 compradores: de 100 compradores: de 1000 compradores: Total 3,00 + 27,00 10 x 27,00 100 x 27,00 1000 x 27,00 R$ 30 000,00 = 30,00 = 270,00 = 2 700,00 = 27 000,00

Total de pares de sapatos vendidos = 1111 Receita mdia da loja por par de sapatos: R$ 30 000,00/1111 R$ 27,03
25

Concluso A loja vende cada par de sapatos a R$ 30,00 e recebe na prtica R$ 27,00 e no R$ 30,00, como supostamente se poderia pensar. V-se, portanto, que cada pessoa para ganhar um par de sapatos precisa entregar o sinal (entrada) e ter o trabalho de vender dez outros sapatos. O caso em estudo um processo que traz embutido um trabalho de venda como custo. Custo esse que pago pela loja (30,00 27,03) = 2,97 por par de sapatos. uma comisso de venda. Tudo claro, Botelho? Fiquei a pensar. Como as coisas ainda estavam algo confusas dentro de mim, pedi apoio Revista do Professor de Matemtica. A resposta da RPM 1. Se a histria se passasse no instante em que nosso amigo Botelho acabou de vender seus dez selinhos, o que estaria acontecendo que dez pessoas (os compradores dos selinhos) teriam se cotizado para comprar um par de sapatos para ele. 2. Na histria, nada obriga que cada comprador se limite a adquirir um par de sapatos apenas. Para citar um caso extremo, podemos supor que o primeiro comprador, em vez de vender os 10 selinhos que recebeu da loja, fica com eles e com isso compra mais dez pares de sapatos a R$ 27,00 cada, recebe 100 selinhos, etc., at acabar com o estoque da loja. Depois, revende todos os sapatos ao preo oficial de R$ 30,00. Em vez de um par de sapatos de graa, ganha muito mais. 3. Do ponto de vista da loja, o que ela fez corresponde simplesmente a vender cada par de sapatos a R$ 27,00, exceto o primeiro, vendido por R$ 30,00. Os selinhos so apenas um truque de marketing. A loja vende por R$ 27,00, mas, como o preo usual R$ 30,00, a diferena dividida entre alguns felizardos, ou espertos. O exemplo do economista Ado, em que cada habitante da ilha compra apenas um par de sapatos, o extremo oposto do caso 2 acima. Na prtica ocorrem, em geral, situaes intermedirias em que algumas pessoas formam estoque para revenda (podendo em seguida organizar cartis para manipular os preos, mas isso j seria outra histria).
Baseado no artigo Na ilha dos sapatos gratuitos Manoel Henrique C. Botelho, RPM 7

26

Fraes egpcias

Quando se menciona Fibonacci, ou seja, Leonardo Fibonacci (1170, 1240?), tambm conhecido como Leonardo Pisano ou Leonardo de Pisa, pensa-se logo no clebre problema dos coelhos, apresentado e resolvido no seu Liber Abaci, conduzindo clebre seqncia 1, 1, 2, 3, 5, 13, ..., que at hoje leva seu nome. Mas o livro contm muito mais: entre os problemas nele tratados, a maioria sem grande interesse para ns, leitores de hoje, pois tratam de Aritmtica usando os algarismos indoarbicos ou de Matemtica Comercial, encontramos verdadeiras jias matemticas, como um relacionado com a maneira egpcia de lidar com fraes. Como sabemos, os egpcios s trabalhavam com fraes unitrias, isto , da forma 1/n, sendo n um nmero natural [ exceo de 2/3 e, s vezes, das fraes da forma n/(n + 1)]. Obviamente, em seus problemas matemticos apareciam fraes da forma m/n, que deviam ento ser escritas usando-se somente fraes unitrias distintas. Ou seja, era necessrio escrever
m 1 1 1 = + +L + n n1 n2 nk , com n1, n2, ..., nk naturais

distintos. No discutiremos aqui as interpretaes apresentadas pelos eruditos para essa insistncia egpcia em trabalhar com fraes unitrias. Esse hbito, embora pesado e inconveniente, sobreviveu at a Idade Mdia. Em verdade, os egpcios, por meio de tabelas apropriadas e mtodos
27

engenhosos, conseguiam lidar muito bem com as fraes unitrias. O leitor mais curioso poder consultar o livro Mathematics in the Time of the Pharaohs de autoria de R. J. Gillings, Dover, 1982, ou, para uma leitura leve, a RPM 15, p. 21. No bvio que qualquer nmero racional m/n, com m < n, possa ser escrito como soma de fraes unitrias. Uma prova da acuidade matemtica de Fibonacci ter percebido a necessidade de mostrar isso. Ele no apresenta uma demonstrao formal, como o faramos hoje, mas d um mtodo inteiramente geral que resolve o problema. A regra ... que voc divide o nmero maior pelo menor; e quando a diviso no exata, verifique entre que dois naturais a diviso est. Tome a maior parte, subtraia-a, e conserve o resto ... Em linguagem de hoje, a regra seria: Subtraia da frao dada a maior frao unitria que no maior do que ela. Repita o processo at obter 0. Por exemplo, escrevamos a frao 4/13 como soma de fraes unitrias distintas: 3 < 13/4 < 4 1/3 > 4/13 > 1/4 Portanto, 4/13 1/4 = 3/52. Mas, ento, 17< 52/3 < 18 1/17 > 3/52 > 1/18. Logo, 3/52 1/18 = 2/936 = 1/468. Aqui, a diviso de 936 por 2 exata, e o processo termina. Assim, 4/13 = 1/4 + 1/18 + 1/468. No difcil demonstrar que o processo descrito por Fibonacci sempre funciona. Para mostrar que o mtodo funciona, demonstraremos que os numeradores das diferenas sucessivas (mesmo antes de simplificar) decrescem estritamente (no exemplo acima, as diferenas so 3/52 e 2/936). Ento, como toda sucesso estritamente decrescente de nmeros naturais no negativos finita (veja O princpio da descida infinita de Fermat, RPM 32), o processo obrigatoriamente tem fim. a com a < b. Com efeito, consideremos a frao b
28

a 1 Suponha que b = qa + r, 0 r < a. Se r = 0, ento, b = q e a


demonstrao est terminada. Podemos, portanto, supor que r 0. Ento, Assim,
b r =q+ a a implicando q < b < q + 1, ou a

1 a 1 > > . q b q +1

a 1 r + a . = b q + 1 b(q + 1)

Mas, como a r < a, os numeradores das diferenas sucessivas so estritamente decrescentes quando r 0, o que queramos demonstrar.
Baseado no artigo Um problema de Fibonacci Joo Pitombeira de Carvalho, RPM 17

29

As dzimas peridicas e a calculadora

Em uma prova de concurso, destinado principalmente a professores de Matemtica, figurava a seguinte questo: Os nmeros racionais a e b so representados, no sistema decimal, pelas dzimas peridicas

a = 3, 0181818... = 3, 018 e b = 1,148148... = 1,148


Encontre, justificando, uma representao decimal de a b. Como a e b so racionais, tambm o a b; e, portanto, sua representao decimal peridica. Na prova, era permitido o uso de calculadora. Mas por meio da calculadora jamais se descobrir o perodo, pelo menos com a certeza exigida pelo justifique. Alm disso, a calculadora no conseguir nem mesmo dar uma idia do perodo, se ele for muito longo. De fato, o perodo pode ter um comprimento maior do que o nmero de dgitos que a calculadora exibe no visor. Um primeiro expediente que poderia ocorrer seria fazer a subtrao por meio do esquema usado habitualmente para decimais finitos. Isso funcionaria bem em casos mais simples. Por exemplo: 0, 444... 0, 333... 0,111... o que estaria correto, pois
30

4 3 1 = . 9 9 9

Mas, no caso em questo, o desencontro entre os perodos das duas dzimas apresentadas dificulta o emprego dessa estratgia (a qual, alis, precisaria ser discutida em termos conceituais). Vejamos: Como a subtrao usual feita da direita para a esquerda, no se sabe bem por onde comear, antes de descobrir o perodo. Por conseguinte, o caminho natural calcular as geratrizes de a e b, subtrair as fraes correspondentes, e ento encontrar uma representao decimal para essa frao. Utilizando esse procedimento, encontra-se:

a b = 3+

18 148 1292 2777 1 + = . =1+ 990 999 1485 1485

Neste ponto, o mtodo mais usado por todo mundo dividir 2777 por 1485 (ou 1292 por 1485, ganhando uma etapa) pelo algoritmo tradicional, e aguardar o primeiro resto que repete. Deste modo, obtm-se:
1 2 9 2 0 1 0 4 0 0 5 0 0 0 5 4 5 0 9 9 5 0 1 0 4 0 1485 0, 8700336

Como se repetiu o resto 1040, a partir da, os algarismos 7, 0, 0, 3, 3, 6 iro se repetir. Logo, a b = 1, 8700336 . Vamos agora fazer alguns comentrios: 1. Algumas pessoas envolvidas no processo de aprendizagem da Matemtica (alunos, professores, pais, etc.) expressam s vezes a crena de que, com o advento da calculadora, nunca mais haver ocasio de usar o algoritmo tradicional da diviso. Alguns at usam isso como um argumento para proibir o uso da calculadora em certas fases iniciais da aprendizagem: necessrio primeiro que o aluno aprenda o algoritmo tradicional, e s depois lhe ser permitido usar a calculadora; seno, ele no ter motivao para aprender tal algoritmo.

31

Na realidade, o exemplo aqui tratado mostra que ns, professores, temos que exercer nossa criatividade para criar problemas desafiadores, que coloquem em xeque at mesmo a calculadora, deixando claras as suas limitaes, em vez de proibir o uso da calculadora, que uma atitude antiptica, repressora, e totalmente contrria ao que um aluno espera de um professor de Matemtica. De fato, para um leigo ou iniciante em Matemtica, nada mais matemtico do que uma calculadora, e ele espera que um professor v inici-lo ou ajud-lo com essa ferramenta, e no proibi-lo de us-la. Note-se tambm que, mesmo usando o algoritmo tradicional da diviso, como fizemos, a calculadora permanece til para efetuar as multiplicaes e subtraes envolvidas no processo, minorando as possibilidades de erro e poupando trabalhos repetitivos e inteis. 2. O trabalho de diviso ficaria simplificado, se tivssemos observado que o divisor 1485 tem o fator comum 5 com a base do sistema decimal (um detalhe nem sempre lembrado). Desse modo:

1292 1292 1 2584 1 208 = = = 8 + = 1485 5 297 10 297 10 297 1 208 = 0, 8 + 0, 070336 = 0, 8700336, pois 0, 8 + 10 297
2 0 8 0 1 0 0 0 1 0 9 0 1 9 9 0 2 0 8 297 0, 70336

Os nmeros envolvidos no algoritmo da diviso ficam menores. 3. Existiria um outro mtodo para encontrar uma representao decimal de

208 1292 , mas j vimos que basta o primeiro), que no fosse (ou de 297 1485 o algoritmo tradicional da diviso? A resposta sim.
Basta tomar as sucessivas potncias de 10, a saber: 10, 100, etc., at que encontremos uma que deixe resto l, quando dividida por 297.
32

No difcil fazer isso, experimentando com a calculadora: 103 = 3 x 297 + 109; 104 = 33 x 297 + 199; 106 = 3367 x 297 + 1. A partir da, obtm-se: 105 = 336 x 297 + 208;

1 1 = 3367 6 , e portanto: 297 10 1

208 1 1 106 = 208 3367 6 = 700336 = 297 10 1 1 1 106 700336 1 1 1 + 6 + 12 + K = 0, 700336700336700336K = 0, 700336, 6 10 10 10
onde a ltima passagem vem da propriedade das progresses geomtricas infinitas: 1 + q + q 2 + K =

1 , quando 1 < q < 1. 1 q

Observe que o perodo da dzima tem comprimento 6, que justamente o expoente da menor potncia de 10 que deixa resto 1, quando dividida por 297. 4. Pode-se ter certeza de que, ao testar as potncias de 10, vamos acabar encontrando sempre uma que deixe resto 1, quando dividida por 297 ? A resposta positiva, sempre que o denominador (no caso, o 297) for primo com 10 ( por isso que devemos antes deixar de fora os fatores 2 e 5), e pode ser encontrada nos livros de Teoria dos Nmeros.

Baseado no artigo As dzimas peridicas e a calculadora Jos Paulo Q. Carneiro, RPM 52

33

Mania de Pitgoras

Elisha Scott Loomis, professor de Matemtica em Cleveland, Ohio (Estados Unidos), era realmente um apaixonado pelo Teorema de Pitgoras. Durante 20 anos, de 1907 a 1927, colecionou demonstraes desse teorema, agrupou-as e as organizou num livro, ao qual chamou The Pythagorean Proposition (A Proposio de Pitgoras). A primeira edio, em 1927, continha 230 demonstraes. Na segunda edio, publicada em 1940, esse nmero foi aumentado para 370 demonstraes. Depois do falecimento do autor, o livro foi reimpresso, em 1968 e 1972, pelo National Council of Teachers of Mathematics daquele pas. O Professor Loomis classifica as demonstraes do teorema de Pitgoras em basicamente dois tipos: provas algbricas (baseadas nas relaes mtricas nos tringulos retngulos) e provas geomtricas (baseadas em comparaes de reas). Ele se d ao trabalho de observar que no possvel provar o teorema de Pitgoras com argumentos trigonomtricos porque a igualdade fundamental da Trigonometria, cos2 x + sen2 x = 1, j um caso particular daquele teorema. Como sabemos, o enunciado do teorema de Pitgoras o seguinte: A rea do quadrado cujo lado a hipotenusa de um tringulo retngulo igual soma das reas dos quadrados que tm como lados cada um dos catetos. Se a, b so as medidas dos catetos e c a medida da hipotenusa, o enunciado equivale a afirmar que a2 + b2 = c2.
34

Documentos histricos mostram que os egpcios e os babilnios muito antes dos gregos conheciam casos particulares desse teorema, expressos em relaes como
3 1 32 + 42 = 52 e 12 + ( ) 2 = (1 ) 2 . 4 4

O fato de que o tringulo de lados 3, 4 e 5 retngulo era (e ainda ) til aos agrimensores. H tambm um manuscrito chins, datado de mais de mil anos antes de Cristo, onde se encontra a seguinte afirmao: Tome o quadrado do primeiro lado e o quadrado do segundo e os some; a raiz quadrada dessa soma a hipotenusa. Outros documentos antigos mostram que na ndia, bem antes da era Crist, sabia-se que os tringulos de lados 3, 4, 5, ou 5, 12, 13, ou 12, 35, 37 so retngulos. O que parece certo, todavia, que nenhum desses povos sabia demonstrar o teorema. Tudo indica que Pitgoras foi o primeiro a prov-lo. (Ou algum da sua Escola o fez, o que d no mesmo, pois o conhecimento cientfico naquele grupo era propriedade comum.) A mais bela prova Qual foi a demonstrao dada por Pitgoras? No se sabe ao certo, pois ele no deixou trabalhos escritos. A maioria dos historiadores acredita que foi uma demonstrao do tipo geomtrico, isto , baseada na comparao de reas. No foi a que se encontra nos Elementos de Euclides, e que ainda hoje muito encontrada nos livros de Geometria, pois tal demonstrao parece ter sido concebida pelo prprio Euclides. A demonstrao de Pitgoras pode muito bem ter sido a que decorre das figuras abaixo:
a b a c c b a b b c a

Do quadrado que tem a + b como lado, retiremos 4 tringulos iguais ao dado. Se fizermos isso como na figura esquerda, obteremos um quadrado de lado c. Mas se a mesma operao for feita como na figura direita, restaro dois quadrados, de lados a e b respectivamente. Logo, a rea do
35

quadrado de lado c a soma das reas dos quadrados cujos lados medem a e b. Essa , provavelmente, a mais bela demonstrao do teorema de Pitgoras. Entretanto, no livro de Loomis ela aparece sem maior destaque, como variante de uma das provas dadas, no sendo sequer contada entre as 370 numeradas. Apresentamos a seguir algumas demonstraes do teorema de Pitgoras que tm algum interesse especial, por um motivo ou por outro. As 4 primeiras constam da lista do Professor Loomis. A prova mais curta tambm a mais conhecida. Baseiase na seguinte conseqncia da semelhana de tringulos retngulos: Num tringulo retngulo, cada cateto a mdia geomtrica entre a hipotenusa e sua projeo sobre ela. Assim, se m e n so respectivamente as projees dos catetos a e b sobre a hipotenusa c, temos a2 = mc, b2 = nc, enquanto m + n = c. Somando, vem a2 + b2 = c2. A demonstrao do presidente James Abram Garfield, presidente dos Estados Unidos durante apenas 4 meses (pois foi assassinado em 1881), era tambm general e gostava de Matemtica. Ele deu uma prova do teorema de Pitgoras baseada na seguinte figura. A rea do trapzio com bases a, b e altura a + b igual semi-soma das bases vezes a altura. Por outro lado, a mesma rea tambm igual soma das reas de 3 tringulos:
a+b a2 b2 c2 + ab + = ab + , ( a + b) = 2 2 2 2 2 2 2 implicando a + b = c .

a m c n

b a

c b

36

A demonstrao de Leonardo da Vinci O grande gnio tambm concebeu uma demonstrao do teorema de Pitgoras, que se baseia na figura ao lado. Os quadrilteros ABCD, DEFA, GFHI e GEJI so congruentes. Logo, os hexgonos ABCDEF e GEJIHF tm a mesma rea. Da resulta que a rea do quadrado FEJH a soma das reas dos quadrados ABGF e CDEG. A demonstrao de Papus
B A F G

C D E

H I

Na realidade, no se trata apenas de uma nova demonstrao, mas de uma generalizao bastante interessante do teorema de Pitgoras. Em vez de um tringulo retngulo, toma-se um tringulo arbitrrio ABC; em vez de quadrados sobre os lados, tomam-se paralelogramos, sendo dois deles quaisquer, exigindo-se que o terceiro cumpra a condio de CD ser paralelo a HA, e com o mesmo comprimento. G H I L K F J A
B E M N C D

O teorema de Papus afirma que a rea do paralelogramo BCDE a soma das reas de ABFG e AIJC. A demonstrao se baseia na simples observao de que dois paralelogramos com bases e alturas de mesmo comprimento tm a mesma rea. Assim, por um lado, AHKB tem a mesma rea que ABFG e, por outro lado, a mesma rea que BMNE. Segue-se que as reas de BMNE e ABFG so iguais. Analogamente, so iguais as reas de CDNM e CAIJ. Portanto, a rea de BCDE a soma das reas de ABFG e CAIJ. O teorema de Pitgoras caso particular do de Papus. Basta tomar o tringulo ABC retngulo e trs quadrados em lugar dos trs paralelogramos.
37

O argumento de Polya No meu entender, entretanto, a demonstrao mais inteligente do teorema de Pitgoras no est includa entre as 370 colecionadas pelo Professor Loomis. Ela se acha no livro Induction and Analogy in Mathematics, de autoria do matemtico hngaro George Polya. O raciocnio de Polya se baseia na conhecida proposio, segundo a qual as reas de duas figuras semelhantes esto entre si como o quadrado da razo de semelhana. Lembremos que duas figuras F e F dizem-se semelhantes quando a cada ponto A da figura F corresponde um ponto A em F, chamado o seu homlogo, de tal maneira que se, A, B so pontos quaisquer de F e A, B so seus homlogos em F ento a razo AB/AB uma constante k, chamada a razo de semelhana de F para F. Por exemplo, dois tringulos so semelhantes se, e somente se, os ngulos de um deles so congruentes aos ngulos do outro. Por outro lado, dois quadrados quaisquer, um de lado l e outro de lado l, so semelhantes e a razo de semelhana do primeiro para o segundo k = l/l. Em vez do teorema de Pitgoras, Polya procura provar a seguinte proposio mais geral (que, diga-se de passagem, j se acha nos Elementos de Euclides):
F F

Se F, F e F so figuras semelhantes, construdas respectivamente sobre a hipotenusa c e sobre os catetos a, b de um tringulo retngulo, ento a rea de F igual soma das reas de F e F. O enunciado acima implica que a razo de semelhana de F para F b/a, de F para F c/a e de F para F c/b. Por simplicidade, escrevamos F em vez de rea de F, G em vez de rea de G, etc.
38

Se G, G, G so outras figuras semelhantes construdas sobre a hipotenusa e os catetos, respectivamente, em virtude da proposio acima enunciada, teremos:

G F G b 2 F , logo = . = 2 = G" a F" G" F "


De modo anlogo teremos

G G = . F F

Portanto, G/F = G/F = G/F = , digamos. Escrevendo de outro modo: G = F, G = F e G = F. Que significam essas 3 ltimas igualdades? Elas querem dizer que, se conseguirmos achar 3 figuras semelhante especiais F, F e F, construdas sobre a hipotenusa e os catetos do nosso tringulo, de tal maneira que se tenha F = F + F, ento teremos tambm G = G + G sejam quais forem as figuras semelhantes G, G e G construdas do mesmo modo. Com efeito, teremos G = F, G = F e G = F, logo G + G = F + F = (F + F) = F = G. Agora s procurar as figuras especiais. Mas elas esto facilmente ao nosso alcance. Dado o tringulo retngulo ABC, tracemos a altura CD, baixada do vrtice do ngulo reto C sobre a hipotenusa AB.

A figura F ser o prprio tringulo ABC. Para F escolheremos ADC e faremos F = BCD. Evidentemente, F, F e F so figuras semelhantes. Mais evidentemente ainda, temos F = F + F.
Baseado no artigo Mania de Pitgoras Euclides Rosa, RPM 02

39

Usando reas

Neste artigo, procuraremos mostrar que diversas demonstraes em Geometria e Trigonometria tornam-se fceis e elegantes quando usamos o conceito de rea. Como primeiro exemplo, comparemos duas solues de um conhecido problema. Provar que a soma das distncias de um ponto qualquer interior a um tringulo equiltero aos lados constante. 1a soluo Consideremos o tringulo equiltero ABC da figura, um ponto P interior e as perpendiculares PX, PY e PZ aos seus lados. Tracemos por P, BC paralelo a BC, nformando o tringulo equiltero ABC. Tracemos ainda as alturas AE e CF desse tringulo e a perpendicular PQ a CF. A
F Z B B P E

Y C C

X D

Pela congruncia dos tringulos PQC e PYC, conclumos que PY = CQ e, como PQFZ um retngulo, temos que PZ = QF. Logo,
40

PY + PZ = CQ + QF = CF . (Para simplificar a notao, usaremos o mesmo smbolo para representar um segmento ou a sua medida.) Ora, as alturas AE e CF do tringulo equiltero ABC so iguais e, portanto, PY + PZ = AE. (1) Prolongando AE at a base BC do tringulo, obtemos ED = PX. Finalmente, na igualdade (1), somamos PX do lado esquerdo e ED do lado direito para obter PX + PY + PZ = AE + ED = AD, altura de ABC. 2a soluo Consideremos agora o tringulo equiltero ABC com lado a e altura h, como na figura ao lado. Traando os segmentos PA, PB e PC, temos que a soma das reas dos tringulos PBC, PCA e PAB igual rea de ABC. Logo,
A

Z P B X

aPX aPY aPZ ah + + = 2 2 2 2

e o problema est resolvido. Repare que na primeira soluo usamos apenas o conceito de congruncia de tringulos, mas a construo das linhas auxiliares pode ser considerada um pouco artificial. Na segunda soluo, quando o conceito de rea foi utilizado, o resultado apareceu de forma bem mais natural. Vejamos, ento, alguns teoremas que podem ser demonstrados com o auxlio das reas. 1) O teorema da bissetriz A bissetriz de um ngulo de um tringulo divide o lado oposto em segmentos proporcionais aos lados adjacentes. Esse enunciado quer dizer que se, AD for bissetriz do ngulo A do tringulo ABC, ento

DB AB = . DC AC
41

Para demonstrar, preciso lembrar que, se dois tringulos possuem mesma altura, a razo entre suas reas igual razo entre suas bases. Portanto, na figura, a razo entre as reas dos tringulos ADB e ADC igual a BD/DC. Por outro lado, qualquer ponto da bissetriz de um ngulo eqidista de seus lados e, portanto, as perpendiculares DE e DF aos lados AB e AC so iguais. Logo, A

1 AB.DE BD A ( ABD) 2 AB = = = 1 DC A ( ADC ) AC.DF AC 2


B

E F D C

e o teorema est demonstrado. 2) O teorema de Tales Sejam B e C pontos dos lados AB e AC, respectivamente, do tringulo ABC. Se BC for paralelo a BC, ento Demonstrao Se BC paralelo a BC, ento os tringulos BCB e BCC tm mesma rea porque possuem mesma base BC e alturas relativas a essa base tambm iguais. Acrescentando a esses tringulos o tringulo ABC, conclumos que os tringulos ABC e ABC tambm possuem mesma rea. Se dois tringulos possuem mesma altura, ento a razo entre suas reas igual razo entre suas bases, logo,
A
AB AC = . AB AC

B B

C C

AB A (ABC ) A (ABC ) AC = = = AB A (ABC ) A (ABC ) AC

o que prova o teorema. O teorema de Tales e sua recproca so importantssimos em Geometria porque a partir deles podemos obter os teoremas relativos semelhana de tringulos e as propriedades da homotetia. A vantagem da demonstrao
42

que aqui apresentamos est no fato que nela no importa se os segmentos AB e AB so comensurveis ou no. A demonstrao tradicional, que usa o feixe de paralelas, s fica completa com a incmoda passagem ao limite. 3) As frmulas trigonomtricas As funes trigonomtricas aparecem pela primeira vez na escola secundria, definidas para ngulos agudos, como razes entre lados de um tringulo retngulo. Usando figuras particulares, conseguimos calcular os valores das funes trigonomtricas para 30, 45, 60 e 18 e podemos antecipar diversas frmulas que, mais tarde, sero deduzidas em contexto mais geral. Para ilustrar, vamos mostrar a frmula do seno do arco duplo. Se 0 < a < 45, ento sen2a = 2 sen a cos a Para demonstrar, consideremos a figura formada por dois tringulos retngulos congruentes OCA e OCB, em que fizemos OA = OB = 1. Temos, ento, que CA = CB = sen a, OC = cosa e, traando AD perpendicular a OB, AD = sen 2a. Ora, o dobro da rea do tringulo OAB igual a OB . AD e tambm igual a AB . OC.
O
a a

A
1

C D
q

Logo, 1 sen 2a = 2 sen a cos a, ficando demonstrada a frmula. 4) A lei dos senos Os lados de um tringulo so proporcionais aos senos dos ngulos opostos. Para uma demonstrao alternativa da lei dos senos, podemos partir do fato de que a rea, A , de um tringulo igual metade do produto de dois lados pelo seno do ngulo formado por eles, ou seja,
1 1 1 A = bc sen A = ac sen B = ba sen C. 2 2 2

Ora, considerando a primeira igualdade e multiplicando por a ambos os membros, obtemos


1 a abc aA = abc sen A ou = . 2 sen A 2A
43

Como o mesmo pode ser feito para as outras igualdades, conclumos

a b c = = . sen A sen B sen C


Muitos problemas possuem tambm solues bonitas e elegantes usando reas e este artigo termina convidando o leitor a incluir a idia em sua caixa de ferramentas de soluo de problemas.
Baseado no artigo Usando reas Eduardo Wagner, RPM 21

44

Trigonometria e um antigo problema de otimizao

Regiomontanus A cidade de Kningsberg, na Prssia (atual Kalimingrado, na Rssia), conhecida na Matemtica devido ao famoso problema das pontes (ver artigo neste livro), resolvido pelo matemtico suo Leonhard Euler (1707-1783). Outro acontecimento importante que marca a vida da cidade, cujo nome significa Montanha do Rei, o fato de nela ter nascido Johann Mller (1436-1476), um dos maiores matemticos do sculo XV, mais conhecido como Regiomontanus, uma latinizao do nome de sua cidade natal. Regiomontanus realizou diversos estudos nas reas de Astronomia, Geometria e Trigonometria. Em seu livro mais famoso, De Triangulus Omnimodes, escrito em 1464 e impresso apenas em 1533, Regiomontanus apresenta uma viso moderna da Trigonometria com dados tabelados de vrias funes trigonomtricas. curioso notar que, mesmo tendo sido escrito antes do conceito de notao decimal, as tabelas trigonomtricas contidas no livro no apresentam fraes devido utilizao de um crculo de raio 100 000 000 de unidades, o que produzia apenas valores inteiros para as aproximaes utilizadas. A importncia dos conhecimentos em Astronomia de Regiomontanus fez com que ele fosse convidado pelo Papa Sisto IV para trabalhar na confeco de um calendrio mais acurado do que o que vinha sendo usado pela Igreja. Aps a realizao do trabalho, a gratido do Papa foi tal que rapidamente o astrnomo se tornou seu principal conselheiro. Depois de um ano em Roma, Regiomontanus faleceu, tendo sido anunciado, como causa

45

de sua morte, o flagelo de uma peste. Existem especulaes de que ele tenha sido envenenado por alguma pessoa descontente com a alta influncia de um no italiano sobre o Papa e a Igreja romana. Alguns historiadores especulam ainda que, se no tivesse falecido to cedo, talvez tivesse condies de realizar uma moderna compreenso do sistema solar, como a feita por Coprnico, 100 anos depois. Entre os interessantes problemas propostos por Regiomontanus, destacamos um de 1471 como o primeiro problema de extremos encontrado na Histria da Matemtica desde a antiguidade. O problema o seguinte: Suponha uma esttua de altura h sobre um pedestal de altura p. Um homem de altura m (m < p) enxerga do p ao topo da esttua sob um ngulo , que varia de acordo com a distncia d entre o homem e a base do pedestal. Determinar d para que o ngulo de viso seja o maior possvel. Uma soluo engenhosa para o problema Apesar de o problema poder ser resolvido com tcnicas do Clculo, apresentamos uma soluo que, embora engenhosa, dispensa essas tcnicas. Inicialmente marcamos na figura os pontos A, B e C representando respectivamente o A topo da esttua, o p da esttua e os olhos do l observador. Em seguida, traamos a reta r que passa por C e paralela linha do cho. Traamos ento a nica circunferncia , com B centro na mediatriz do segmento AB, que a C r passa pelos pontos A e B e tangencia a reta Ct r. Chamamos de Ct o ponto de tangncia da circunferncia com a reta r. Se C percorrer livremente a reta r, qualquer possibilidade para o ngulo de viso, , ser dada por uma localizao de C em r. Provaremos que assume o maior valor possvel quando C coincide com Ct . Para isso, mostraremos que, se a medida do ngulo ACtB, ento > para qualquer posio de C diferente de Ct.

46

Se D o ponto de encontro da reta AC com a circunferncia , temos que tambm a medida do ngulo ADB e, denotando por a medida do ngulo CBD, tem-se, no tringulo BCD,

B
r

D C Ct C

+ + 180 = 180. Logo, = + implicando > .

Uma vez verificado que ACtB o ngulo de mximo campo visual, determinaremos agora a distncia d, entre o observador e a base do pedestal, para que esse ngulo seja atingido. Se Q o ponto de interseco da reta AB com r, sendo as retas r e AB, respectivamente, tangente e secante a aplicando potncia no ponto Q encontraremos a distncia d procurada: (QCt)2 = QB.QA ou d2 = (p m)(p m + h) Uma aplicao Em outubro de 1931, aps cinco anos de construo, foi inaugurado no alto do morro do Corcovado o carto de visitas do Rio de Janeiro, a esttua do Cristo Redentor. A altura total da esttua de 30 m, seu pedestal mede 8 m, e admitiremos um observador com 1,70 m de altura. A que distncia esse observador deve ficar da base do pedestal do Cristo Redentor para que o seu ngulo de viso seja o maior possvel? Usando a frmula d2 = (p m)(p m + h) para p = 8 m, m = 1,70 m e h = 30 m, obtemos uma distncia de aproximadamente 15 m. Seria preciso, porm, que o terreno em volta do Cristo fosse aproximadamente plano dentro desse raio.
Baseado no artigo Trigonometria e um antigo problema de otimizao Jos Luiz Pastore Mello, RPM 52
47

Vale para 1, para 2, para 3, ... Vale sempre?

As afirmaes abaixo, sobre nmeros naturais, so verdadeiras para os nmeros 1, 2, 3 e muitos outros. Perguntamos: elas so verdadeiras sempre? Verdadeiro ou falso? 1. n N, n < 100. 2. n N, n2 + n + 41 um nmero primo. 3. n N*, 991n2 + 1 no um quadrado perfeito. 4. n N*, a soma dos n primeiros nmeros mpares n2. 5. n N*, 2n + 2 a soma de dois nmeros primos. Vejamos: 1. n < 100 uma sentena verdadeira para n = 1, n = 2, n = 3 e outros, mas torna-se falsa para qualquer nmero natural maior do que 99. Portanto, n N, n < 100 uma sentena falsa. 2. n2 + n + 41 um nmero primo uma sentena verdadeira para n = 1, n = 2, n = 3 e outros. De fato, ela verdadeira para todos os nmeros naturais menores do que 40 (o que foi verificado por Euler em 1772). Porm, o nmero 402 + 40 + 41 = 40(40 + 1) + 41 = 41 x 41 no primo, mostrando que a sentena n N, n2 + n + 41 um nmero primo uma sentena falsa.
48

3. 991n2 + 1 no um quadrado perfeito uma sentena verdadeira para n = 1, n = 2, n = 3 e, mesmo aps muitas e muitas tentativas, no se acha um nmero que a torne falsa. Pudera! O menor nmero natural n para o qual 991n2 + 1 um quadrado perfeito 12 055 735 790 331 359 447 442 538 767 e, portanto, a sentena n N*, 991n2 + 1 no um quadrado perfeito falsa. 4. A soma dos n primeiros nmeros mpares n2 uma sentena verdadeira para n = 1, n = 2, n = 3 e, como no caso anterior, aps muitas e muitas tentativas, no se acha um nmero natural que a torne falsa. Neste caso, tal nmero no existe, pois, como veremos adiante, essa sentena verdadeira sempre. 5. 2n + 2 a soma de dois nmeros primos uma sentena verdadeira para n = 1, n = 2, n = 3 e, como nos dois exemplos anteriores, aps muitas e muitas tentativas, no se encontra um nmero natural que a torne falsa. Mas agora temos uma situao nova: ningum, at hoje, encontrou um nmero que tornasse a sentena falsa e ningum, at hoje, sabe demonstrar que a sentena verdadeira sempre. A sentena a famosa conjetura de Goldbach feita em 1742, em uma carta dirigida a Euler: Todo inteiro par, maior do que 2, a soma de dois nmeros primos. No se sabe, at hoje, se essa sentena verdadeira ou falsa. Em suma, dada uma afirmao sobre nmeros naturais, se encontrarmos um contra-exemplo, saberemos que a afirmao no sempre verdadeira. E se no acharmos um contra-exemplo? Nesse caso, suspeitando que a afirmao seja verdadeira sempre, uma possibilidade tentar demonstr-la recorrendo ao princpio da induo. Princpio da induo finita Seja S um conjunto de nmeros naturais, com as seguintes propriedades: 1. 0 S 2. k N, se k S, ento k + 1 S. Nessas condies, S = N.

49

Vamos ver como esse princpio nos permite demonstrar que verdadeira a sentena 4: n N*, a soma dos n primeiros nmeros mpares n2. Demonstrao Seja S o conjunto dos nmeros naturais n para os quais a soma dos n primeiros nmeros mpares n2. 1. 1 S, pois a soma dos 1 primeiros nmeros mpares 1 = 12. 2. Vamos supor que k S, isto , que a soma dos k primeiros nmeros mpares seja k2. Vamos provar que k + 1 S, isto , que a soma dos k + 1 primeiros nmeros mpares (k + 1)2. Estamos supondo que 1 + 3 + 5 + ... + 2k 1 = k2 e queremos provar que 1 + 3 + 5 + ... + 2k + 1 = (k + 1)2. Basta observar que 1 + 3 + 5 + ... + (2k 1) + (2k + 1) = k2 + (2k + 1) = (k + 1)2. O princpio da induo nos garante, agora, que S = IN*, ou seja, a afirmao a soma dos n primeiros mpares n2 verdadeira para todos os nmeros naturais maiores do que zero. Uma lenda Aps a criao do mundo, em um mosteiro escondido na ndia, o Grande Criador colocou uma placa de bronze e nela fixou trs bastes cobertos de diamantes. Em um dos bastes, em ordem decrescente de tamanho, colocou 64 discos de ouro. E assim disse aos monges: Transfiram esta pilha de discos para outro basto, movendo, ininterruptamente, um disco de cada vez e nunca permitindo que um disco fique acima de um menor. Quando terminarem esta tarefa e os 64 discos estiverem em outro basto, este templo se reduzir a p e com um estrondo de troves o mundo acabar. Dizem os sbios que o mundo foi criado h 4 bilhes de anos aproximadamente e os monges, desde a criao, esto movendo os discos na razo de um disco por segundo. Ser que veremos o mundo acabar? Como muito difcil imaginar os movimentos feitos com uma pilha de 64 discos, imaginemos uma pilha com Um disco: a transferncia se d com apenas 1 movimento: m1 = 1.
50

Dois discos

Para 2 discos, a transferncia requer 3 movimentos: m2 = 3. Trs discos: m3 = 7.

Quatro discos: m4 = 15.

J podemos deduzir como deslocar n discos com um menor nmero possvel de movimentos. Para tal, observe que o deslocamento do maior disco, do basto em que se encontra inicialmente para um outro, requer que esse segundo basto esteja vazio, pois o maior disco no pode ficar sobre um menor. Como, para se mover o maior disco, nenhum outro pode estar sobre ele, todos os outros discos tero que estar no terceiro basto. Assim, a estratgia com menor nmero de movimentos ser: movem-se n 1 discos para o basto de trs, com mn-1 movimentos; em seguida, move-se o n-simo disco para o outro basto da frente, com 1 movimento; finalmente movem-se os n 1 discos do basto de trs para o da frente, com mn1 movimentos. Tem-se: mn = mn1 + 1 + mn1 = 2mn1 + 1 Faamos uma tabela com o nmero de discos e o nmero de movimentos mnimo para mud-los de um basto para outro:

51

n mn

1 1

2 3

3 7

4 15

5 31

6 63

... ...

Precisamos descobrir o valor de m64 porque, m64 segundos aps a criao do mundo, ele acabar e j se passaram 4 bilhes de anos! Observando a segunda linha da tabela, vemos que os seus nmeros so, a menos de 1: 2, 4, 8, 16, 32, 64, ou seja, 21, 22, 23, 24, 25, 26, o que nos leva a fazer a seguinte conjetura: mn = 2n 1 Essa sentena verdadeira para n = 1, 2, 3, 4, 5, 6, mas ser verdadeira sempre? Tentemos demonstr-la por induo. Seja S o conjunto dos nmeros naturais n tais que n discos so movidos com 2n 1 movimentos. 1. 1 S, pois para 1 disco necessitamos de 1 = 21 1 movimentos. 2. Vamos supor que k S, isto , k discos so removidos com 2k 1 movimentos. Vamos provar que k + 1 S, isto , que mk +1 = 2k +1 1. J vimos que mk + 1 = 2mk + 1. mk + 1 = 2k 1 + 1 + 2k 1 = 2 . 2k 1 = 2k + 1 1, e isso mostra que k + 1 S. O princpio da induo nos garante que n discos podem sempre ser removidos com 2n 1 movimentos e, em particular, m64 = 264 1. E assim ficamos sabendo que, 264 1 segundos aps a criao do mundo, ele terminar. Com um pouco mais de Matemtica ficaremos sabendo se isso ocorrer logo. Faamos alguns clculos. Quantos segundos tem um ano? Resposta:
1 60 60 24 365 = 31 557 600 < 225 = 1024 1024 32 = 33 554 432. 4

52

Exagerando, vamos supor que os monges faam 225 movimentos por ano (na verdade fazem uns milhes a menos). Com isso, o mundo acabar em

264 = 239 anos. 225


239 = 210 x 210 x 29 = 1 024 x 1 024 x 1 024 x 512 > 512 x 109

Passaram-se at hoje 4 bilhes de anos, ou seja, 4 x 109 anos. Podemos ficar tranqilos faltam mais do que 508 bilhes de anos para os monges terminarem sua tarefa isso, supondo que eles no errem no caminho.
Baseado no artigo Vale para 1, para 2, para 3, ... Vale sempre? Renate Watanabe, RPM 09

53

Semelhanas, pizzas e chopes

As histrias que vamos contar envolvem dois amigos que gostam de freqentar bares e restaurantes, alm de discutir problemas de Matemtica. Em pelo menos duas situaes, surgiram interessantes problemas cujas solues, alm de elegantes, so bastante educativas. Primeira histria Augusto e Joo foram a um restaurante para comer pizza. O primeiro pediu uma grande e o segundo, uma mdia e uma pequena, todas do mesmo sabor. Curiosamente, o preo da pizza grande era exatamente igual soma dos preos das pizzas mdia e pequena. Logo aps os pedidos, surgiu naturalmente o problema de saber quem vai comer mais. O fato de os preos a pagar serem iguais no quer dizer nada, porque, nos restaurantes, o preo no costuma ser proporcional quantidade de comida servida. Augusto argumenta que, se tivesse uma rgua, poderia medir os dimetros, calcular as reas e verificar se a rea da pizza grande maior, igual ou menor do que a soma das reas das outras duas. Porm, no havia rgua disponvel. Pensando um pouco, Joo, bom gemetra, declarou ter resolvido o problema, dizendo que assim que as pizzas chegassem diria quem comeria mais, e para isso usaria apenas objetos que estavam em cima da mesa. Augusto, estupefato, duvidou. Como possvel? No temos instrumento de medida algum. Em cima da mesa s h talheres, copos, guardanapos e o cardpio, responsvel por nossa incrvel discusso! A espera no
54

foi longa e as pizzas chegaram. Rapidamente, ento, Joo cortou ao meio cada uma delas, obtendo as reas A, B e C.

A B C

Sobre a mesa (de mrmore) juntou os dimetros para formar um tringulo. Utilizando o canto do cardpio como um modelo para o ngulo reto, Joo verificou que o ngulo, , oposto ao dimetro da maior metade era menor do que 90 e declarou eu como mais. E Augusto, aps pensar alguns momentos, concordou. Qual a explicao? A explicao depende de dois resultados importantes. O primeiro bastante conhecido e o segundo no muito. 1. A razo entre as reas de figuras semelhantes igual ao quadrado da razo de semelhana. 2. Se figuras semelhantes so construdas sobre a hipotenusa e sobre os catetos de um tringulo retngulo, ento a rea da figura maior igual soma das reas das outras duas. A demonstrao desse segundo resultado pode ser vista no artigo Mania de Pitgoras, publicado neste mesmo exemplar. Para concluir que no nosso problema Joo estava certo, observe que, se o ngulo oposto ao lado a do tringulo de lados a, b e c, temos: < 90 a2 < b2 + c2 A < B + C, > 90 a2 > b2 + c2 A > B + C, Portanto, se na nossa histria Joo constatou que o ngulo era menor que 90, ento a rea da semipizza grande era menor que a soma das reas das outras duas metades.
55

Segunda histria Dias depois, Augusto, afobado com o calor, senta em um bar e pede um chope (na verdade, o primeiro de muitos). Nesse lugar, o chope servido em tulipas, que so copos com a forma de um cone invertido. O garom chega com a bebida ao mesmo tempo que Joo encontra seu amigo. Como vai, Joo? Sente e tome rpido a metade deste copo. Eu tomo a outra metade. A fisionomia de Joo mostra alguma tristeza. Como determinar a altura do nvel da bebida quando um copo cnico contm a metade do seu contedo? Augusto ento alivia a situao. Meu caro amigo, para esse problema, seus artifcios so insuficientes. Eu hoje vim prevenido e trouxe uma rgua e uma calculadora. Desculpe a brincadeira e vamos juntos resolver o nosso problema. Augusto ento saca de sua rgua, calculadora, caneta e sobre um guardanapo mostra a soluo sob o olhar de um estupefato garom. Observe, Joo, que o copo tem 20 cm de altura. Desejamos obter a altura da superfcie do lquido que corresponda metade do volume do copo. Para isso, precisamos recordar dois outros fatos: 3. Toda seo paralela base de um cone forma um outro cone semelhante ao primeiro. 4 . A razo entre o volume de slidos semelhantes igual ao cubo da razo de semelhana. Augusto continua sua explicao. Se voc tiver tomado uma parte do contedo deste copo, teremos aqui, por 2, dois objetos semelhantes: o cone formado pelo lquido e o prprio copo. A razo de semelhana entre esses dois cones a razo entre suas alturas, ou seja, h/20. Como desejamos que o lquido tenha a metade do volume do copo, por 3, podemos escrever:
3 h 1 1 h =3 . = , ou seja, 20 2 2 20

20
3

Assim, a altura que corresponde metade do volume do copo h = 10 4 cm.

56

Joo concorda com a perfeita explicao, mas repara que a resposta no resolve ainda o problema porque ele no tem a menor idia de quanto
10 3 4 . E ento Augusto, com a sua calculadora e seu sorriso irnico, diz: Ah! bom saber que esse valor d aproximadamente 16 cm.

Bem. O problema foi resolvido e o chope, j meio quente, foi adequadamente dividido. Falta apenas o final da histria. Nessa altura, as pessoas das outras mesas ouviam atentamente nossos personagens com um misto de admirao e espanto. Nisso, Joo faz uma descoberta, que anuncia em alto e bom som: Esse problema me revela que quando somos servidos em tulipas com 4 cm de colarinho estamos tomando apenas metade do contedo do copo. Assim, se eu digo que tomei 10 chopes, na verdade tomei 5, mas paguei 10!. E foram expulsos do bar.
Baseado no artigo Semelhanas, pizzas e chopes Eduardo Wagner, RPM 25

57

Sorrisos, sussurros, antenas e telescpios

Elipses, sorrisos e sussurros Para cuidar do sorriso dos pacientes, os dentistas utilizam uma luminria com espelho elptico. De maneira diferente dos holofotes comuns, como os faris de carro, que refletem os raios luminosos em uma mesma direo (valendo-se, para isso, de um espelho parablico), os holofotes dentrios se valem de espelhos elpticos para concentrar os raios luminosos emitidos pela lmpada em um determinado ponto: o dente a ser tratado. Isso possvel devido ao fato de que, como veremos adiante, todo raio emitido em um dos focos se dirigir, aps a reflexo no espelho elptico, exatamente para o outro foco (estamos pensando na elipse plana com focos na lmpada e no dente sendo tratado e parcialmente contida no espelho elptico). Isso tambm explica o funcionamento de diversos aparelhos de emisso de raios usados em tratamentos mdicos, como, por exemplo, o de radioterapia, cujos raios devem destruir os tecidos doentes sem afetar os tecidos sadios que se encontram ao redor. J as salas de sussurros so construes de forma oval onde esto marcados dois pontos no cho. Duas pessoas em p, uma em cada um desses pontos, podem se comunicar em voz sussurrada, inaudvel no restante da sala. A forma da sala de fundamental importncia. Ao projet-la, fixam-se dois pontos P e Q, que ficam
58

na altura da cabea das pessoas que vo se comunicar. A seguir, toma-se uma elipse E que admita P e Q como focos, e a sala construda de tal maneira que qualquer plano que passe por esses pontos intercepte a sala segundo uma elipse congruente com a escolhida.

A elipse de focos P e Q por definio o conjunto dos pontos de um plano por P e Q tais que a soma das distncias do ponto aos focos constante. Assim, todas as ondas sonoras emitidas em um dos focos, ao se refletirem nas paredes da sala, chegaro ao segundo foco tendo percorrido a mesma distncia, ou seja, ao mesmo tempo, o que, sem dvida, proporciona uma amplificao natural do som, explicando o funcionamento das salas de sussurros. Vejamos ento uma propriedade da elipse da qual decorre a propriedade de reflexo mencionada. Propriedade Seja uma elipse E com focos P e Q e seja um ponto X E. Nesse caso a reta r, tangente a E em X, forma ngulos iguais com PX e QX.
a

X
a

Para no quebrar a continuidade do estudo das propriedades de reflexo das outras cnicas, colocamos a demonstrao da propriedade da elipse no final do artigo. Para concluir que a propriedade da elipse garante os fenmenos anteriormente citados, lembremos duas leis fsicas sobre a reflexo. A primeira diz que o ngulo de incidncia e o ngulo de reflexo em um plano so iguais. A outra lei diz que a reflexo em cada ponto de uma superfcie se comporta como se fosse no plano tangente superfcie, no respectivo ponto. Logo, a propriedade garante os fenmenos de reflexo mencionados. Por que as antenas so parablicas? A palavra parbola est, para os estudantes do ensino mdio, associada ao grfico da funo do segundo grau. Entretanto, quase todos conhecem as antenas parablicas, mas nem todos fazem ligao entre uma coisa e outra. Os espelhos dos telescpios e dos faris dos automveis tambm so parablicos. Por qu?
59

Consideremos uma reta d e um ponto F. A parbola de foco F e diretriz d , por definio, o conjunto de todos os pontos do plano definido por F e d cuja distncia reta d igual distncia ao ponto F. O segmento EF chama-se parmetro da parbola e o ponto mdio V, mdio de EF, o vrtice da parbola. A equao da parbola

D E d V F

Em um sistema de coordenadas, no difcil encontrar a equao da parbola, dados o foco e a diretriz. Tomemos F = (0, p) como foco e y = p como diretriz. Se P = (x, y) tal que PF = PD, temos:
x 2 + ( y p ) 2 = y + p.

Elevando ao quadrado e cancelando os termos iguais dos dois lados, obtemos: x 2 = 4 py ou y =

1 2 x , o que mostra que a equao de uma 4p


y

parbola da forma y = ax2 (uma funo polinomial de grau 2).

F = (0, p) y = -p

P = (x, y)
x

Reciprocamente, dada uma funo da forma y = ax2, fcil provar que qualquer um de seus pontos possui distncia ao ponto (0, distncia reta y =

1 ) igual 4a

1 , o que mostra que o grfico de y = ax2 uma 4a parbola. Com um pouco mais de trabalho, o leitor poder demonstrar que o grfico de y = ax2 + bx + c (com a 0) tambm uma parbola,
60

exatamente igual ao grfico de y = ax2, mas agora com vrtice no ponto


( b b 2 4ac , ). 2a 4a

Antenas e espelhos Vamos voltar agora s nossas perguntas iniciais. Por que as antenas que captam sinais do espao so parablicas? Por que os espelhos dos faris dos carros so parablicos? Nas antenas os sinais que recebemos (ondas de rdio ou luz) so muito fracos. Por isso, necessrio capt-los em uma rea relativamente grande e concentr-los em um nico ponto para que sejam naturalmente amplificados. Portanto, a superfcie da antena (ou do espelho) deve ser tal que todos os sinais recebidos de uma mesma direo sejam direcionados para um nico ponto aps a reflexo. Nos faris dos carros usa-se a propriedade em direo contrria: os raios de luz emitidos pela lmpada se refletem no espelho parablico e saem paralelos iluminando uma regio maior.

antena

farol

Esses fenmenos so garantidos pela propriedade enunciada a seguir. No demonstraremos aqui essa propriedade, uma vez que sua demonstrao anloga correspondente para a elipse (consultar a RPM 33, p. 14). Propriedade Consideremos agora um ponto P qualquer da parbola de foco F e diretriz d. Se PD perpendicular reta d, ento a reta tangente parbola em P forma ngulos iguais com PF e PD.

P
aa

Q
d

D
61

A hiprbole e os telescpios Dados dois pontos F e F, a hiprbole de focos nesses pontos o conjunto dos pontos de um plano por F e F cuja diferena das distncia a F e F uma constante. De modo anlogo elipse e parbola, a hiprbole tambm tem uma propriedade de reflexo que conseqncia da enunciada a seguir (os interessados podero encontrar uma demonstrao na RPM 34, p. 27). Propriedade Seja uma hiprbole com focos F e F e seja um ponto P da hiprbole. Nesse caso, a reta t, tangente hiprbole em P, forma ngulos iguais com PF e PF. A A propriedade garante que um raio de luz proveniente de um ponto A, de forma que P F F a reta AP passe pelo foco F, que incide num espelho hiperblico em P, seja refletido de modo a passar pelo outro foco F. O telescpio refletor nada mais do que um espelho parablico no fundo de um tubo. Os raios provenientes de um corpo celeste distante (estrela, galxia, planeta, etc.) formam um feixe praticamente paralelo, que se reflete no espelho e vai formar a imagem do objeto no foco F.

O problema agora que, para observar essa imagem, o observador teria de estar com seu olho no foco da parbola, mas isso impossvel na prtica. Isaac Newton (1642-1727) resolveu esse problema em seu telescpio refletor, colocando um espelho plano E entre o espelho parablico e o foco F. Com isso, os raios que iriam formar a imagem em F so novamente refletidos e vo formar essa imagem num ponto F fora do tubo do telescpio, onde se posiciona o observador.

E F F

62

Em 1672 o astrnomo francs Cassegrain props a utilizao de um espelho hiperblico E, em lugar do espelho plano de Newton. Um dos focos da hiprbole coincide com o foco F da parbola.

Agora os raios que iriam formar a imagem no foco F so refletidos pelo espelho E e formaro essa imagem no outro foco F da hiprbole. Para compreender a vantagem desse espelho hiperblico de Cassegrain sobre o espelho plano de Newton, devemos observar que o espelho plano no pode ficar muito prximo do foco F, sob pena de o ponto F ficar dentro do telescpio; em conseqncia, o espelho plano precisa ser de razovel tamanho, o que resulta num bloqueio significativo da luz incidente no espelho parablico que forma a parte principal do telescpio. O espelho de Cassegrain, pelo contrrio, pode ser construdo mais prximo ou mais afastado do foco F, mantendo-se fixa a distncia FF entre os focos da hiprbole; em conseqncia, o tamanho desse espelho pode ser maior ou menor. A distncia entre os focos F e F tambm pode ser alterada sem mudar a posio do foco F. A combinao desses fatores permite grande flexibilidade na montagem do refletor hiperblico E, adequando-a, assim, s exigncias das observaes. Demonstrao da propriedade da elipse Lembramos que, tal como na circunferncia, uma reta r tangente a uma elipse E no ponto X se, e somente se, r E = {X}. Denotando a distncia entre dois pontos R e S por d(R, S) e caracterizando a elipse E como o lugar geomtrico dos pontos X que satisfazem a propriedade mtrica, d(X, P) + d(X, Q) = k (constante), segue-se que um ponto A no estar na elipse se e somente se d(A, P) + d(A, Q) k. Logo, uma reta r ser tangente elipse E em um ponto X se e somente se intersectar E em X e qualquer que seja o ponto A em r, A X, se tenha:
63

d(A, P) + d(A, Q) d(X, P) + d(X, Q) Seja, agora, um ponto X na elipse E e tomemos uma reta r (bissetriz de um dos ngulos formados pelas retas PX e QX) passando por X de tal forma que o ngulo entre PX e r seja igual ao ngulo entre QX e r. Se mostrarmos que r tangente a E em X, teremos mostrado a propriedade, devido unicidade da tangente elipse por um de seus pontos. Seja X um ponto de E, logo d(X, P) + d(X, Q) = k. Tomemos sobre r um ponto A X e consideremos o ponto P, simtrico de P em relao a r. A reta r ento mediatriz de PP. Logo, d(X, P) = d(X, P) e tambm d(A, P) = d(A, P). Por construo, a reta r faz ngulos iguais com XP e XQ e, pela simetria, os ngulos AXP e AXP so tambm iguais. Da, os segmentos XQ e XP fazem ngulos iguais com r e, portanto, os pontos P, X e Q so colineares. Segue-se ento: k = d(X, P) + d(X, Q) = d(X, P) + d(X, Q) = d(P, Q) < d(A, P) + d(A, Q) = d(A, P) + d(A, Q), ou d(A, P) + d(A, Q) > k, o que mostra que A E. Conclumos que X o nico ponto de r que pertence elipse, o que mostra que essa reta tangente em X a essa elipse.
Baseado nos artigos Por que as antenas so parablicas? Eduardo Wagner, RPM 33 A hiprbole e os telescpios Geraldo vila, RPM 34 Elipses, sorrisos e sussurros Renato J. C. Valladares, RPM 36

X
r

64

A Matemtica do GPS

O que o GPS e como funciona? O estudo da esfera e seus elementos fica naturalmente contextualizado quando exploramos sua associao com o globo terrestre. Conceitos geogrficos como paralelos, meridianos, latitudes, longitudes e fusos horrios esto baseados em importantes idias geomtricas, e o estabelecimento das relaes entre eles conduz a problemas geomtricos relevantes. Aqui veremos a fundamentao matemtica necessria para o entendimento de um moderno sistema de navegao por satlites, o GPS. A sigla GPS a abreviatura para Global Positioning System (sistema de posicionamento global). O sistema NAVSTAR, nome oficial dado pelo Departamento de Defesa dos Estados Unidos ao GPS, consiste em um segmento espacial, formado por 24 satlites, um segmento de controle, formado pelas estaes terrestres de gerenciamento, e um segmento do usurio. O projeto foi iniciado em 1973 pelo Departamento de Defesa dos Estados Unidos com propsitos militares, e logo estendido tambm para uso civil. Hoje em dia, com auxlio do piloto automtico e do GPS, uma aeronave civil capaz de percorrer distncias transatlnticas e pousar sem a interferncia do piloto com erro de alguns centmetros com o eixo da pista. O GPS tem se mostrado til em diversas situaes, tais como: roteiro de viagens; monitoramento

65

de abalos ssmicos; meteorologia; localizao para resgate; monitoramento de caminhes de carga. Os satlites orbitam em torno da Terra a uma altura aproximada de 20.200 km acima do nvel do mar, em seis rbitas estveis e predeterminadas, com quatro satlites em cada rbita. Percorrem uma rbita completa a cada 12 horas e cada satlite tem 28 de visualizao sobre a Terra. Isso assegura que todo ponto da superfcie terrestre, em qualquer instante, esteja visualizado por pelo menos quatro satlites. Vrias reas da Terra so, por alguns momentos, visualizados por at dez satlites. Todos os satlites so controlados pelas estaes terrestres de gerenciamento. Uma estao master, com o auxlio de cinco estaes de gerenciamento espalhadas pelo planeta, monitora o desempenho total do sistema, corrigindo as posies dos satlites e reprogramando o sistema, quando necessrio. Aps o processamento de todos esses dados, as correes e sinais de controle so transferidos de volta para os satlites. Cada um dos satlites do GPS transmite por rdio um padro fixado, que recebido por um receptor na Terra (segmento do usurio), funcionando como um cronmetro extremamente acurado. O receptor mede a diferena entre o tempo que o padro recebido e o tempo que foi emitido. Essa diferena, no mais do que um dcimo de segundo, permite que o receptor calcule a distncia ao satlite emissor, multiplicando a velocidade do sinal (aproximadamente 2,99792458.108 m/s a velocidade da luz ) pelo tempo que o sinal de rdio levou do satlite ao receptor. Essa informao localiza uma pessoa sobre uma imaginria superfcie esfrica com centro no satlite e raio igual distncia acima calculada. Cada satlite programado para emitir o que se chama efemride, que informa a sua posio exata, naquele instante, em relao a um fixado sistema ortogonal de coordenadas. Tal posio permanentemente rastreada, conferida e processada pelas estaes terrestres. Com a posio do satlite e a distncia acima calculada, obtm-se a equao da imaginria superfcie esfrica. Coletando-se sinais emitidos por quatro satlites, o receptor determina a posio do usurio calculando-a como interseco das quatro superfcies esfricas obtidas. A localizao dada, no em coordenadas cartesianas, mas por meio das coordenadas geogrficas (latitude, longitude e a elevao).
66

A preciso do tempo essencial na operao do GPS. Um erro de um microssegundo (106 segundos) no registro do lapso de tempo desde a transmisso at a sua recepo resulta num erro de 300 metros. Unidades receptoras do GPS extremamente precisas (e caras!) podem determinar sua posio a menos de um metro. A superfcie esfrica em coordenadas cartesianas Em um sistema ortogonal de coordenadas cartesianas em trs dimenses, a distncia entre os pontos P = (x, y, z) e C = (u, v, w) dada pela frmula
d ( P, C ) = ( x u ) 2 + ( y v) 2 + ( z w) 2 . Portanto, sendo r um nmero real positivo e C = (u, v, w) um ponto fixado, a superfcie esfrica S de centro C e raio r, que o conjunto dos pontos do espao cuja distncia a C igual a r, tem equao (denominada equao reduzida de S):

(x u)2 + (y v)2 + (z w)2 = r2.

(1)

Desenvolvendo os quadrados em (1), obtemos (a chamada equao geral de S) x2 + y2 + z2 2xu 2yv 2zw + u2 + v2 + w2 r2 = 0 que uma equao da forma x2 + y2 + z2 + ax + by + cz + d = 0 onde a, b, c, d so nmeros reais. A interseo de duas superfcies esfricas de centros distintos vazia, ou um ponto ou uma circunferncia, conforme a distncia entre os seus centros maior que, igual a ou menor que a soma dos raios. O teorema a seguir desempenha um papel importante na fundamentao matemtica do funcionamento do GPS: Se quatro superfcies esfricas se intersectam e seus centros so no coplanares, ento essa interseco consiste em um nico ponto. Demonstrao Sejam S1, S2, S3 e S4 superfcies esfricas de centros C1, C2, C3 e C4, satisfazendo as hipteses. Sendo x2 + y2 + z2 + ajx + bjy + cjz + dj = 0 as equaes gerais de Sj, onde j = 1, 2, 3, 4, ao subtrairmos essas equaes duas a duas, obtemos equaes lineares em x, y e z, uma vez que os termos x2, y2 e z2 so eliminados.
67

(2)

(3)

Tal equao linear determina um plano que contm a correspondente interseco. Por exemplo, subtraindo as equaes de S1 e S2, obtm-se a equao de um plano que contm S1 S2. Considerando-se os planos que contm S1 S2, S1 S3 e S1 S4, temos que, se P = (x, y, z) est em S1 S2 S3 S4, ento (x, y, z) soluo do sistema linear (a1 a2)x + (b1 b2)y + (c1 c2)z + (d1 d2) = 0 (*) (a1 a3)x + (b1 b3)y + (c1 c3)z + (d1 d3) = 0 (a1 a4)x + (b1 b4)y + (c1 c4)z + (d1 d4) = 0 A demonstrao do teorema estar terminada se mostrarmos que o sistema (*) tem uma nica soluo, pois a existncia de dois pontos distintos em S1 S2 S3 S4 acarretaria duas solues distintas do sistema linear (*). Sendo Cj = (uj, vj, wj) o centro de Sj, j = 1, 2, 3, 4, comparando as equaes (2) e (3), temos aj = 2uj, bj = 2vj, cj = 2wj de modo que a1 a2 a1 a3 a1 a4 b1 b2 b1 b3 b1 b4 c1 c2 u2 u1 v2 v1 c1 c3 = 8 u3 u1 v3 v1 c1 c4 u4 u1 v4 v1 w2 w1 w3 w1 w4 w1

Como C1, C2, C3, C4 so no coplanares, segue que o determinante direita no nulo e, portanto, (*) um sistema linear com determinante no nulo, tendo assim uma nica soluo. Note que o simples fato de o sistema linear (*) ter uma nica soluo, o que equivale a dizer que os centros so no coplanares, no acarreta necessariamente que a interseco das quatro superfcies esfricas consiste em um nico ponto P. A hiptese S1 S2 S3 S4 essencial para a validade do teorema. interessante observar que, na situao real do GPS, essa hiptese comprovada pela existncia do prprio usurio! As coordenadas geogrficas de um ponto do espao Fixemos um sistema ortogonal de coordenadas cartesianas com origem O no centro da Terra, o eixo Oz positivo apontando na direo do Plo Norte, o plano Oxy sendo o plano do equador com o eixo Ox positivo cortando o meridiano de Greenwich e o eixo Oy positivo cortando o meridiano de longitude 90E.

68

Dado um ponto P = (x, y, z) do espao, sejam e as medidas dos ngulos assinalados na figura a seguir.

(0, 0, z) = B
q = m(AOP) j = m(COA)

P = (x, y, z) 0
j q

(x, 0, 0) = C

A = (x, y, 0)

Quando P est sobre a superfcie terrestre, os valores e acima indicados correspondem exatamente habitual latitude e longitude do ponto P e, por isso, manteremos a mesma nomenclatura para e . A diferena entre OP = d (O, P ) = x 2 + y 2 + z 2 e o raio da Terra chamada elevao (ou altitude) de P = (x, y, z). A latitude, a longitude e a elevao so chamadas coordenadas geogrficas do ponto P. Vejamos como relacion-las com as coordenadas cartesianas de P. No tringulo retngulo OPB da figura acima, temos: cos(90 ) = sen = OB = OP z x2 + y 2 + z 2 .

Essa expresso atribui a um nico valor entre 0 e 90 quando z > 0 e um nico valor entre 90 e 0 quando z < 0. No primeiro caso, dizemos que a latitude de P N (norte), enquanto no segundo a latitude de P () S (sul). Por outro lado, no tringulo retngulo OAC temos sen = AC = OA y x +y
2 2

e cos =

OC = OA

x x + y2
2

Essas expresses definem um nico entre 0 e 180 quando y > 0 e dizemos que a longitude de P E (leste). Quando y < 0, assume um nico valor entre 180 e 0 e, nesse caso, a longitude de P () W (oeste).
69

Como exemplo, vamos determinar as coordenadas geogrficas do ponto P cujas coordenadas cartesianas so dadas, em metros, por

P = (3 3.106 , 3.106 , 6 3.106 ) .


Temos x 2 + y 2 + z 2 = 27.10 12 + 9.10 12 + 108.10 12 = 144.10 12 e x + y2 = 27.1012 + 9.1012 = 36.1012.
2

Logo, sen =

6 3.106 12.10
6

3 ; portanto, = 60. 2

1 3 3.106 3 e cos = , obtemos = 6 6 2 2 6.10 6.10 = 30. Assim, as coordenadas geogrficas de P so = 60 N e = 30 W. Supondo o raio da Terra igual a 6,4.106 metros, temos que a elevao de P mede 12.106 6,4.106 = 5,6.106 metros.

Como sen =

3.106

Uma situao real O exemplo abaixo retrata uma situao real em que um usurio do GPS detectado por quatro satlites. A tabela indica as efemrides (em metros) de cada satlite tomadas em relao ao nosso fixado sistema ortogonal de coordenadas cartesianas. x Satlite 1 Satlite 2 Satlite 3 Satlite 4 1,877191188.106 1,098145713.107 2,459587359.107 3,855818937.106 y 1,064608026.107 1,308719098.107 4,336916128.106 7,251740720.106 z 2,428036099.107 2,036005484.107 9,090267461.106 2,527733606.107

O receptor GPS registra os seguintes lapsos de tempo (em segundos) entre a transmisso e a recepo do sinal de cada satlite. Satlite 1 0,08251731391 Satlite 2 0,07718558331 Satlite 3 0,06890629029 Satlite 4 0,07815826940

Note que as informaes transmitidas no sistema GPS envolvem, por uma questo de preciso, dez ou mais dgitos, tornando imprescindvel a utilizao de calculadoras ou softwares com capacidade de resolver sistemas
70

lineares com coeficientes dessa ordem. Outra alternativa, abrindo mo da preciso, trabalhar com um nmero menor de dgitos e utilizar a notao cientfica. Multiplicando cada lapso de tempo pela velocidade da luz, 2,99792458.108 m/s, obtemos a distncia entre o receptor e cada satlite. Isso permite escrever as equaes reduzidas das superfcies esfricas centradas em cada satlite e raios iguais s distncias calculadas: S1 : (x 1,8.106 )2 + (y + 10,6.106 )2 + (z 24,2.106 )2 = 611,9.1012 S2 : (x 10,9.106 )2 + (y + 13.106 )2 + (z 20,3.106 )2 = 535,4.1012 S3 : (x 24,5.106 )2 + (y + 4,3.106 )2 + (z 9.106 )2 = 426,7.1012 S4 : (x 3,8.106 )2 + (y 7,2.106 )2 + (z 25,2.106 )2 = 549.1012 Desenvolvendo os quadrados, obtemos as respectivas equaes gerais, e o sistema linear (*) dado por 18,2x 4,88y 7,84z 76,52.106 = 0 45,43x + 12,61y 30,38z 185,23.106 = 0 3,95x + 35,79y + 1,99z 62,95.106 = 0 cuja nica soluo x = 0,5660.107, y = 0,0978.107 e z = 0,2775.107. O ponto P com essas coordenadas cartesianas pertence simultaneamente s quatro imaginrias superfcies esfricas e suas coordenadas geogrficas, calculadas como no pargrafo anterior (considerando o raio da Terra medindo 6,378164.106 metros), so Latitude: = 26 N; Longitude: = 10 E; Elevao: 919,71 metros. Consultando um atlas geogrfico ou um globo terrestre, identificamos a posio desse usurio do GPS como sendo a cidade de Djanet, localizada nos Montes Tssili, na fronteira entre Arglia e Lbia.
Baseado no artigo A Matemtica do GPS Srgio Alves, RPM 59

71

O problema do amigo oculto

Por ocasio das festas de fim de ano, um grupo de 9 pessoas resolveu planejar a clebre brincadeira do amigo oculto (ou secreto). Foi escrito o nome de cada pessoa em um papelzinho, e procedeu-se ao sorteio, para determinar quem iria dar presente a quem. Feito o sorteio, logo apareceu algum que tirou a si mesmo. Sendo contra as regras da brincadeira que algum presenteie a si mesmo, e para preservar o sigilo, foi necessrio proceder a outro sorteio. No segundo sorteio, o mesmo fenmeno ocorreu, dessa vez com outra pessoa. Uma das pessoas presentes levantou a questo: Isso vai ficar acontecendo a vida toda? Qual a probabilidade de isso acontecer?. Na realidade, essa uma ocorrncia de um clebre problema de Anlise Combinatria, o das chamadas permutaes caticas. Cada sorteio define uma funo f do conjunto das 9 pessoas em si mesmo. f(x) = y significa que x deve presentear y. Como duas pessoas diferentes no podem tirar o mesmo amigo oculto (o sorteio feito sem reposio), e todas as 9 pessoas sero presenteadas, f uma bijeo do conjunto A das 9 pessoas sobre si mesmo, ou seja, uma permutao desse conjunto. Algum ser amigo oculto de si mesmo quando existir em A um certo x tal que f(x) = x. Na nomenclatura usual de funes, um tal x chamado ponto fixo de f. O problema agora consiste em determinar, dentre o total das 9! = 362.880 permutaes dos elementos de A, quantas so
72

as que tm ponto fixo correspondentes aos sorteios fracassados e quantas no tm ponto fixo correspondentes aos sorteios que deram certo. Pode parecer estranho que justamente os casos que aqui do certo que so chamados, na nomenclatura clssica, de permutaes caticas. O motivo que essa nomenclatura se prende interpretao de permutaes como arrumaes dos elementos 1, . . .,9 nos lugares de 1 a 9; uma permutao catica ento uma permutao em que todo o mundo est fora de seu lugar natural. Antes de resolver o problema, vamos introduzir uma forma de representar permutaes. Adotando o clssico smbolo a b para designar que f(a) = b, e numerando as pessoas de 1 a 9, uma possvel permutao , por exemplo: 1 8 2 1 3 3 4 9 5 7 6 6 7 4 8 2 9 5 Observe que podemos colocar essas informaes na seguinte ordem: l82l 33 49574 66 Note que as pessoas 1; 8; 2; 1 formam, nessa ordem, um ciclo (de tamanho 3): 1 presenteia 8, que presenteia 2, que presenteia 1. Representaremos esse ciclo por (182). O mesmo ciclo poderia ser representado tambm por (821) ou (218) (certo?), mas no por (128), que significaria: 1 2 8 1, que diferente. Situao anloga ocorre com os elementos 4; 9; 5; 7, que formam o ciclo (4957). Os pontos fixos 3 e 6 podem ser considerados como ciclos de tamanho 1. Desse modo, essa permutao pode ser representada por: (182) (3) (4957) (6). Repare que, se trocarmos os ciclos de lugar, nada muda nas informaes, de modo que a mesma permutao poderia ser representada, por exemplo, por (4957) (6) (3) (182). J trocar a ordem das pessoas dentro dos ciclos pode alterar ou no a permutao, como vimos. Fica claro agora que, quando procuramos as permutaes que no possuem pontos fixos, estamos procurando quais as permutaes que no apresentam ciclos de tamanho 1. Para adquirir uma familiaridade com o problema, comecemos por examinar como seria o problema com nmeros menores. Chamando de n o nmero de pessoas, e de Kn o nmero de permutaes do conjunto dessas pessoas, que no tm elementos fixos, ento a probabilidade de que o sorteio d certo ser: pn = Kn/n!.
73

Para n = 1, a nica permutao que existe : 1 1, ou, na nossa notao: (1), a qual tem ponto fixo. claro ento que K1 = 0 e p1 = 0. Para n = 2, as duas permutaes so: (1) (2) e (12). S a segunda catica; portanto: K2 = 1 e p2 = 1/2. Para n = 3, existem 6 permutaes: (1)(2)(3), (1)(23), (2) (13), (3) (12), (123) e (132). Dessas, s as duas ltimas no tm ciclos de tamanho 1, isto , no tm pontos fixos. Logo, K3 = 2 e p3 = 1/3. claro que no podemos contar dessa maneira para o caso n = 9, com um total de mais de 300 mil permutaes. Vamos ento fazer um raciocnio mais sutil, para esse caso. Imaginemos todas as permutaes caticas das 9 pessoas. Fixemos a ateno na pessoa de nmero 9. Em qualquer das 9! permutaes, essa pessoa tem que estar em algum ciclo de tamanho maior que 1 (lembre-se de que no h ponto fixo numa permutao catica!). Chamemos ento de D9 o nmero de permutaes caticas (das 9 pessoas) em que a pessoa 9 est num ciclo de tamanho 2, e de B9 o nmero de permutaes caticas (das 9 pessoas) em que a pessoa 9 est num ciclo de tamanho maior que 2. claro que K9 = B9 + D9. Se tomarmos uma permutao catica em que 9 esteja num ciclo de tamanho maior que 2 (por exemplo, (15) (3246) (798)) e suprimirmos o 9, obteremos uma permutao catica das 8 pessoas restantes (no exemplo anterior, obteramos: (15) (3246) (78)); por outro lado, o caminho inverso ou seja, inserir o 9 nessa permutao catica das 8 primeiras pessoas, para obter uma permutao catica das 9 originais pode ser feito de 8 maneiras diferentes, como vemos no exemplo dado: (195)(3246)(78), ou (159)(3246)(78), ou (15)(39246)(78), ou (15)(32946)(78), ou (15)(32496)(78), ou (15)(32469)(78), ou (15)(3246)(798), ou (15)(3246)(789)). Na realidade, o processo descrito nesse caminho inverso consiste em substituir cada flecha a b por a 9 b. No exemplo, fizemos isso, sucessivamente, com as flechas 1 5, 5 1, 3 2, 2 4, 4 6, 6 3, 7 8, 8 7, que so as oito flechas da permutao. Portanto, a concluso que cada permutao catica de 8 pessoas gera, por esse processo, 8 permutaes caticas de 9 pessoas nas quais a pessoa 9 est num ciclo de tamanho maior que 2, ou seja: B9 = 8K8. Se tomarmos agora uma permutao catica em que 9 esteja num
74

ciclo de tamanho igual a 2 (por exemplo, (178) (3426) (59)) e suprimirmos o 9, obteremos no uma permutao catica das 8 pessoas restantes, e sim uma permutao das 8 pessoas com um nico ponto fixo (no exemplo anterior, obteramos: (178) (3426) (5)). Essa pode ser olhada como um ponto fixo (no caso, o 5) justaposto a uma permutao catica das outras 7 pessoas. Como existem 8 candidatos a serem o ponto fixo, conclui-se que cada permutao catica de 7 pessoas gerar, pelo processo de acrescentar o 9 ao ponto fixo, 8 permutaes caticas de 9 pessoas nas quais 9 est num ciclo de tamanho 2, ou seja: B9 = 8K7. Como K9 = B9 + D9, segue que: K9 = 8K8 + 8K7. O leitor pode agora repetir o mesmo raciocnio para n em vez de 9, para concluir que: Kn = (n 1)Kn1 + (n 1)Kn2. Dividindo por n! e simplificando, passa-se s probabilidades que nos interessam, obtendo:

1 1 pn = (1 ) pn 1 + pn 2 . n n

(*)

Essa uma frmula de recorrncia, que permite calcular pn, uma vez que j saibamos as probabilidades anteriores pn1 e pn2 . Por exemplo:

1 1 2 1 1 1 p3 = (1 ) p2 + p1 = + 0 = . 3 3 3 2 3 3 1 1 3 1 1 1 3 p4 = (1 ) p3 + p2 = + = . 4 4 4 3 4 2 8
n pn

0 = 0,00000 1/2 = 0,50000 1/3 = 0,33333 3/8 = 0,37500 11/30 = 0,36667 53/144 = 0,36806

Continuando, encontram-se:

2 3 4 5 6

e assim por diante.


75

Para obter uma frmula geral, observemos que a frmula (*) pode ser escrita como: pn pn-1 = (1/n)(pn-1 pn-2), ou ainda, chamando pn pn1 de dn , como: dn = (l/n)dn1. Observando ainda que d2 = p2 p1 = 1/2 0 = 1/2, tem-se, sucessivamente:

d2 =

1 1 1 1 1 1 1 1 1 ; d3 = d 2 = = ; d 4 = d3 = = ; etc. 2! 3! 3! 2! 3! 4 4 3! 4!

De um modo geral: d n = (1) n

1 . n!
1 1 1 1 + K + (1) n 2! 3! 4! n!

Por fim, a relao pn pn1 = dn acarreta:


pn = ( p2 p1 ) + ( p3 p2 ) + K + ( pn pn 1 ) =

que a frmula geral que resolve o problema. Observando a tabela de valores de pn, o leitor vai reparar que esses valores crescem (cada vez menos) quando n passa de mpar para par, e diminuem (cada vez menos) quando n passa de par para mpar, sugerindo que pn deva tender a se aproximar de um certo valor (entre 0,36667 e 0,36806), ora por excesso, ora por falta. Isso de fato verdade. Esse valor l/e, em que e 2,71828 a clebre base dos logaritmos naturais. Se o leitor tiver acesso a uma calculadora com a tecla ex, poder verificar que l/e = e1 0,36788. A justificativa desse fato pode ser feita atravs da frmula de Taylor para a funo exponencial, estudada em Clculo Diferencial, segundo a qual: ex = 1 + (x/l!) + (x2/2!) + ... . Em suma, pode-se dizer que a probabilidade de que o sorteio do amigo oculto d certo oscila em torno de aproximadamente 37% (conseqentemente 63% de no dar certo), estando j bem perto desse valor a partir de 5, pessoas.
Baseado no artigo O problema do amigo oculto Jos Paulo Carneiro, RPM 28 Sobre o mesmo assunto, veja tambm Amigo oculto, por C. G. Tamm Moreira, RPM 15 e Uma pequena prola de Euler, por Geraldo Garbi, RPM 50
76

O princpio da casa dos pombos

A Anlise Combinatria, que poderia ser chamada de arte de contar, inspira, freqentemente, temor ou desagrado aos alunos do ensino mdio, s voltas com problemas mecnicos envolvendo combinaes, permutaes, arranjos, etc. No entanto, trata-se de uma parte fascinante da Matemtica que contm problemas de enunciado extremamente simples, mas que exigem, por vezes, para sua soluo, raciocnios penetrantes e engenhosos. Grandes matemticos, como Euler, atacaram problemas de Combinatria. Hoje, com o rpido desenvolvimento da chamada Matemtica Finita, principalmente devido ao uso dos computadores, a Combinatria cresce rapidamente, atraindo a ateno de muitos matemticos jovens e promissores. Um dos princpios bsicos da Combinatria o chamado princpio da casa dos pombos, ou ainda princpio das gavetas de Dirichlet, que diz simplesmente: Se forem dados n objetos, n > 2, a serem colocados em, no mximo, (n 1) gavetas, ento uma delas conter pelo menos dois objetos. Certamente poucos duvidaro da veracidade do princpio. Para os mais cticos pode-se argumentar por reduo ao absurdo. Se cada uma das gavetas contiver,
77

no mximo, um objeto, o nmero total de objetos colocados nelas ser, no mximo, (n 1), o que uma contradio. Uma aplicao trivial do princpio :

Exemplo 1
Dado um conjunto de 13 pessoas, pelo menos duas delas tero aniversrios no mesmo ms. No entanto, o princpio da casa dos pombos se presta a aplicaes mais interessantes e significativas do que essa; de outra maneira, no valeria a pena apresent-lo.

Exemplo 2
Escolha 101 nmeros quaisquer dentre os inteiros 1, 2,...,200. Mostre que entre os nmeros escolhidos h dois nmeros tais que um deles divisvel pelo outro. Soluo Em primeiro lugar, observe que qualquer inteiro n se escreve sob a forma n = 2k b, sendo k um inteiro no negativo, e b um inteiro mpar. Por exemplo, 36 = 22 9; 25 = 20 25; 16 = 24 1. Assim, se n pertence ao conjunto {1, 2,..., 200}, n = 2k b e b um dos nmeros mpares 1, 3, 5,..., 199. Ora, h 100 nmeros mpares no conjunto {1, 2,..., 200}. Logo, quando escolhemos 101 nmeros desse conjunto, dois deles tero suas partes mpares iguais, pelo princpio da casa dos pombos; sejam n1 e n2 esses nmeros. Ento, n1 = 2rb e n2 = 2sb. Se r < s, ento n1 divide n2, pois

n2 2 s b = = 2 s r . Se s < r, ento n1 2 r b

n2 dividir n1, o que conclui a demonstrao.

Exemplo 3
Mostre que em um conjunto de n (n > 2) pessoas h duas pessoas que conhecem exatamente o mesmo nmero de pessoas do conjunto (obs.: se A conhece B, B conhece A, ou seja, conhecer uma relao simtrica). Soluo Observe, em primeiro lugar, que qualquer das pessoas do conjunto conhece no mnimo zero e no mximo (n 1) das outras pessoas.
78

Seja P = {A1, A2, ..., An} o conjunto das n pessoas. Dividiremos a demonstrao em dois casos. 1o caso Todas as pessoas conhecem pelo menos uma outra pessoa do conjunto. Nesse caso, podemos colocar as pessoas em n 1 gavetas como segue: 1a gaveta: pessoas de P que conhecem exatamente uma outra pessoa do conjunto P. 2a gaveta: pessoas de P que conhecem exatamente duas outras pessoas do conjunto P.

M
(n 1) gaveta: pessoas de P que conhecem exatamente outras (n 1) pessoas do conjunto P.
a

Temos ento n pessoas a serem distribudas por (n 1) gavetas, e o problema est resolvido, pois, pelo princpio da casa dos pombos, duas das pessoas ocuparo a mesma gaveta. 2o caso Uma das pessoas, que chamaremos de A1, conhece zero pessoa (ou seja, no conhece ningum do conjunto). Nesse caso, nenhuma pessoa de P conhece A1 . Portanto, ningum conhece mais do que outras n 2 pessoas e novamente podemos colocar as n pessoas em (n 1) gavetas como segue: 1a gaveta: pessoas de P que conhecem zero pessoa do conjunto P. 2a gaveta: pessoas de P que conhecem exatamente uma outra pessoa do conjunto P.

M
(n 1) gaveta: pessoas de P que conhecem exatamente outras (n 2) pessoas do conjunto P.
a

Novamente, pelo princpio da casa dos pombos, duas das pessoas

ocuparo a mesma gaveta. O princpio da casa dos pombos pode ser reformulado da seguinte forma.
79

Teorema Se m pombos ocupam n casas, ento pelo menos uma casa contm

m 1 n + 1 pombos ([x] o maior inteiro menor do que ou igual a x). m 1 Demonstrao: Se cada casa contiver, no mximo, pombos, ento n m 1 n(m 1) m 1 < m , uma o nmero mximo de pombos ser n n n contradio, j que temos m pombos.
Ainda outra formulao possvel para o princpio da casa dos pombos a seguinte:

Teorema
Sejam n gavetas e r um inteiro positivo dado. Coloquemos a1 objetos na primeira gaveta, a2 objetos na segunda, e assim sucessivamente, at an objetos na n-sima gaveta. Ento, se a mdia (a1 + a2 + ... + an )/n for maior do que r 1, uma das n gavetas conter pelo menos r objetos. Demonstrao: A demonstrao bem simples. Se todos os ai forem menores do que r, ento a1 < r 1; a2 < r 1; ...; an < r 1. Logo, a1 + a2 + ... + an < nr n = n(r 1), que implica (a1 + a2 + ... + an )/n < r 1, o que uma contradio. Observao O teorema anterior pode ser apresentado sem nenhuma referncia a objetos e gavetas, mas to-somente como uma propriedade simples da mdia: se a mdia dos nmeros naturais a1, a2, ..., an for maior do que r 1, ento um deles dever ser maior do que ou igual a r. O princpio da casa dos pombos pode ser deduzido desse ltimo teorema. Com efeito, se tivermos n objetos para distribuir entre (n 1) gavetas, ento a mdia n /(n 1) certamente ser maior do que 1. Logo, fazendo r = 2, teremos que uma das gavetas deve conter pelo menos 2 objetos.

80

O teorema ainda pode ser usado para demonstrar o seguinte resultado, que pode parecer surpreendente primeira vista. Exemplo 4 So dados dois discos, A e B, cada um deles dividido em 200 setores iguais. Os setores dos discos so pintados de branco ou de preto. Sabe-se que no disco A h 100 setores brancos e 100 pretos, em ordem desconhecida. O nmero de setores brancos de B arbitrrio e desconhecido. Coloquemos o disco A sobre o disco B de modo que cada setor de A fique exatamente sobre um setor de B (sempre que dissermos que o disco A foi colocado sobre o disco B, fica convencionado que h essa coincidncia de setores). Mostre que possvel escolher a posio de A de maneira que existam pelo menos 100 setores de A que tenham a mesma cor que os correspondentes setores de B. Soluo Coloque A sobre B. Seja n1 o nmero de setores sobrepostos com cores coincidentes. Mantendo B fixo, gire A de um ngulo igual a um setor no sentido dos ponteiros do relgio. Seja ento n2 o nmero de setores sobrepostos coincidentes. Continue com o processo, girando A sempre de um setor no sentido dos ponteiros dos relgios e obtendo n3, n4, ..., n200. ento verdade que o nmero total de coincidncias n1 + n2 +... + n200 = (200 100) = 2 (l00)2. Com efeito, fixado um setor do disco B (preto, por exemplo), como o disco A tem exatamente 100 setores pretos, haver 100 posies em que esse setor de B ter a mesma cor que o setor correspondente de A. Assim, o nmero total de coincidncias ser o nmero de setores de B (200) vezes 100 (o nmero de setores vezes o nmero de coincidncias por setor). Ento, pelo teorema, temos (n1 + n2 +... + n200 )/ 200 = 100 > 100 1 (neste caso, r = 100).
81

Logo, pelo menos um dos ni deve ser maior ou igual a 100, ou seja, para uma das posies o nmero de coincidncias de pelo menos 100. Esperamos que os exemplos apresentados tenham dado uma idia de como aplicar o princpio da casa dos pombos. Como Matemtica s se aprende fazendo, propomos a seguir alguns exerccios sobre o assunto. Se possvel, tente generalizar os enunciados e demonstrar suas generalizaes. Exerccios 1. Mostre que, se do conjunto {1, 2,..., 2n} retirarmos (n + 1) nmeros ao acaso, ento: a) um deles dividir um outro. b) dois dos nmeros sero primos entre si. 2. Escolha 5 pontos ao acaso sobre a superfcie de um quadrado de lado 2. Mostre que pelo menos um dos segmentos que eles determinam tem comprimento menor do que ou igual a 2 . 3. Em uma gaveta, h 12 meias brancas e 12 meias pretas. Quantas meias devemos retirar, ao acaso, para termos certeza de obtermos um par de meias da mesma cor? 4. Chame um ponto B = (x, y, z) de R3 de bom se todas as suas trs coordenadas forem inteiras. Considere nove pontos bons de R3. Mostre que o ponto mdio de algum dos segmentos que ligam esses pontos bom. 5. Seja x um nmero real e n um inteiro positivo. Mostre que, entre os nmeros x, 2x, 3x, ..., (n 1)x, existe um cuja distncia a algum inteiro , no mximo, 1/n.
Baseado no artigo Princpio da casa dos pombos Joo Bosco Pitombeira, RPM 08

82

Probabilidade geomtrica:
os problemas dos ladrilhos, do encontro e do macarro

Conde de Buffon, os ladrilhos e as agulhas Georges Louis Leclerc, Conde de Buffon, nasceu em 7 de setembro de 1707, em Montbard, na Frana, e morreu em 16 de abril de 1788, em Paris. Nascido na aristocracia, estudou Medicina e Direito. Mostrou interesse pela Matemtica, tendo descoberto sozinho a Frmula do Binmio e mantido correspondncia com Cramer sobre Mecnica, Geometria, Probabilidade, Teoria dos Nmeros e Clculo Diferencial e Integral. Mas era a Natureza a sua paixo. Dedicou-se principalmente Histria Natural, tendo sido o maior responsvel pelo crescimento do interesse pela Histria Natural na Europa, no sculo XVIII. No 4o volume do seu Suplemento Histria Natural, publicado em 1777, tem 3 de suas 35 sees dedicadas ao Clculo de Probabilidades. Uma delas Sur le jeu de franc-carreau (Sobre o jogo do ladrilho), na qual Buffon discute o jogo do ladrilho e apresenta o Problema da Agulha, que no discutiremos aqui, uma vez que sua soluo exige tcnicas de integrao (pode ser encontrado na RPM 20). Foi o primeiro escrito sobre o que hoje se conhece por Probabilidade Geomtrica: problemas de probabilidades que tm espaos amostrais equivalentes a pontos representados por figuras geomtricas. A probabilidade de um determinado evento pode ser calculada pela razo entre medidas geomtricas como comprimento, rea ou volume.
83

O jogo do ladrilho Era bastante jogado pelas crianas francesas no sculo XVIII. Uma pequena moeda de raio R lanada ao acaso em um cho coberto por ladrilhos quadrados de lado l (l > 2r). As crianas apostavam que a moeda cairia inteiramente dentro de um ladrilho. Buffon notou que a probabilidade de a moeda cair inteiramente dentro de um ladrilho era a probabilidade de o centro da moeda cair dentro de um quadrado de lado l 2r.
l-2r

r
favorvel no favorvel

Essa probabilidade a razo entre as reas do quadrado e do ladrilho, pois a probabilidade de o centro da moeda cair em uma regio proporcional rea dessa regio. Portanto, a probabilidade de a moeda cair inteiramente dentro de um ladrilho

( l 2r ) 2
2 l

Um exemplo atual: considerando um piso formado por quadrados de Paviflex de 30 cm de lado e um disco de raio 6 cm, a probabilidade de o disco cair inteiramente dentro de um dos ladrilhos igual a

(30 12) 2 30
2

324 = 0, 36 ou 36%. 900


(30 d ) 2 302

Poderamos tambm perguntar, nessa situao, qual o dimetro d do disco que daria 60% de chances de vitria ao jogador: que implica d = 6,77 cm.
= 0, 60 , o

84

O problema do encontro Duas pessoas decidiram se encontrar em um determinado local entre 11 e 12 horas. Combinou-se previamente que a primeira pessoa a chegar esperar no mximo 15 minutos pela outra. Ache a probabilidade P de o encontro acontecer, admitindo que cada uma das pessoas pode chegar, de modo eqiprovvel, em qualquer instante entre 11 e 12 horas. Podemos associar os instantes de chegada das duas pessoas, no intervalo de 60 min, entre 11 e 12 horas, a um par (x,y) de [0, 60] x [0, 60] representados por pontos em eixos ortogonais x e y em R2. Cada ponto teria coordenadas x, y numericamente iguais quantidade de minutos dos respectivos instantes de chegada, 11h e x min, 11h e y min , das duas pessoas. De acordo com o enunciado, o encontro somente ter lugar se y x 15, ou seja, y x + 15 e y x 15. Essas duas inequaes definem a regio em cinza da figura.
y y = x + 15 y = x - 15

Logo, se A a rea da regio cinza, temos P = 602/A. A = 602 2(45 x 45)/2 = 602 452 = 105 x 15 = 1 575 P = 1 575/3600 = 0,4375 ou 43,75%.

60

15 15 60
x

O problema do macarro Em uma sala de aula distribuiu-se um espaguete para cada aluno, pedindo a cada um que partisse o espaguete, ao acaso, em trs pedaos. Em seguida, pediu-se que cada um verificasse se era possvel formar um tringulo com os seus trs pedaos. Colocou-se a pergunta: supondo que todas as possveis divises ocorram de forma eqiprovvel, qual a probabilidade de se obter um tringulo?

85

O problema pode ser enunciado do seguinte modo: Dividindo-se aleatoriamente um segmento em trs partes, qual a probabilidade de que esses novos segmentos formem um tringulo? Tomemos um segmento de reta AB de comprimento 1. Vamos dividi-lo em trs partes: uma de comprimento x, outra de comprimento y e a terceira, naturalmente, de comprimento 1 x y.
A x y 1-x-y
y

Cada forma de dividir o segmento unitrio fica ento associada ao par ordenado (x, y) com x > 0, y > 0 e x + y < 1. Isso corresponde no plano cartesiano regio triangular da figura. Portanto, cada forma de dividir um segmento em trs partes est agora representada por um ponto interior ao tringulo da figura.

Entretanto, no so todas as divises que formam tringulos. Um tringulo existe se, e somente se, cada lado for menor que a soma dos outros dois. Isso equivalente a dizer que, em um tringulo, cada lado menor que o seu semipermetro, que no nosso caso igual a 1/2. Temos, portanto, x < 1/2, y < 1/2 e 1 x y < 1/2 ou x + y > 1/2. Logo, a regio favorvel o interior do tringulo formado pelos pontos mdios dos lados do tringulo inicial, que tem rea igual a 1/4 da rea do tringulo grande, o que nos leva a concluir que a probabilidade de que os trs segmentos formem um tringulo 0,25 ou 25%.
y

1 1
x

Baseado nos artigos Determinao de probabilidades por mtodos geomtricos Nelson Tunala, RPM 20 Probabilidade Geomtrica Eduardo Wagner, RPM 34 O problema do jogo dos discos Roberto R. Paterlini, RPM 48

86

Alguns problemas clssicos sobre grafos

O conceito de grafo simples, porm frtil em aplicaes e problemas atraentes. Ele j foi abordado, nesta Revista, em pelo menos trs ocasies: no nmero 3, quando o Prof. G. de La Penha descreveu o problema das pontes de Knigsberg, no nmero 10 (implicitamente), quando o Prof. J. B. Pitombeira tratou da questo de determinar o nmero de regies em que n retas em posio geral decompem o plano e, no nmero 11, quando este mesmo autor estudou o problema de ligar gua, luz e telefone em trs casas. Creio que nossos leitores apreciaro uma anlise do problema das pontes. E, para aproveitar o embalo, ofereceremos solues diferentes para os outros dois problemas acima mencionados. sempre instrutivo ter diversas alternativas para resolver questes interessantes. As setes pontes de Knigsberg Imaginemos um rio, com duas margens A e B. No rio, duas ilhas C e D. A ilha C est ligada a cada uma das margens por duas pontes. Em cada margem, h tambm uma ponte para a ilha D. A stima ponte liga as ilhas entre si. O problema das sete pontes de Knigsberg consiste em achar um caminho, ao longo do qual um pedestre, partindo de uma das margens ou de qualquer das ilhas, percorra todas as pontes, sem passar mais de uma vez por qualquer uma delas.
87

A D

B
figura 1

Este problema foi resolvido, em 1735, pelo matemtico suo Leonhard Euler. Ele fez a observao fundamental de que, para efeito da questo proposta, as margens e as ilhas so como A se fossem pontos A, B, C, D. As pontes so como arcos que tm esses pontos como 1 extremidades. Tudo se resume a analisar o 2 3 diagrama ao lado, onde os arcos ligam os 1 C D pontos, de acordo com a disposio das 4 pontes dada no enunciado do problema. 5
1

O desenho da figura 2, provavelmente, B o primeiro exemplo de um grafo a ocorrer figura 2 como modelo matemtico para resolver um problema, que agora se exprime assim: partindo de um dos vrtices A, B, C, ou D, achar um caminho que percorra todo o grafo sem passar mais de uma vez pelo mesmo arco. De um modo geral, um grafo isto: um conjunto finito de pontos, chamados os vrtices do grafo, e um conjunto finito de arcos, chamados as arestas do grafo. As extremidades de cada aresta devem ser vrtices. Alm disso, duas arestas quaisquer do grafo no podem ter pontos interiores em comum: ou so disjuntas ou se tocam apenas numa ou em duas das extremidades. Euler chamou ateno para uma noo muito simples, porm crucial, que a ordem de um vrtice do grafo. A ordem de um vrtice o nmero de arcos que emanam dele. Por exemplo, no grafo das pontes de Knigsberg, o vrtice C tem ordem 5, enquanto os demais vrtices A, B e D tm todos ordem 3.

88

Um caminho num grafo uma seqncia finita de vrtices = (A0, A1, ..., Ap) tal que, para cada i = 1, ..., p, Ai1 e Ai so extremidades de uma aresta (juntamente com a escolha da aresta ligando Ai1 e Ai, j que pode haver mais de uma aresta). Diz-se que o caminho parte do vrtice A 0, percorre as arestas escolhidas e termina no vrtice Ap. Um caminho chama-se unicursal quando no percorre a mesma aresta mais de uma vez. Um grafo G chama-se unicursal quando existe um caminho unicursal que percorre todas as arestas de G. Observe-se que um caminho unicursal pode passar vrias vezes pelo mesmo vrtice. Toda vez que um caminho unicursal chegar a um vrtice, deve sair dele por um arco diferente daquele por onde chegou. (A menos que esse vrtice seja o fim do caminho.) Portanto, se um caminho unicursal percorrer todas as arestas do grafo, os vrtices desse grafo, com exceo do incio e do fim do caminho, devem ter todos um nmero par de arestas emanando deles, isto , devem ter ordem par. O vrtice que serviu de incio e o que serviu de fim para o caminho tm ordem mpar. Se o incio e o fim do caminho coincidirem (isto , se o caminho for fechado), ento todos os vrtices do grafo, sem exceo, tm ordem par. Conclumos ento que se um grafo unicursal, ou todos os seus vrtices tm ordem par ou exatamente dois vrtices tm ordem mpar. No primeiro caso, todo caminho unicursal fechado. No segundo caso, um caminho unicursal deve comear num dos vrtices de ordem mpar e terminar no outro. Segue-se, da, que o grafo da figura 2 no unicursal, pois seus quatro vrtices tm todos ordem mpar. Fica, ento, resolvido o problemas das sete pontes: impossvel percorr-las todas, sem passar duas vezes por alguma ponte.
Observao: A cidade de Knigsberg ficava na Prssia, regio do leste da Alemanha. Hoje, ela se chama Kaliningrado, pertence Rssia e j possvel percorrer todas as suas pontes sem passar mais de uma vez por cada uma delas. que foi construda uma nova ponte. A bem da verdade, devemos esclarecer tambm que, de fato, Euler no menciona a ilha D. No seu mapa h uma pennsula D, a partir da qual o rio Pregel se bifurca, depois de passar pela ilha C (que se chama Kneiphof). Mas claro que o problema fica bem mais fcil de enunciar se substituirmos a pennsula por uma ilha, o que no faz diferena alguma do nosso ponto de vista.
89

Voltando s antigas pontes de Knigsberg, podemos trocar o ponto de vista terrestre pelo aqutico e indagar: seria possvel a um barqueiro (ou nadador) no rio passar por baixo das sete pontes sem passar mais de uma vez sob nenhuma delas? Esta questo, ao que parece, nunca foi considerada por Euler. O leitor interessado pode, entretanto, tomar seu lpis e papel. Se tiver um pouco de pacincia vai conseguir uma rota adequada para o barqueiro, como por exemplo a da figura 3.

A D

B
figura 3

Tendo sido bem-sucedido em sua tentativa, o leitor pode indagar se foi apenas uma questo de sorte ou se existe uma razo matemtica que permita ao barqueiro cruzar as pontes, do mesmo modo que probe o pedestre de percorr-las. Seria possvel reformular este segundo problema em termos de grafos, como fizemos com o primeiro? Existe sim, a razo matemtica. , sim, possvel enquadrar o barqueiro no contexto dos grafos. Vejamos como. Um grafo no plano divide esse plano em regies. Por exemplo, o grafo da figura 2 determina cinco regies. A regio exterior, que naquela figura indicamos com o algarismo 1, e mais quatro regies limitadas, as quais indicamos com os algarismos 2, 3, 4 e 5 na figura. Usando essas regies, pode-se, a partir de um grafo G, construir um novo grafo G*, chamado o dual de G. Os vrtices de G* so tantos quantas so as regies de G. Dois vrtices do novo grafo G* estaro ligados por tantas arestas quantas forem as arestas adjacentes s regies correspondentes. Por exemplo, seja G o grafo da figura 2. Para formar o grafo dual G* tomamos cinco vrtices, correspondentes s cinco regies 1, 2, 3, 4 e 5. A regio 1, no grafo G, adjacente a todas as outras. Logo devemos traar
90

arestas em G* ligando o vrtice 1 a todos os outros quatro. As regies, 2 e 3, 3 e 4, 4 e 5 so adjacentes. Ento devem existir arestas em G* ligando os vrtices com esses nmeros. Por outro lado, no h outros pares de regies adjacentes. Logo no h outras arestas em G*. O grafo G*, dual daquele na figura 2, est desenhado na figura 4.
3 4

1
figura 4

Note-se que, no grafo G*, apenas dois vrtices (3 e 4) tm ordem mpar (ambos tm ordem 3). Os vrtices 2 e 5 tm ordem 2 e o vrtice 1 tem ordem 4. Portanto G* cumpre a condio necessria para ser unicursal. Pode-se demonstrar que essa condio tambm suficiente para um grafo conexo. Mais ainda, um caminho unicursal no grafo G* deve comear num dos vrtices 3 ou 4 e terminar no outro. Isso justifica matematicamente por que um barqueiro pode passar por baixo das sete pontes de Knigsberg sem repetir nenhuma delas, mas um pedestre no pode fazer seu passeio unicursal ao longo dessas pontes. que o grafo G no unicursal, enquanto seu dual G* . Alm disso, o percurso do barqueiro deve comear ao lado da ponte que liga as duas ilhas e terminar do outro lado dessa mesma ponte. De um modo geral, juntamente com o problema de percorrer todas as arestas de um grafo plano, pode-se sempre considerar o problema dual de, partindo de uma das regies por ele determinadas, descrever um caminho que corte todas as arestas uma nica vez. Isto corresponde a indagar se o grafo dual unicursal. O leitor convidado a desenhar diferentes grafos e examinar, para cada um deles, a possibilidade de traar um caminho unicursal, no grafo ou no seu dual.

91

Em quantas regies n retas dividem o plano? A pergunta formulada acima no admite uma resposta nica. Com 3 retas distintas, por exemplo, podemos dividir o plano em 4, 6 ou 7 regies, conforme se v na figura 5.

1 6

2 5

3 4

1 5

2 2 3 4 6 1 7 5 3 4

4 regies

6 regies

6 regies
figura 5

7 regies

A formulao correta do problema, para que ele tenha uma resposta nica, a seguinte: qual o nmero mximo de regies em que n retas dividem o plano? Evidentemente, o nmero mximo de regies ocorre quando essas retas esto situadas de modo a terem o nmero mximo possvel de pontos de interseco. Esse nmero mximo acontece quando: 1o) Entre as retas dadas no h paralelas; 2o) Nenhum ponto a interseco de mais de duas retas dadas. Neste caso, diz-se que as n retas dadas esto em posio geral. Dadas n retas em posio geral, para determinar o nmero R de regies em que elas dividem o plano, procederemos da seguinte maneira. Em primeiro lugar, traamos um crculo to grande que contenha em seu interior todos os pontos de interseco das n retas. Os requisitos 1o) e 2o) acima asseguram que, para cada duas das n retas dadas, h um ponto de interseco e vice-versa. Logo, o nmero dos pontos de interseco, todos

n situados no interior do nosso crculo, = n(n 1) / 2. 2


Na figura 6, temos quatro retas em posio geral. Seus 6 pontos de interseco esto no interior do crculo ali traado. Agora consideremos o grafo plano G, obtido quando desprezamos as partes das retas que ficam no exterior do crculo que traamos.
92

figura 6

Os vrtices de G so as interseces das n retas duas a duas e mais os 2n pontos em que essas n retas intersectam a circunferncia: ao todo, temos V = 2n + n(n l)/2 vrtices no grafo G. As arestas de G so os 2n arcos de crculo correspondentes e mais os segmentos de reta interiores ao crculo. Sobre cada uma das n retas h n + 1 vrtices, a saber: os n 1 pontos de interseco dessa reta com as n 1 outras e os 2 pontos em que ela corta a circunferncia. Logo, temos n segmentos, isto , n arestas do grafo G, sobre cada uma das n retas dadas. Ao todo, so n2 arestas de G interiores ao crculo, com o total de A = n2 + 2n arestas em G. O nmero R de regies em que as n retas dadas dividem o plano igual ao nmero de regies determinadas pelo grafo G menos uma, que a regio exterior ao crculo. A frmula de Euler diz que se um grafo com V vrtices e A arestas decompe o plano em F regies, tem-se V A + F = 2. Pela frmula de Euler temos, portanto, V A + R = 1, ou seja, 2n + n(n 1)/2 n 2 2n + R = 1, donde R = 1 + n(n +1)/2. Equivalentemente: R = 1 + n + gua, luz e telefone Um problema muito popular, desde meus tempos de ginsio, consiste em propor que se ligue, em trs casas, gua, luz e telefone, a partir de 3 centrais diferentes. Casas, centrais, e ligaes esto no mesmo plano. No se permite que as ligaes se cruzem.
n(n 1) n n n = + + . 2 0 1 2

93

No possvel fazer isto. Uma demonstrao dessa impossibilidade foi apresentada no nmero 11 da RPM, usando a frmula de Euler. No que se segue, daremos uma demonstrao diferente do mesmo resultado, sem fazer uso daquela frmula. Representaremos as centrais de gua, luz e telefone pelas letras A, L, T e as trs casas por pontos X, Y e Z. Comecemos ligando gua e luz s casas X e Y. Obteremos um quadriltero XAYL, cujos lados podem ser curvilneos. A central telefnica T pode estar dentro ou fora deste quadriltero. Isto no far diferena alguma mas, para fixar as idias, suponhamos que esteja fora, como na figura 7. T
X 3 A 1 2 L
figura 7

Y 3

Liguemos o telefone nas casas X e Y. Ficamos com dois quadrilteros adjacentes XAYL e XLYT, os quais decompem o plano em trs regies, que designamos por 1, 2 e 3. (As regies 1 e 2 so interiores aos quadrilteros, enquanto a regio 3 exterior.) A terceira casa, Z, dever estar numa dessas trs regies. Examinemos cada uma das possibilidades. Se Z estiver na regio 1, poderemos ligarlhe gua e luz porm no telefone. Se estiver na regio 2, ficar com luz e telefone, mas sem gua. Finalmente, se Z estiver na regio 3, poder ter gua e telefone, mas no ter luz. Portanto, as nove ligaes no podem ser todas feitas sem que se cruzem, e o problema est resolvido.
Baseado no artigo Alguns problemas clssicos sobre grafos Elon Lages Lima, RPM 12

94

Srie harmnica

Introduo O objetivo deste artigo o de fazer uma apresentao simples da chamada srie harmnica, que possui propriedades muito interessantes. Um pouco de Histria As sries infinitas so conhecidas desde a antiguidade, e a primeira a ocorrer na Histria da Matemtica uma srie geomtrica de razo 1/4, que intervm no clculo da rea da parbola feito por Arquimedes. Depois da ocorrncia de uma srie geomtrica num trabalho de Arquimedes, as sries infinitas s voltaram a aparecer na Matemtica cerca de 1500 anos mais tarde, no sculo XIV. Nessa poca havia um grupo de matemticos na Universidade de Oxford que estudava a cinemtica, ou fenmeno do movimento; e, ao que parece, foi esse estudo que levou reconsiderao das sries infinitas. Ao lado dos pesquisadores de Oxford, havia tambm pesquisadores em outros centros. Na Universidade de Paris, em particular, havia um professor chamado Nicole Oresme (1325-1382), um destacado intelectual em vrios ramos do conhecimento, como Filosofia, Matemtica, Astronomia, Cincias Fsicas e Naturais. Alm de professor universitrio, Oresme era conselheiro do rei, principalmente na rea de finanas pblicas; e nessa funo revelou-se um homem de larga viso,
95

recomendando medidas monetrias que tiveram grande sucesso na prtica. Ao lado de tudo isso, Oresme foi tambm bispo de Lisieux. Um dos trabalhos mais notveis de Oresme sobre as sries infinitas est ligado srie harmnica. Antes, porm, de falar da srie harmnica, temos de explicar o que significa dizer que uma srie convergente ou divergente. A idia de srie infinita aparece na Matemtica quando imaginamos a operao de somar parcelas sucessivamente sem que essa operao termine aps um nmero finito de parcelas somadas. Deixando de lado qualquer preocupao com a rigorizao desse conceito, vamos examinar algumas sries infinitas simples. Por exemplo,
1+ 1 1 1 1 1 1 + + + + + +K 2 4 8 16 32 64 1 e a soma 2

Trata-se de uma progresso geomtrica infinita de razo de seus termos dada por S =
1

= 2. 1 2 Sries que tm soma finita so chamadas de sries convergentes. Mas fcil imaginar sries que no sejam convergentes. Por exemplo, claro que as sries 1

1 + 2 + 3 + 4 + 5 + ... ,

2 + 4 + 6 + 8 + . . .,

1 + 2 + 1 + 2 + 1 + 2...,

no so convergentes; elas so ditas divergentes. Um exemplo menos

1 2 3 4 n + + + +K + + K . Para 2 3 4 5 n +1 ver que essa srie diverge, basta notar que todos os seus termos, a partir do segundo, so maiores do que 1/2.
trivial de srie divergente dado por A srie harmnica A srie harmnica uma srie muito simples, dada por

n =1 + 2 + 3 + 4 + 5 + K
n =1 96

Como se v, os termos da srie harmnica esto decrescendo para zero. Mas ser que, quando o termo geral de uma srie tende a zero, ela converge? Se for assim e primeira vista parece que , ento a srie harmnica deve ser convergente. Vamos investigar. Aps a soma de um grande nmero de termos da srie harmnica, quando chegarmos a n = 1020, n= 1030, n = 10100, etc., estaremos somando to pouco que teremos a impresso de que a soma de todos os termos da srie infinita realmente um nmero finito. Alis, hoje em dia, com a ajuda do computador, podemos at fazer clculos experimentais interessantes. Vamos supor que fssemos capazes de somar cada termo da srie em um segundo de tempo. Como um ano tem aproximadamente 365,25 x 24 x 60 x 60 = 31 557 600 segundos, nesse perodo de tempo seramos capazes de somar a srie at n = 31 557 600, obtendo para a soma um valor pouco superior a 17; em 10 anos a soma chegaria a pouco mais de 20; em 100 anos, a pouco mais de 22. Como se v, somas parciais de termos da srie harmnica jamais nos levariam a suspeitar que ela diverge. Pelo contrrio, essas somas s nos levam a pensar que a srie seja convergente. Isso, todavia, falso! Embora surpreendente, esse resultado pode ser facilmente demonstrado. Para isso agrupamos os termos da srie assim:
1 1 1 1 + + + +K 2 3 4 5 1 1 1 1 1 1 1 1 1 1 =1+ + ( + ) + ( + + + ) + ( + +K ) +K 2 3 4 5 6 7 8 9 10 16 1+

Observe agora que a soma dentro de cada parntese sempre maior do que 1/2. Veja:
1 1 1 1 1 + > + = ; 3 4 4 4 2 1 1 1 1 1 1 1 1 1 + + + > + + + = ; 5 6 7 8 8 8 8 8 2 1 1 1 1 1 1 1 + +K + > + +K + = , 16 2 9 10 16 16 16

e assim por diante.


97

Ento,

n =1 + 2 + 3 + 4 + 5 + K > 1 + 2 + 2 + 2 + K = ,
n =1

o que

prova que a srie mesmo divergente. A demonstrao de que a srie harmnica diverge, feita pela primeira vez por Oresme, mostra como decisivo o papel do raciocnio lgico para estabelecer uma verdade que jamais seria descoberta de outra maneira. De fato, como vimos acima, mesmo somando os termos da srie durante um sculo (se isso fosse possvel), no chegaramos a um resultado que nos desse qualquer indcio de que a srie seria divergente... Para terminar, vamos fazer mais um exerccio de imaginao. Hoje em dia temos computadores muito rpidos, e a tecnologia est produzindo mquinas cada vez mais rpidas. Mas isso tem um limite, pois, como sabemos, nenhum sinal fsico pode ser transmitido com velocidade superior da luz. Portanto, nenhum computador poder efetuar uma soma em tempo inferior a 10-23 segundos, que o tempo gasto pela luz para percorrer uma distncia igual ao dimetro de um eltron. Pois bem, com tal computador, em um ano, mil anos e um bilho de anos, respectivamente, o nmero de termos que poderamos somar seria 315576 x 1025, 315576 x 1028 e 315576 x 1034. Veja os resultados aproximados que obteramos para a soma da srie harmnica, em cada um desses casos, respectivamente: 70,804; 77,718 e 91,5273 . Imagine, finalmente, que esse computador estivesse ligado desde a origem do universo, h 16 bilhes de anos. Ele estaria hoje obtendo o valor aproximado de 94,2999 para a soma da srie harmnica, um nmero ainda muito pequeno... O leitor tem toda razo em perguntar: Como se chega ao nmero 94,2999, se o (idealizado) computador mais rpido que se possa construir deveria ficar ligado durante 16 bilhes de anos? Sim, no h como fazer essa soma diretamente, mas existem mtodos que permitem substituir a soma Sn dos n primeiros termos da srie por uma expresso matemtica que aproxima Sn e que pode ser calculada numericamente, o que, no entanto, requer conhecimentos de Clculo Integral.

98

Alergia pelo nmero 7


Imaginem um matemtico alrgico ao nmero 7 que decidisse eliminar da srie harmnica todas as fraes que contivessem o algarismo 7. A nova srie ficaria assim:

1 1 1 1 1 1 1 1 1 1 1 1 1 + + + + + + + + +K + + + + +K 1 2 3 4 5 6 8 9 10 15 16 18 19
Como todos os demais algarismos podero ser usados, salvo, to-somente, o 7, era de se esperar que a nova srie tambm divergisse. Mas, vejam s, a srie acima converge e a sua soma no chega a oitenta! S provando para acreditar: * Cada uma das 8 primeiras fraes, de 1 at 1 , menor ou igual a 1. A 1 9 soma dessas fraes menor do que 8.

1 , menor ou igual * Cada uma das 8 x 9 fraes seguintes, de 1 at 99 10 1 a 1 . A soma dessas fraes menor do que 8 9 . 10 10
* Cada uma das 8 x 9 x 9 fraes seguintes, de ou igual a

1 1 at , menor 100 999

1 1 . A soma dessas fraes menor do que 8 92 2 . 100 10

E assim, sucessivamente, a soma dos termos da srie ser menor do que 8 + 8 9 8 92 8 93 8 + + +K + = 80. 2 3 9 10 10 10 1 10
Baseado nos artigos As sries infinitas Geraldo vila, RPM 30 Alergia pelo nmero 7 Renate Watanabe, RPM 31
99

No incrvel?

O que tem mais: racionais ou naturais?

Cantor e a Teoria dos Conjuntos A reforma do ensino da Matemtica de 50 anos atrs introduziu a utilizao de conjuntos no ensino bsico, mas apenas a parte referente notao e linguagem de conjuntos, nada de substancial sobre a verdadeira Teoria dos Conjuntos. Em conseqncia, no apenas os alunos, mas tambm muitos professores so pouco informados sobre a importncia desse ramo de estudos, da a razo de tratarmos aqui de alguns poucos aspectos interessantes dessa disciplina. O criador da Teoria dos Conjuntos foi o matemtico alemo Georg Cantor (1845-1918), que foi professor na Universidade de Halle, onde iniciou uma srie de pesquisas sobre as chamadas sries trigonomtricas. Essas sries ocuparam a ateno dos mais eminentes matemticos durante todo o sculo XIX; e seu estudo, pelos muitos desdobramentos e ramificaes que teve, foi, em verdade, o impulso mais significativo para o progresso da Anlise Matemtica durante a maior parte do sculo. Atravs de suas investigaes nesse domnio, Cantor foi levado a estudar os conjuntos de pontos de descontinuidade das funes que considerava, logo chegando a estudar conjuntos infinitos de pontos de descontinuidade. Da ele passou naturalmente a estudar conjuntos em si, sem referncia a funes. Assim nascia a Teoria dos Conjuntos.

100

Conjuntos enumerveis Um dos primeiros fatos surpreendentes que surgem na considerao de conjuntos infinitos diz respeito possibilidade de haver uma equivalncia entre um conjunto e um seu subconjunto prprio. Isso pode ser visto facilmente atravs da seguinte correspondncia (restrita a nmeros positivos, por simplicidade):
1 2 3 4 5 6 7 8 9 10 K b b b b b b b b b b 2 4 6 8 10 12 14 16 18 20 K

Existe aqui uma correspondncia biunvoca entre elementos dos dois conjuntos (n 2n) de tal sorte que a cada elemento de cada conjunto corresponde um nico elemento do outro. Segundo Cantor, dois conjuntos so equivalentes, ou tm a mesma cardinalidade, quando possvel estabelecer entre eles uma tal correspondncia. No caso de conjuntos finitos, serem equivalentes corresponde a terem o mesmo nmero de elementos, de sorte que o conceito de equivalncia ou cardinalidade uma extenso, a conjuntos infinitos, da noo de nmero de elementos de um conjunto. Cantor passou a chamar de enumervel a todo conjunto que tem a mesma cardinalidade do conjunto dos nmeros naturais 1, 2, 3, 4, ...Vamos mostrar que os nmeros racionais tambm formam um conjunto enumervel. Por simplicidade restringimo-nos aos racionais positivos, que distribumos em vrios grupos, cada grupo contendo as fraes cujos numerador e denominador tm a mesma soma; por exemplo,

1 2 3 4 , , , o grupo de 4 3 4 1 todas as fraes cujos termos tm soma 5. Vamos fazer uma lista de todos esses grupos, comeando com aquele cuja soma dos termos das fraes 2 (e que s contm a frao 1/1); depois o grupo das fraes 1/2 e 2/1, cuja soma dos termos 3; e assim por diante, sucessivamente. Ao mesmo tempo, riscamos as fraes que representam o mesmo nmero j representado por fraes que apareceram antes. Eis a lista: 1 ; 1 1 2 3 4 , , , ; 4 3 2 1 1 2 , ; 2 1 1 2 3 , , 3 2 1 1 2 3 4 5 , , , , ; 5 4 3 2 1
101

1 2 3 4 5 6 , , , , , ; 6 5 4 3 2 1 1 2 3 4 5 6 7 , , , , , , ; 7 6 5 4 3 2 1 1 2 3 4 5 6 7 8 , , , , , , , ; 8 7 6 5 4 3 2 1
claro que esse procedimento resulta numa lista de todos os nmeros racionais. Basta agora enumer-los na ordem em que aparecem, isto ,

r1 = 1, r5 = 3, r9 = 4,

r2 = 1 / 2, r3 = 2, r6 = 1 / 4, r7 = 2 / 3, r10 = 1 / 5, r11 = 5,

r4 = 1 / 3, r8 = 3 / 2, r12 = 1 / 6, etc.

Dessa maneira obtemos uma correspondncia biunvoca entre o conjunto dos nmeros racionais (positivos) e dos nmeros naturais, que tambm podemos expressar assim:
1 2 3 4 5 6 7 8 9 10 K b b b b b b b b b b 2 1/ 2 2 1/ 3 3 1/ 4 2 / 3 3 / 2 4 1/ 5 K

Isso mostra que os nmeros racionais formam, de fato, um conjunto enumervel. Conjuntos no-enumerveis Se todos os conjuntos infinitos fossem enumerveis, tendo, pois, a mesma cardinalidade, esse conceito no teria utilidade. O primeiro grande mrito de Cantor foi a descoberta de que os nmeros reais no so enumerveis. O leitor interessado encontra a demonstrao desse fato, por exemplo, na RPM 4. Com essa descoberta, Cantor estabeleceu um fato muito surpreendente, qual seja, o de que existem pelo menos dois tipos diferentes de infinito: o do conjunto dos nmeros naturais e o do conjunto dos nmeros reais. Cantor provou outro fato no menos perturbador: o de que, dado um conjunto qualquer, sempre possvel construir outro conjunto maior ainda, isto , cuja cardinalidade maior que a do conjunto dado. Ele obteve assim um modo de construir toda uma infinidade de conjuntos infinitos com cardinalidades diferentes, e ordenou os conjuntos infinitos de acordo com
102

sua cardinalidade, do mesmo modo que se ordenam os conjuntos finitos de acordo com o nmero de seus elementos, ou seja, sua cardinalidade. Surgia assim a teoria dos nmeros transfinitos. Conseqncias do trabalho de Cantor As descobertas de Cantor tiveram grande impacto no mundo matemtico de fins do sculo passado e comeo do presente sculo. Para bem apreciar o que ento acontecia, bom lembrar que desde o incio do sculo XIX era crescente a preocupao com o rigor, primeiro na Anlise Matemtica, porm mais tarde tambm na Geometria, depois das descobertas das geometrias no euclidianas. A partir de 1870, quando Cantor iniciava sua vida profissional, as atividades de pesquisa na rea de axiomatizao e fundamentos intensificavam-se rapidamente. E a Teoria dos Conjuntos, que ento se desenvolvia, revelou-se muito adequada para ser o fundamento de toda a Matemtica. H uma outra razo por que a Teoria dos Conjuntos importante em Matemtica, fora da rea dos fundamentos propriamente dita. que desde os tempos de Cantor muitas disciplinas matemticas novas surgiram e se desenvolveram extensamente, como a Topologia, a lgebra Abstrata, a Teoria da Medida e Integrao, a Teoria da Probabilidade, a Anlise Funcional e outras mais. E em todas essas disciplinas que, ao contrrio de estanques e separadas, no mais das vezes se entrelaam atravs de fronteiras indistinguveis a linguagem, a notao e os resultados da Teoria dos Conjuntos se revelaram instrumento natural de trabalho, a ponto de ser impossvel conceber o desenvolvimento de toda essa Matemtica sem a Teoria dos Conjuntos. Tentando uma analogia, diramos que a Teoria dos Conjuntos aqui to necessria e indispensvel como a notao literal necessria e indispensvel lgebra Elementar. Para concluir queremos deixar claro que a Teoria dos Conjuntos uma disciplina cuja importncia difcil exagerar, no s para a Matemtica, mas para o conhecimento humano de um modo geral. Ela no importante, isto sim, para o ensino bsico da Matemtica, onde somente um pouco de notao e linguagem de conjuntos suficiente.
Baseado no artigo A Teoria dos Conjuntos e o ensino de Matemtica Geraldo vila, RPM 4
103

Problemas
1. Uma calculadora cientfica com diversos circuitos danificados s est fazendo adies, subtraes, multiplicaes, divises e calculando as funes trigonomtricas seno e cosseno e suas inversas. Como podemos obter a raiz quadrada de um nmero positivo com essa calculadora usando um nmero finito de operaes? 2. (O teste da diagonal) Dois retngulos ABCD e ABCD so sobrepostos como na figura. Prove que os retngulos so semelhantes se e s se as diagonais AC e AC esto na mesma reta. Logo, para verificar se dois retngulos so semelhantes, basta coloc-los como na figura e verificar se as diagonais AC e AC esto na mesma reta. C D
D A=A C C B B

3. Sabe-se que o nmero de 7 algarismos 21358ab, em que a o dgito das dezenas e b o das unidades, divisvel por 99. Determine a e b. 4. As medidas dos lados de um tringulo retngulo (numa mesma unidade) podem ser nmeros primos? 5. Um trem atravessa uma ponte de 171 m em 27 segundos. Determine a velocidade e o comprimento do comboio se o tempo de passar um pedestre, que anda em sentido contrrio, com a velocidade de 1 m/s, de 9 segundos. 6. Dados dois espelhos planos paralelos, considere dois pontos A e B entre eles. Determine a trajetria que deve percorrer um raio de luz partindo de A para atingir o ponto B aps ter sido refletido 3 vezes num espelho e 2 vezes no outro. Admite-se que o ngulo de incidncia seja igual ao de reflexo.

104

7. Prove que todo tringulo com duas bissetrizes iguais issceles. 8. Prove que vale a seguinte desigualdade log56 + log67 + log78 + log85 > 4.

9. Sabe-se que cada uma dentre as pessoas A, B e C diz a verdade em


qualquer situao, com probabilidade 1/3. Suponha que A faa uma afirmao e que C diz que B diz que A falou a verdade. Qual a probabilidade de que A realmente tenha falado a verdade? 10. Prove que
cos 2 8 2 14 8 14 3 cos + cos cos + cos cos = . 9 9 9 9 9 9 4

11. a) Dada uma equao do segundo grau, com coeficientes inteiros, mostre que o seu discriminante no pode ser igual a 23. b) Para quantos valores reais do nmero a equao x2 +ax +6a = 0 possui somente razes inteiras? 12. Prove que, se sen(2x + y) = 5 sen y, ento tg ( x + y ) =
3 tg x. 2

13. Pelo ponto mdio M do lado BC de um quadriltero ABCD, traar a reta que divide esse quadriltero em duas partes de reas iguais. 14. Uma urna contm n bolas numeradas de 1 a n. Bolas so retiradas dessa urna sucessivamente, sem reposio, at que pela primeira vez aparea um nmero maior que todos os anteriores. Caso isso no acontea, o processo prossegue at que se esgotem as bolas da urna. Para k = 2, ..., n, determine a probabilidade de que o processo pare na k-sima retirada. 15. Numa circunferncia de raio R fixe dois pontos B e C. Mostre que o lugar geomtrico dos baricentros dos tringulos ABC, onde A um ponto qualquer dessa circunferncia, uma outra circunferncia de raio R/3 que corta BC em trs segmentos congruentes. 16. Determine as solues inteiras e positivas da equao. x3 y3 = 602
(Sugesto: fatore x3 y3 e 602).

105

17. Se dois tringulos tm dois ngulos respectivamente iguais e dois ngulos

respectivamente suplementares, mostre que os lados opostos aos ngulos iguais so proporcionais aos lados opostos aos ngulos suplementares. 18. Considere o conjunto A de todas as combinaes simples de 10 elementos em grupos de 5. Duas combinaes distintas so escolhidas ao acaso no conjunto A. Determine as probabilidades de que elas: a) no tenham nenhum elemento em comum; b) tenham exatamente 4 elementos em comum. 19. Num icosaedro regular de aresta a, cada vrtice est ligado a 5 outros vrtices formando uma pirmide pentagonal. Calcule a altura dessa pirmide. 20. Dados dois pontos A e B do plano e uma constante o lugar geomtrico dos pontos P do plano tais que

m > 0, determinar n PA m = . PB n

21. Encontre todos os nmeros naturais de dois dgitos tais que sua soma com o nmero formado pelos mesmos dgitos em ordem contrria resulta um quadrado perfeito. 22. Considere em um plano as retas paralelas a, b, c distintas duas a duas. Mostre que existem tringulos equilteros cujos vrtices A, B, C so pontos das retas a, b, c respectivamente. 23. Use um argumento combinatrio para determinar o valor de
2 2 2

n n n + +L + 0 1 n sendo n um inteiro maior ou igual a 1.

24. O produto de 3 nmeros pares e consecutivos 88 XXXXX 2, onde cada X representa um algarismo que falta. Determine esses 5 algarismos. 25. Mostre que, quaisquer que sejam os nmeros inteiros a, b, c, d, e, a equao
106

x7 + 2x6 + 3x5 + ax4 +bx3 + cx2 + dx + e = 0 no pode ter todas as razes reais. 26. Supondo que o polinmio P(x) = x100 600x99 + a98x98 + ... + a1x + a0 tenha 100 razes reais e que P(7) > 1, mostre que existe pelo menos uma raiz maior do que 7. 27. Prove que um pentgono com os cinco lados congruentes e trs ngulos congruentes regular. 28. Mostre que, se a, b, c so nmeros inteiros mpares, ento a equao ax2 + bx + c = 0 no tem razes racionais. 29. Suponha que cada ponto de um plano seja pintado de uma cor escolhida entre trs cores dadas. Prove que existem dois pontos de mesma cor cuja distncia k, sendo k > 0 um nmero real dado. 30. Considere, num plano, uma infinidade de pontos. Sabendo-se que a distncia entre dois quaisquer desses pontos um nmero inteiro, mostre que eles so colineares.

107

...probleminhas 1. Um homem entra numa livraria, compra Pequenos Golpes, que custa 20 reais, e paga com uma nota de 100 reais. Sem troco, o livreiro vai at a banca de jornais e troca a nota de 100 por 10 notas de 10 reais. O comprador leva o livro e 8 notas de 10 reais. Em seguida entra o jornaleiro dizendo que a nota de 100 reais falsa. O livreiro troca a nota falsa por outra de 100, verdadeira. Sem o dinheiro do troco, sem o livro e sem a nota que deu ao jornaleiro, qual foi, afinal, o prejuzo do livreiro? 2. Uma pessoa, escrevendo a sucesso dos nmeros naturais (comeando pelo zero), interrompeu seu trabalho em um certo nmero. Qual esse nmero se, at parar, a pessoa escreveu 7350 algarismos?

=
3. Escreva nas casas vazias algarismos de 1 a 8 de modo que as igualdades se verifiquem, no sentido das flechas.

=
4. Qual o maior nmero que se pode escrever usando nica e exclusivamente quatro vezes o algarismo 2? 5. Construa trs cercas quadradas de modo que todas as nove ovelhas fiquem presas e separadas.

6. Preencha os quadrados com nmeros distintos de 0 a 9 de modo que os nmeros que se lem nas linhas, colunas e diagonais sejam mltiplos de 11.

108

7. Uma pessoa ctica quanto s boas intenes da humanidade afirma que 70% dos homens so desonestos, 70% so intolerantes e 70% so violentos. Se ela estiver certa, numa amostra perfeita de 100 homens, qual o nmero mnimo de pessoas simultaneamente desonestas, intolerantes e violentas? 2 8. Complete a estrela mgica com os nmeros 1, 3, 4, 5, 8, 9, 10, 12 de modo que a soma de cada linha seja igual a 26.
7
11

9. Uma loja est fazendo uma promoo na venda de balas: Compre x balas e ganhe x% de desconto. A promoo vlida para compras de no mximo 60 balas. Carlos e Daniel compraram 30 e 45 balas, respectivamente. Qual deles poderia ter comprado mais balas e gasto a mesma quantia, se empregasse melhor seus conhecimentos de Matemtica? 10. Quatro vacas negras e trs marrons do tanto leite em cinco dias quanto trs vacas negras e cinco marrons em quatro dias. Qual raa de vacas melhor leiteira, as negras ou as marrons? 11. Em um povoado vivem 700 mulheres. 4% delas usam um pendente cada uma, metade das restantes usa dois pendentes cada uma e o restante no usa adornos. Quantos pendentes usa o total das mulheres? 12. 95% da massa de uma melancia de 10 quilos constituda de gua. A fruta submetida a um processo de desidratao (que elimina apenas a gua) at que a participao da gua na massa de melancia se reduz a 90%. Qual a massa da melancia aps o processo de desidratao? 13. O nmero 15873 interessante, pois, se o multiplicarmos por um nmero de um algarismo e depois por 7, o resultado ser um nmero formado apenas pelo algarismo escolhido. Por exemplo: 15 873 x 5 = 79 365 e 79 365 x 7 = 555 555. Por qu? 14. Se gato e meio come rato e meio em minuto e meio, em quanto tempo um gato come dois ratos?

109

15. Coloque parnteses para que a expresso 5 2 x 1 + 4 6 = 5 se torne verdadeira. 16. Num concurso de televiso trs concorrentes procuram acertar o nmero de caramelos contidos numa taa de cristal. Jos diz que h 260, Maria cr que h 274 e Carlota prope que sejam 234. Sabe-se que um deles se enganou em 9 caramelos, outro em 17 e outro em 31. Pode-se deduzir qual o nmero de caramelos na taa? 17. Um nmero formado por 7 algarismos escolhidos entre os algarismos 1, 2, 3, 4, 5, 6 e 7. Se a soma de cada par de algarismos sucessivos igual soma do primeiro par e a soma de todos os algarismos 15, qual o nmero? 18. Um ciclista saiu para treinar levando consigo uma terceira roda de reposio. Durante o percurso de 60 km foi alternando as rodas de maneira que cada um rodasse uma distncia igual das outras. Quantos quilmetros rodou cada roda? 19. O Bernardo e o seu irmo Artur receberam no Natal um quebra-cabea com 2005 peas. Nesse mesmo dia, decidiram comear a constru-lo. O Bernardo desafiou o seu irmo: Vamos fazer um jogo. Voc comea por colocar uma, duas, trs ou quatro peas do quebra-cabea. Em seguida, coloco eu uma, duas, trs ou quatro peas, e assim sucessivamente. Quem colocar a ltima pea perde. Entusiasmados, preparavam-se para comear a jogar, quando, de repente, um deles exclamou: Jogue voc como jogar, eu vou conseguir ganhar!. Sabendo que ele tinha razo, qual deles disse isso e que estratgia planejou? 20. Sobre uma mesa h 137 fichas iguais, cada uma com um lado vermelho e outro azul, sendo que 10 esto com o lado vermelho para cima e as outras com o lado azul. Voc est de olhos vendados e deve separar as fichas em dois grupos, cada um com a mesma quantidade de fichas vermelhas. Voc pode virar as fichas, se necessrio. Como fazer? 21. Um destacamento de soldados precisa atravessar um rio muito profundo e sem pontes. Eles pedem ajuda a dois meninos que esto passando pelo rio num barco. Porm, o barco to pequeno que nele s cabem os dois meninos ou um soldado de cada vez. Como eles fizeram para todos os soldados atravessarem o rio?

110

22. Num reino distante quaisquer dois cavaleiros ou so amigos ou inimigos e cada cavaleiro tem exatamente trs inimigos. Nesse reino vigora a seguinte lei entre os cavaleiros: Um inimigo do meu amigo meu inimigo. Quantas possibilidades h para o nmero de cavaleiros desse reino? 23. Andr escreveu um nmero inteiro em cada crculo e depois, em cada quadrado, escreveu o resultado da multiplicao dos nmeros que estavam nos dois crculos vizinhos. Coloque na figura os nmeros que foram apagados. 24. Num hotel para ces e gatos, 10% dos ces julgam que so gatos e 10% dos gatos julgam que so ces. Aps cuidadosas observaes, conclui-se que 20% de todos os hspedes pensam que so gatos e que os restantes pensam que so ces. Se no hotel esto hospedados 10 gatos, quantos so os ces hospedados? 25. Csar e Sergio so amigos e gostam de fazer caminhadas. Enquanto Csar d 4 passos, Sergio d 5 passos, contudo, 2 passos de Csar equivalem a 3 passos de Sergio. Certo dia eles resolveram caminhar juntos, porm o Csar chegou atrasado e o Sergio j havia dado 20 passos. Quantos passos Csar teve que dar para alcanar seu amigo, que no alterou o seu ritmo at o momento do encontro? 26. Ordene os cartes 1, 2, 3 e 4 de cor cinza e 5, 6, 7 e 8 de cor branca, de modo que todas as frases resultem verdadeiras.
Os dois seguintes so brancos.

Os dois seguintes so de cores distintas.

O anterior da mesma cor que o seguinte.

H tantos brancos antes como depois.

O anterior da mesma cor que o seguinte.

O anterior cinza.

Os dois seguintes so da mesma cor.

O anterior branco.

27. Um casal tem filhos e filhas. Cada filho tem um nmero de irmos igual ao nmero de irms. Cada filha tem um nmero de irmos igual ao dobro do nmero de irms. Quantos filhos e filhas tem o casal?

111

28. Dois trens esto a uma distncia de 200 km e se aproximam um do outro com uma velocidade de 50 km/h cada um. Uma mosca voa constantemente entre as locomotivas dos 2 trens, de um pra-choque ao outro, com uma velocidade de 75 km/h at o instante em que os trens se chocam e a mosca morre esmagada. Qual foi a distncia total percorrida pela mosca? 29. Mostre que em qualquer ano existe pelo menos uma sexta-feira 13.

30. O que 100% pior do que cair um raio sobre a sua cabea?

112

Solues dos problemas

1. Soluo 1 Dado x > 1, seja N = x e seja 0 < < /2 tal que


1 2 = 2 cos cos 2 + 1 2 cos 2 + 1 = x 2 x 2 x 1 cos 2 = e = arc cos x x 2 1 N= 1 2 x cos arc cos x 2 x=

Dado x tal que 0 < x < 1, use o mtodo acima para 1/x e inverta o resultado. Soluo 2
x = tg 2 x + 1 = sec 2 1 = cos 2 x +1 1 1 x 2 1 = = cos 2 x +1 1+ x 1 x arc cos = 2 1+ x 1 1 x arc cos = 2 1+ x 1 x 1 sen arc cos 1+ x 2 = tg = x . 1 x 1 cos arc cos 1+ x 2
113

Se x > 0 ento x = tg2 para algum [0, /2[. Ento:

2. Suponhamos inicialmente que as diagonais estejam na mesma reta. Nesse caso, os tringulos ABC e ABC so semelhantes e, portanto,
A B B C = . Analogamente, v-se AB BC que os tringulos ACD e ACD so semelhantes.

D D A=A C C

A D A B D C B C = = = . Como os ngulos correspondentes dos AD DC AB BC retngulos so iguais e os lados correspondentes so proporcionais segue a semelhana dos retngulos ABCD e ABCD. Vamos agora provar a recproca. Suponhamos que os retngulos sejam

Logo,

A B B C e, como os ngulos em B e B = AB BC so retos, os tringulos ABC e ABC so semelhantes. Assim, os ngulos


semelhantes. Temos ento
AB C AB e C so congruentes e, portanto, as diagonais esto na mesma reta.

3. Como 21358ab divisvel por 99, temos que 21358ab = 99q, q natural e 0 < a, b < 9. Logo, 2135800 < 99q < 2135899 ou 21573,7373... < q < 21574,7373... . Sendo q um natural, ento, q = 21574. Assim, 21358ab = 99. 21574 = 2135826, o que implica a = 2 e b = 6. 4. A resposta no. Do teorema de Pitgoras temos a igualdade a2 = b2 + c2. Sendo a, b e c primos, no podem ser todos mpares e, como a > b e a > c, devemos ter b = 2 ou c = 2. Digamos c = 2. Teremos ento: a2 b2 = 4 (a + b)(a b) = 4 e analisando os possveis valores de a + b e a b, que so 1, 2 ou 4, conclumos que a situao impossvel.
5. Seja x o comprimento do trem e v a sua velocidade. Assim:
114

v=

171 + x x9 e v= . 27 9
E2 E1 A1 a a l-a l+a A2 a A B
5

Resolvendo x = 99 m e v = 10 m/s.

6.

A5

2l

A3

2l

2l

A4

B4 B2

B3 B1
a

B
l

Construmos A1 simtrico de A com respeito ao espelho E1, A2 simtrico de A1 com respeito ao espelho E2, A3 simtrico de A2 com respeito ao espelho E1, A4 simtrico de A3 com respeito ao espelho E2, A5 simtrico de A4 com respeito ao espelho E1. A trajetria do raio de luz a poligonal AB5B4B3B2B1B onde B1 = BA5 E1, B2 = B1A4 E2, B3 = B2A3 E1, B4 = B3A2 E2, B5 = B4A1 E1.
d , o que permite achar o ngulo de 4l + a + b incidncia, conhecendo-se a, b, l e d.

Temos tambm

tg =

7. Lema 1

AB < AC C < B
B C

115

Lema 2

A c B a C B

A c a C

AC < A B < B C

Na figura a seguir voc v o ABC e as bissetrizes BD e CE dos ngulos B e C. Seja BD = CE. Construindo o paralelogramo BDFE, temos que EF = BD = EC e portanto + = + (1) Imagine que os ngulos B e C sejam desiguais, B > C , por exemplo. Ento teramos: A B > C F > > (paralelograma BDFE) a q < (por (1)) D E DC < DF (Lema 1) q DC < BE (paralelograma BDFE) b a < (Lema 2) b a C B B < C (Contradio!) Como chegaramos tambm a uma contradio supondo inicialmente que

B < C conclumos que B = C .


8. Soluo 1 Aplicamos a identidade logab . logbc = logac e a desigualdade clssica entre a mdia aritmtica e geomtrica. imediato que

1 (log 5 6 + log 6 7 + log 7 8 + log8 5) 4 log 5 6 log 6 7 log 7 8 log8 5 = 1, 4 pois os logaritmos envolvidos so positivos. A igualdade vale se e somente se todos os termos forem iguais, mas isto no nosso caso no acontece, pois os trs primeiros logaritmos so maiores que 1 e o quarto menor que 1. Logo vale a desigualdade estrita.
116

Soluo 2 fcil provar que se a > b > 1 e x > 1, logax < logbx. Transformando os termos do lado esquerdo da desigualdade pelo modelo

log 5 6 = 1 + log 5
basta provar que

6 5

6 7 8 5 + log 6 + log 7 + log8 > 0 . 5 6 7 8 Mas isso verdade, pois em base 8, log 5
log 5 6 7 8 5 6 7 8 5 6.7.8.5 + log 6 + log 7 + log8 > log8 + log8 + log8 + log8 = log8 =0 5 6 7 8 5 6 7 8 5.6.7.8

9. Cada uma das 3 pessoas, A, B ou C, ao fazer uma afirmao, poder estar mentindo (M) ou falando a verdade (V). Como o problema envolve afirmaes das 3 pessoas, o conjunto das possibilidades ser formado por ternos ordenados dos smbolos M ou V. Assim, por exemplo, (M, M, V) representaria o caso no qual A fala a verdade e B e C mentem. Nessas condies, o conjunto das possibilidades (ou, como dizem os probabilistas, o espao amostral do experimento) seria formado pelos pontos: (M, M, M); (V, M, M); (M, V, M); (M, M, V); (M, V, V); (V, M, V); (V, V, M) e (V, V, V). Nesse espao, vamos considerar os eventos: E A fala a verdade F C diz que B diz que A falou a verdade O que o problema pede a probabilidade condicional, P(E/F), do evento E dado que ocorreu o evento F. Por definio, essa probabilidade dada por:
P( E / F ) = P( E F ) P( F ) .

claro que o evento E formado pelos pontos (V, V, V), (M, V, V), (V, M, V) e (M, M, V). Vamos, agora, identificar quais os pontos que pertencem ao evento F. Para maior clareza vamos considerar separadamente dois casos: 1o caso: A fala a verdade Nesse caso, para que F ocorra, necessrio que o nmero de mentiras ditas por B e C seja par, pois s assim elas iro se anular, permitindo que
117

C diga que B disse que A falou a verdade. Segue-se, portanto, que, nesse caso, os pontos de F so (V, V, V) e (M, M, V). 2o caso: A mente Um raciocnio anlogo mostra que F s ir ocorrer se uma e apenas uma das pessoas B ou C mentir. Logo os pontos de F, nesse caso, so (M, V, M) e (V, M, M). Segue-se, portanto, que o evento F formado pelos quatro pontos, (V, V, V), (M, M, V), (V, M, M) e (M, V, M). Admitindo-se, agora, a independncia entre as afirmaes das 3 pessoas, teremos:

1 1 1 2 2 1 1 2 2 2 1 2 13 P( F ) = + + + = 3 3 3 3 3 3 3 3 3 3 3 3 27
Por outro lado, fcil ver que o evento E F formado pelos pontos (V, V, V) e (M, M, V) e portanto: P(E F) = 5/27. Segue-se finalmente que: P ( E / F ) =

5 / 27 5 = . 13 / 27 13
2 1 e que: = 3 2

10. Observe inicialmente que cos


cos

2 14 8 6 8 + cos = 2 cos .cos = cos 9 9 9 9 9

2 8 14 + cos + cos = 0 . Desenvolvendo o quadrado 9 9 9 dessa expresso vemos que aquilo que queremos calcular igual a:

Segue-se que cos

1 2 8 14 cos 2 + cos 2 + cos 2 2 9 9 9

Usando a relao cos 2a = 2 cos2a 1, essa expresso se transforma em:


1 4 16 28 cos + cos + cos + 3 4 9 9 9

Por outro lado,

cos
118

4 28 16 12 16 16 2 2 + cos = 2 cos = 2 cos 1 = cos cos 2 cos 9 9 9 9 9 3 9

de onde se segue que:


1 4 16 28 3 cos + cos + cos + 3 = . 4 9 9 9 4

11. a) Seja ax2 + bx + c = 0, com a, b e c inteiros e a 0. Suponhamos b2 4ac = 23. Segue-se que b2 4ac = 23 mpar e, portanto, b mpar. Se b mpar, b 1 e b + 1 so pares, e, portanto, b2 1 = (b + 1)(b 1) mltiplo de 4. Mas b2 1 = 4ac + 22 e, como 22 no mltiplo de 4, segue-se que b2 4ac no pode ser igual a 23. b) 1) claro que a deve ser inteiro, uma vez que a soma das razes a. 2) a2 24a deve ser o quadrado de um nmero inteiro. Suponha a2 24a = n2, com n inteiro. Como a2 24a = (a 12)2 144, temos (a 12)2 = 122 + n2. Essa equao admite a soluo trivial n = 0 e, nesse caso, a = 0 ou a = 24. Se n diferente de zero, n e 12 podem ser pensados como os catetos de um tringulo retngulo cuja hipotenusa (a 12). O leitor deve verificar que existem 4 tringulos pitagricos com um cateto igual a 12 (veja, por exemplo, RPM 47, p. 49). So eles: (5, 12, 13), (9, 12, 15), (12, 16, 20), (12, 35, 37) para para para para n = 5, n = 9, n = 16, n = 35, a = 1 a = 3 a = 8 a = 25 ou ou ou ou a = 25 a = 27 a = 32 a = 49

Se acrescentarmos os valores correspondentes a n = 0, teremos exatamente dez valores de a que satisfazem as condies do problema.

119

12. Decorre da hiptese que cos x = 0 se, e somente se, cos (x + y) = 0 (verifique!). Ou seja, se um dos membros da igualdade que queremos mostrar no estiver definido, o outro tambm no estar. Suponhamos, ento, que cos (x + y) 0 e cos x 0. Ora, de 5 sen y = sen 2x cos y + sen y cos 2x e como 1 2 tg x 1 tg 2 x cos y 0 (pois, sen y ), sen 2 x = e cos 2 x = temos 5 1 + tg 2 x 1 + tg 2 x 5tg y = 2 tg x 1 tg 2 x + tg y o que implica 1 + tg 2 x 1 + tg 2 x

5tg y + 5tg y tg 2 x = 2 tg x + tg y tg y tg 2 x .
E, como de cos(x + y) 0 tem-se 1 tg x tg y 0, podemos deduzir
tg x + tg y 6 3 = tg x , donde, finalmente tg ( x + y ) = tg x. 1 tg x tg y 4 2

13. Traar AE // BC. Pelo ponto mdio F de AE, traar FG // MD. (Quando BC // AD, F e G coincidem.) Afirmamos que MG a soluo. De fato, pela construo, rea MBADF = rea MFDC .
B M C E D

H G

Por meio de MG, estamos tirando da rea direita o tringulo MFH e acrescentando o tringulo HGD para obter MGDC. Porm, rea MFH = rea HGD, pois FG // MD e MHD comum aos tringulos MGD e MFD. Logo, rea MGDC = rea MFDC. Do mesmo modo, rea MBAG = rea MBADF. 14. Se imaginarmos que todas as bolas sero retiradas da urna, existiro ao todo n! configuraes numricas possveis para esse experimento. O nosso problema contar em quantas dessas configuraes teremos, na k-sima retirada (2 < k < n), pela primeira vez, uma bola cujo nmero seja maior do que o de todas as anteriores.
120

Em primeiro lugar, vamos observar que o evento no qual estamos interessados no depende dos particulares nmeros das bolas que sairo nos k primeiros lugares, mas apenas da ordem em que eles sairam.

n Existem escolhas possveis para esses nmeros. Uma vez fixados k esses k nmeros, para que o evento ocorra, duas condies precisam ser satisfeitas: 1) A bola com o maior nmero (entre os k escolhidos) deve sair na k-sima retirada. 2) A bola com o 2o maior nmero deve sair na primeira retirada. Observe que a primeira condio garante que o processo pra na k-sima retirada, enquanto que a segunda garante que ele no pra antes da k-sima retirada. As outras k 2 bolas podem ocupar qualquer posio, o que nos d um total de (k 2)! configuraes possveis. De maneira anloga, as n k bolas que sairo aps a k-sima retirada podero aparecer em qualquer ordem, o que nos d (n k)! possibilidades. Segue-se que o nmero total de configuraes nas quais, pela primeira vez, na k-sima retirada, aparece uma bola cujo nmero maior do que todas as n anteriores, vale (k 2)!(n k )! e, portanto, a probabilidade desse k
n (k 2)!(n k )! 1 k = para 2 k < n. n! k (k 1)

evento

O caso k = n tem que ser analisado em separado, pois o processo para no instante n no apenas quando as condies 1 e 2 esto satisfeitas para k = n (a probabilidade de que isso ocorra 1/n(n 1)), mas tambm quando elas no so satisfeitas em nenhum instante. Esta segunda hiptese ocorre se, e somente se, a bola com o nmero n sair na primeira retirada. (A probabilidade de isso ocorrer , obviamente, l/n. ) Para k = n, a probabilidade desejada vale

1 1 1 + = . n(n 1) n n 1

121

15. Sejam O o centro da circunferncia dada de raio R, M o ponto mdio de BC e P OM tal que OM = 3PM. Para cada A na circunferncia dada considere G o baricentro do ABC.
AM GM = 3 , ento, GP // OA e Como PM = OM 3

OA R = . Assim, G pertence 3 3 circunferncia de centro P e raio R/3. GP = 1 1 no segmento BC com BG = BC (GC = BC ). 3 3

O G P B

Observe que nos casos degenerados onde A = B (A = C) consideramos G

Logo os baricentros dos tringulos ABC pertencem circunferncia de centro P e raio R/3. Reciprocamente, cada ponto dessa circunferncia baricentro de algum ABC com A na circunferncia dada. (Por qu?) 16. Como x3 y3 = (x y)(x2 +xy + y2) e 602 = 2.7.43, devemos resolver o sistema xy=A x2 + xy + y2 = B. Como 0 < y < x < x2 < B, basta experimentar os pares (A, B) com A < B tais que A.B = 602: (1,602); (2, 301); (14, 43) e (7, 86). Somente o par (2, 301) fornece solues inteiras, de onde temos que as solues positivas so 11 e 9. 17. Os tringulos do enunciado podem ser considerados justapostos como ABD e AFG da figura. Queremos mostrar que

BD AB = . FG AG Aplicado o teorema da bissetriz interna


ao tringulo ABC, temos

A B C
y s

BD CD = . AB AC

D F

122

Como r // s, temos

CD FG = . Logo, BD = FG . AC AG AB AG

18. Vamos observar inicialmente que o conjunto A tem


10 = 252 elementos. 5

a) Sorteada a primeira combinao nos 251 elementos restantes, existe apenas uma combinao que no tem elementos em comum com a combinao sorteada. Segue-se que a probabilidade pedida vale 1/251.
5 b) Efetuando o 1o sorteio, existem = 5 grupos de 4 elementos da 4 combinao sorteada que podem ser combinados com qualquer um dos 5 elementos que no pertencem a ela para formar uma combinao que tenha 4 elementos em comum com a sorteada. Segue-se que a probabilidade pedida vale 25/25l.

19. Sejam h = altura da pirmide, R = raio da circunferncia circunscrita a um pentgono regular convexo de lado a. Ento a =

R 4a 2 10 2 5 ou = R2 . 2 10 2 5 Temos que a, h e R formam um tringulo retngulo e assim


h2 = a 2 R 2 = a 2
( 50 10 5 ) . 10

4a 2 a 2 (50 10 5 ) = 100 10 2 5

Logo, h = a 20.

Soluo 1 Quando m/n = 1, o lugar geomtrico dos pontos P do plano tais que

PA m = a mediatriz de AB, pois PB n

PA = 1 P mediatriz de AB). PB
123

Quando m/n 1, consideraremos, sem perda de generalidade, o caso m > n > 0. a) A construo ao lado mostra que existem dois pontos P1, P2 da reta AB tais que m Pi A m n = , i = 1, 2. PB n i A P P n B 1 2 A verificao se faz atravs de semelhana de tringulos. b) Os pontos P1, P2 obtidos em a) so os nicos da reta AB tais que
Pi A m = . (Por qu?) PB n i

c) Se P um ponto fora da reta AB tal que PA = m , mostraremos que PB n P pertence circunferncia C que passa por P1 e P2 e tem dimetros P1 P2. De fato: Sendo PC bissetriz interna P do ngulo APB , segue do teorema da bissetriz interna:
AC PA m = = . BC PB n

Sendo PD bissetriz externa do ngulo APB , segue do teorema da bissetriz externa:

AD PA m = = . BD PB n

Como C e D so pontos da reta AB tais que

AC AD m = = , ento BC BD n por b), {C, D} = {P1, P2} (A existncia do ponto D garantida por termos PA PB).

Logo, P pertence circunferncia C, j que as bissetrizes interna e externa so perpendiculares.

124

d) Reciprocamente, dado um ponto P na circunferncia C, mostremos que PA = m e ento concluiremos que o lugar geomtrico dos pontos PB n P a circunferncia C. Tracemos por B a reta r paralela a AP. Temos:

AP AP m = = P (de APP2 BB 2 ) BB BP2 n AP AP m = 1= P (de APP BB" 1 ). 1 BB" BP n 1


Logo, BB = BB, isto , PB mediana do tringulo retngulo PBB. Isso implica (verifique!) PB = BB = BB e, portanto, PA = m . PB n

P B A B
ABmn , obtido de: m2 n2

P 1

P 2

Observao: O raio da circunferncia C


AP = AB P B = AB AP 1 1 1

n ABm AP = , 1 m m+n m ABn . BP2 = AP2 AB = BP2 AB BP2 = mn n

Soluo 2 O problema clssico em Geometria Analtica, com a soluo x2 + y 2 ( x b) 2 + y 2


2

m , n de onde

n2(x2 + y2) = m2(x2 + y2 2bx + b2) ou x (n2 m2) + y2(n2 m2) + 2m2bx m2b2 = 0.

A = (0, 0)

B = (b, 0) x

Se n = m, obtemos a mediatriz; se n m, obtemos a circunferncia.

125

21. Sejam a e b, respectivamente, os algarismos das dezenas e das unidades do nmero procurado. Como a 10 + b + b 10 + a = 11(a + b) um quadrado perfeito, ento 11 um divisor de a + b. Observando que 1 < a + b < 18, resulta a + b = 11. Verificando as possibilidades para a e b, encontramos os seguintes nmeros: 29, 38, 47, 56, 65, 74, 83 e 92. 22. Considere a, b, c retas paralelas coplanares. A seguinte construo pode ser feita usando apenas rgua e compasso. Fixamos A a e fazemos uma rotao de 60o, no sentido anti-horrio, da reta b em torno de A, obtendo uma reta b que corta a reta c no ponto C.

A
90-a 90-a a -30 a 60
0

H
b

Fazemos a rotao, no sentido horrio, do ponto C em torno de A, obtendo na interseo com b o ponto B b. Os tringulos retngulos AHC e AHB so congruentes, uma vez que AH = AH e AB = AC. Logo, ACH = ABH = , que implica B AC = 90o + ( 30o ) = 60o , de modo que o ABC eqiltero. 23. Um grupo de 2n pessoas formado por n homens e n mulheres.
2n Existem maneiras de escolhermos um conjunto de n pessoas desse n grupo. Vamos determinar em quantos desses conjuntos existem exatamente k homens. Para isso vamos observar que os k homens podem ser escolhidos n n de maneiras e as n k mulheres de maneiras. Segue-se k n k

126

que, para k = 0, 1, ..., n o nmero de escolhas de n pessoas que contm n n n exatamente k homens ser dado por = . k n k k n n n 2n Conclui-se portanto que 0 + 1 + K + n = n . 24. Considere o produto dos 3 nmeros pares consecutivos: 88.106 < (x 2) x (x + 2) = x3 4x < x3 Temos: 85184000 = 4403 < 88.106 < 4503 = 91125000 Trs nmeros pares consecutivos podem terminar em: 0, 2, 4 2, 4, 6 4, 6, 8 6, 8, 0 8, 0, 2. O nico produto dessas triplas que termina em 2 4.6.8 = 192, logo os nmeros so 444, 446, 448 cujo produto 88714752. Portanto, os algarismos procurados so 7, 1, 4, 7, 5. 25. Sejam r1, r2, ..., r7 as 7 razes da equao. Temos ento: r1 + r2 + ...+ r7 = 2 r1r2 + r1r3 + ... + r6r7 = 3 Segue-se 4 = (r1 + r2 + ... + r7 )2 = r12 + r22 + ... + r72 + 6 e, portanto,
2 2 2

r
1

= 2 , o que mostra que nem todas as razes podem ser reais.

26. Sejam xi R, i = 1;..., 100 tais que P(xi) = 0 para todo i entre 1 e 100. Suponhamos, por contradio, que xi < 7, i = 1, ..., 100. Ento como P(x) = (x x1)(x x2)...(x x100), temos que: 1 < P(7) = (7 x1)(7 x2)...(7 x100). Logo,
127

1 = 100 1 < 100 (7 x1 )(7 x2 )K (7 x100 ) 7 x1 + 7 x2 + K + 7 x100 1< 100


Mas como

700 xi
i =1

100

100

.
98

P ( x) = ( x x1 )K ( x x100 ) = x100 600 x99 + a j x j ento


j =0

x
i =1

100

= 600. Logo, 1 <

700 600 = 1 (absurdo!). 100

27. Como o pentgono tem todos os lados iguais, basta mostrar que ele inscritvel. 1 o caso: os ngulos congruentes so consecutivos A B E . Os quadrilteros BAED e EABC so trapzios issceles, logo inscritveis e a circunferncia que passa pelos pontos B, A, E e D tambm passa pelos pontos E, A, B e C.
A B E B A E

2o caso: os ngulos congruentes no so consecutivos A B D . O AED issceles, logo os seus ngulos da base AD so congruentes e, portanto, o quadriltero ABCD um trapzio issceles. Portanto, B C , recaindo no 1o caso. 28. Se existe uma raiz racional, temos b2 > 4ac e tambm temos que b2 4ac um quadrado perfeito m2. Sendo b mpar, b2 mpar e, como 4ac par, temos b2 4ac mpar, implicando m2 mpar, que, por sua vez, implica m mpar. Como b2 m2 = 4ac e a diferena dos quadrados de dois nmeros mpares sempre um mltiplo de 8 (verifique!), conclui-se que 4ac mltiplo de 8. Mas, sendo a e c mpares, 4ac no um mltiplo de 8; logo, a equao ax2 + bx + c = 0 no tem razes racionais.

128

29. Suponha que a afirmao seja falsa, isto , os pontos do plano foram pintados usando-se trs cores A, B e C e todos os segmentos de comprimento k possuem extremidades de cores diferentes. Seja O um ponto do plano e, sem perda de X generalidade, suponhamos que ele seja da cor A. k k N M Sejam 1 e 2 as circunferncias de centro O e raios respectivamente k e k 3 .
k k

O Todos os pontos de 1 tero sido pintados de cor B ou C, pois, caso contrrio, haveria um raio (segmento) 1 de cujas extremidades seriam ambas da cor A. Tome um ponto X em 2 e pontos M e N em 1 satisfazendo:
MN = k = MX = NX .

O valor do raio de 2, k 3 , garante a existncia do losango OMXN . Assim, M e N possuem cores diferentes (B e C) e X deve ter a cor A. Como todos os pontos de 2 podem ser obtidos dessa forma, provamos que todos eles esto pintados com a cor A, o que uma contradio, pois sobre 2 existem cordas de comprimento k. 30. Seja S o conjunto dado. Suponhamos que seus pontos no so colineares. Sejam A e B dois deles. Dado P S, temos PA PB AB com AB = n N, logo P pertence a uma das n + 1 hiprboles de focos A e B dadas por PA PB = k com k N, 0 < k < n. Observamos que, para k = 0 e para k = n (casos nos quais o ponto P pertence mediatriz de AB ou reta AB), temos hiprboles degeneradas. Considere C S tal que A, B, C no sejam colineares. Todo ponto P S pertence a um outro feixe de m + 1 hiprboles de focos A e C, onde AC = m N. S est contido na interseco dos dois feixes (finitos) de hiprboles. A interseco de duas hiprboles sempre finita, se elas forem distintas. Como os dois feixes no tem nenhuma hiprbole em comum (convena-se disso), seguiria que S finito. Contradio!

129

130

Respostas dos ...probleminhas


1. 100 2. 2114 3. Comeando no canto superior esquerdo e girando no sentido antihorrio: 6, 3, 2, 4, 8, 7, 1, 5. 4. 22 5.
22

19. Como 2005 = 4 + 400 5 + 1, o primeiro a jogar, Artur, pode colocar 4 peas e, em seguida, coloque o Bernardo o que colocar, Artur, pode jogar de modo que, na sua jogada e na anterior, o total de peas colocadas seja 5. Com essa estratgia, Artur vence e foi ele quem disse a frase. 20. Separe um grupo de 10 fichas quaisquer e vire-as. Este grupo e o grupo das fichas restantes ficam com o mesmo nmero de fichas vermelhas. 21. O menino A fica na margem oposta margem na qual esto os soldados e o menino B leva o barco at os soldados. O primeiro soldado atravessa o rio e o menino A traz o barco de volta. Os dois meninos atravessam o rio, o menino A fica e o menino B leva novamente o barco at os soldados. O segundo soldado atravessa o rio e... 22. 4 ou 6 23. No sentido horrio, a partir do 85: 5, 45, 135, 15, 8 e 17. 24. 70 25. 80 passos 26. 2, 5, 3, 7, 4, 1, 6 e 8 27. 4 filhos e 3 filhas 28. 150 km 30. Cair um dimetro.
131

6.

2 8 6

7 0 4

5 3 9

7. 10 8.
2
1 11 3 5 9 10 8

7 4 12

9. Daniel 10. As vacas marrons. 11. 700 12. 5 kg 13. Porque 15873 7 = 111 111. 14. Em 3 minutos. 15. (5 2) (1 + 4 6) = 5 16. 243 17. 3131313 18. 40 km

RPM
Sociedade Brasileira de Matemtica Presidente: Joo Lucas Marques Barbosa Vice-Presidente: Suely Druck Secretrio-Geral: Orlando Lopes Tesoureiro: Joo Xavier da Cruz Neto Comit Editorial da RPM Alcila Augusto editora responsvel Ana Catarina P. Hellmeister editora executiva Alberto Carvalho P. de Azevedo Antonio Luiz Pereira Eduardo Wagner Elon Lages Lima Geraldo vila Jos Paulo Q. Carneiro Paulo Cezar Pinto Carvalho Renate G. Watanabe

RPM Revista do Professor de Matemtica


Caixa Postal 66281 CEP 05311-970 So Paulo, SP www.rpm.org.br rpm@ime.usp.br telefone/fax: (11) 3091 6124

132

Anda mungkin juga menyukai